全科副主任医师考试试题要点

时间:2019-05-14 11:15:26下载本文作者:会员上传
简介:写写帮文库小编为你整理了多篇相关的《全科副主任医师考试试题要点》,但愿对你工作学习有帮助,当然你在写写帮文库还可以找到更多《全科副主任医师考试试题要点》。

第一篇:全科副主任医师考试试题要点

1.治疗严重上消化道出血首要:迅速补充血容量。

2.大肠癌最好发:直肠与乙状结肠。克隆病好发:末段回肠和右半结肠。3.慢性胃炎特异症状:无 都不对。

4.胃溃疡好发部位:小湾胃角附近。球后溃疡好发:十二指肠降部近端的溃疡。5.胃癌血转最常见:肝脏。

6.消化性溃疡的病里损伤深度浅至少达:粘膜肌层。

7.食管癌放疗用于:上段食管癌和不能切除的中下段食管癌。8.肝硬化早期表现:不对是少数有腹水。9.胃溃疡X表现:胃轮廓之韩影。

10.治疗胆汁反流胃炎B胃炎选:复方铝碳酸镁

胃复安 11.食管癌最常见类型:鳞癌 12.不是克隆病临床表现:黄疸

13.早期胃癌:不超过粘膜下层,不论有无淋巴转移。14.导致为题萎缩性胃炎病因:口腔及咽部慢性病灶。15.胃癌X:充盈缺损。

16.小细胞未分化癌说法错:痰癌细胞检出率低。

17.原发肺癌引起症状中,错误是:发作性胸闷,气喘。18.慢喘急发主要治措施:控制感染。19.慢支临床分型:单纯和喘息型

20.严重支气管哮喘发作病人,重要祛痰方法:补液。21.坐位误吸,肺脓肿好发:下叶后基底段。22.确诊支气管扩张,胸部薄层高风辨率CT.23.单纯性鼾症,小于60DB 24.大叶性肺炎:混合型呼吸困难。

25.肺癌组织细胞学分类:鳞,腺癌,大小细胞未分化癌。

26.睡眠呼吸暂停综合征,不会出现氧饱和度下降。睡眠过程鼻气流停止》=10sm 27.慢性支气管炎早期x表现,是无特殊征象。28.粉红色泡沫样痰是:急性肺水肿。

29.慢性支气管炎可分为:急性发作期,慢性迁延期,临床延缓期。30.支气管扩张最有意义体征是:局限性湿罗音。

31.急性肺水肿在:肺功能不全情况下禁用肺功能不全。32.支气管哮喘典型症状:发作性呼吸困难。33.肺癌最常见的早期症状是:咳嗽。

34.阻塞性肺气肿实验室意义最大:肺残气容量占肺总量百分比。35.胸闷,肺压缩50%,治疗首选:人工

36.哮喘发作:呼气相为主哮鸣音,呼气相延长。37.慢性支气管炎主要依据:临床症状。

38.为防止肺性脑病方法不对:5%碳酸氢钠纠正酸中毒。39.哮喘最适宜治疗方法:注射氨茶碱。40.支气管扩张临床表现错误:急性咳嗽。

41.COPD呼吸性酸中毒时,不宜用巴比妥类药物。COPD的主要特征:气流阻塞。42.不发展肝硬化的病毒:甲型和戊型肝硬化。43.新生儿肺炎可分为:吸入性和感染性肺炎。44.肺癌化疗恶臭味感染:厌氧菌。45.支气管哮鸣音应当为:不给任何治疗也会缓解。46.关于肺炎病因学分类:不包括药物性肺炎。47.对于判断哮喘严重程度更为准确:奇脉。48.慢性支气管炎呈胶冻状痰:克雷白杆菌肺炎。

49.胸部x片检查见右上肺锁骨上下区有云蓄状阴影,密度不均,是浸润性肺结核。50.重度哮喘发作激素:静滴。

51.支气管哮喘不典型症状:胸闷或干咳。52.MRSA首选:青霉素。

53.慢性支气管炎的慢性迁延期:时间延长2个月以上。54.干性支气管扩张,只有反复咳血,一般无咳嗽咳痰。

55.大片实变影,叶间隙下坠,最可能诊断,克雷白杆菌肺炎。56.慢性支气管炎临床表现错误是:疾病晚期可有肺实变体征。

57.引起慢性肺心病肺动脉高压原因:高碳酸血症所致肺血管阻力增加。58.渗出性胸腔积液诊断:胸液蛋白/血清蛋白比值》0.5 59.咳大量痰,静后分层:支气管扩张。

60.支气管哮喘急性发作,最简便客观检查方法:PEF测定峰流速。61.慢性支气管炎表现,多在肺部罗音,多在后背下部。62.耳鼻咽喉:急性会厌炎主要症状:吞咽困难。63.急性脓毒性咽炎:A组乙型链球菌。

64.最常见简便喉镜检查方法:间接喉镜检查。

65.鼻咽癌颈部淋巴结转移,首发:颈部深淋巴结上群。66.变态反应性鼻炎:I型变态反应。

67.星锁征或灯塔证是急性化脓性中耳炎。68.气管切开无价值:在气管隆突水平。69.鼻呼吸区粘膜纤毛运动方向:从前向后。70.上颌穿刺:距下鼻甲前端1.5cm.71.提示鼓室积液的鼓室压图,B型。72.鼻窦恶性肿瘤最多见是:上额窦。73.喉癌最好发部位:声带。

74.吸引器产生负压一般不超过:24KPA。75.常规气管切开术应切:第3.4环。76.鼻腔粘膜温度应保持在:33-35度。77.梅尼病主要病理改变:蜗管扩张。78.咽喉直接相同的区域是:梨状窝

79.鼻腔鼻窦癌最多见:磷癌。喉癌:最多见:磷癌。80.溶血性链球菌引起的急性乳突炎还称:出血性乳突炎。81.鼻咽癌颅内转移最常见的途径:破裂孔。82.骨疡性中耳炎又称:肉芽型。

83.怀疑食管异物首先解决:食管钡餐X检查。84.治疗肥厚性鼻炎硬化剂是:80%甘油。

85.急性鼻炎主要致病微生物是:鼻病毒和腺病毒。86.鼻窦炎和鼻息肉最佳治疗方法:鼻内镜手术。87.变态反应性鼻炎中发挥主要介质作用是:组胺。88.慢性肥厚性鼻炎:鼻甲肥厚,桑葚样改变。89.出生后就有可能感染是:上额窦及筛窦。

90.国际阻塞性呼吸暂停,是指7小时睡眠中暂停30次以上。91.BEZOLD指:耳源性颈侧脓肿。92.鼻咽喉癌好发部位:咽隐窝。93.食管异物最好发部位:食管入口。

94.正常的鼻粘膜上皮为:假复层纤毛柱状上皮。

95.口腔:与智齿冠周炎无关的是:智齿牙根的形态和数量。96.口腔健康促进不包括:教育手段。97.患龋频率最高的是:磨牙。98.牙周炎和牙龈炎区别:牙周袋。

99.牙周基础治疗不包括:牙龈切除术。基础治疗是:洁治术和刮治术 100.小学生重点预防龋坏是第一恒磨牙。

101.易患口腔粘膜白斑:有烟酒嗜好的中年男性。102.致龋能力最强的是:蔗糖。103.牙髓血运主要来自:牙槽动脉。

104.去除牙齿邻面菌斑的最佳方法:牙线。机械性控制菌斑方法:刷牙。105.口腔粘膜上皮:复层鳞状上皮。

106.口腔粘膜病最常见的疾病是:复发性口腔溃疡。107.常见盖髓剂:氢氧化钙。

108.判断牙髓状况:温度刺激试验。109.形成牙本质组织是牙髓。

110.复发性口腔溃疡少见于:硬腭和龈

病程:1-2W 111.与龋病发病有关的最主要细菌是:变形链球菌。112.正常龈沟深度为:3CM 113.残髓炎疼痛特点:多为自发性剧痛。

114.判断有无牙周炎重要指证:是有无附着丧失。115.牙痛,口含冰块缓解是:急性牙髓炎化脓期。116.治疗牙龈炎的首选方法:消除牙石。117.目前已肯定的防龋方法:氟化物防龋。118.食物中天然含氟较高的是:茶叶。

119.口腔预防医学和口腔临床医学的相同之处是:评估或评价。120.急性牙髓炎的诊断步骤:先问珍再查牙,后温度测验。121.禁用皮质激素:单纯疱疹和带状疱疹。122.窝沟龋的早期表现是:损害部位为白俄色。123.x线拍牙槽骨吸收最好的是胎翼片。

124.重型口疮的损害特点是:单个溃疡,基底硬结。125.龋病的好发牙位:B 126.对急性牙髓炎最具诊断特点:自发性阵发性痛,有夜间痛。127.牙周疾病二级预防概念不包括:恢复缺失牙。128.隐诺性龋是指:轴板下龋。129.定期牙周洁治是6-12月。130.轴质龋的特征表现是:白俄斑

131.智齿冠周炎的过氧化氢(双氧水)浓度是:1-3%。132.牙遇甜酸痛,小米大小,充填是复合树脂。133.牙髓的感觉神经:三叉神经。

134.口腔白色念珠菌有效药:碳酸氢钠。135.正常龈沟深度是:1-2cm 136.饮用水适宜的附浓度是:0.7-1ppm 137.白斑的临床类型是5种。

138.急性牙髓炎区别深龋是:有自发痛。139.造成牙周炎的牙菌斑主要是:龈下菌斑。140.冠周炎多发:38 48 141.急性化脓性牙髓炎不包括冷加剧疼痛 142.口腔白斑病:白斑不能擦去。

143.人体三大唾液腺:腮腺,额下腺,舌下腺

144.窝沟浅龋的治疗可采用:充填法和窝沟封闭法。

145.眼科:原发性青光眼可分:闭角型青光眼和开角型青光眼。146.紫外线损伤主要是:造成角膜上皮脱落。147.房水产生于:睫状突。

148.眼部碱性损伤重于酸性是因为:碱性溶解脂肪和蛋白质。149.细菌性角膜溃疡发展最迅速严重是铜绿假单胞性角膜溃疡。150.屈光性近视主要是:角膜或晶状体曲率过大。151.重症沙眼可导致失眠或视力减退。152.儿童验光滴阿托品是睫状肌麻痹 153.原发性青光眼占我国致盲第三位。

154.青光眼主要导致哪个部位损害:视神经损伤。

155.电光性眼炎主要治疗原则:止痛。属于辐射伤。角膜热烫伤治疗原则:防止感染。156.流行性出血性结膜炎:肠道病毒。157.沙眼的病原体:衣原体。

158.眼睛屈光系统:角膜房水,晶状体,玻璃体。159.正常眼压范围:10-20mmhg.160.单纯白内障手术后是屈光状态是高度远视。161.屈光参差:5D以上。

162.酸性物质致伤的特点是能使蛋白质凝固。163.取角膜异物遵循:严格无菌操作。

164.糖尿病性视网膜病变分期:I:微动脉瘤或并有小出血点。III:白色软性渗出或并有出血斑。

IV:眼底有新生血管或并有玻璃体积血。V:眼底有新生血管和纤维增生。

VI:眼底有新生血管,纤维增生和视网膜脱落。

165.老年性白内障手术时机:白内障影响工作和生活。166.急性结膜炎潜伏期:1-3天。

167.急性闭角型青光眼的诱发因素:暗室停留时间过长。分期:6期。168.正常人一般眼压波动范围:《8mmhg

正常:10-21mmhg 169.老年性白内障:药物不能使晶状体变浑浊。170.原发性闭角型青光眼:我国最常见。

171.屈光不正包括:近视,远视,散光,屈光参差。172.眼睛局部用药疗程最少为:10-12W 173.角膜屈光系统组成部分:角膜,房水,晶状体,玻璃体。174.眼球光学系统组成:角膜,晶状体,玻璃体

175.继发性青光眼眼压升高原因:眼球内容增加和房水排除多阻。176.眼部化学烧伤分为:接触期,扩散期,溃疡期,瘢痕期。177.眼部重度碱烧伤并发症:金球粘连,眼球萎缩,角膜白斑。178.皮肤:银屑病:寻常型,脓疱型,关节病型,红皮病型。179.银屑病按病程分期:进行期,稳定期,消退期。180.尿布皮炎不属于变应性接触性皮炎。181.非免疫性荨麻疹:延迟性压力性荨麻疹。182.风团形成主要因素:真皮浅层急性水肿。183.冻疮皮损一般不出现:风团。184.接触性皮炎是:迟发型变态反应。185.可累及关节的病是:银屑病 186.糊剂是粉剂加在凡士林中。187.酊剂是不挥发性药物溶于酒精。188.二期梅毒的皮疹:扁平湿疣。189.软化痂剂型:油膏。

190.慢性肥厚炎症皮损:软膏,糊膏,油膏。

191.常见HNA病毒是:风疹病毒 DNA病毒:传染性软疣病毒。PHV:尖锐湿疣 HSV病毒:生殖器疱疹。

192.艾滋病的癌瘤是:卡波西肉瘤。

193.接触性皮炎呈递细胞:朗格罕氏细胞。

194.引起湿疹的主要原因:过敏体质。婴儿湿疹:丘疱疹和糜烂。195.湿疹的水疱位于:表皮内。196.先天梅毒:不对:经父亲传染。

197.银屑病发病与遗传有关。酒渣鼻禁用:激素。198.反应停:光敏性皮肤病有效。

199.原发性生殖器疱疹潜伏期:2-14天。200.常见急症与急救:大便洗肉水样:嗜盐菌食物中毒;肉毒杆菌中毒:对称性颅神经损伤。201.酒精纳络筒:缩短昏迷时间。

202.服毒后几小时洗胃最有效:4小时。203.胸外心脏按压深度:4-5cm(至少5cm)

204.cohb中毒:30%-50%。Cohb在100%氧气中半衰期为1小时。205.不支持骨折诊断:异常活动。206.包扎目的:止血止痛等全部。207.吞服强酸禁止碳酸氢钠中和。208.心梗室颤应:非同步直流电除颤。209.淹溺者心脏骤停的原因:低氧血症。

210.支气管哮喘分型:内源性外源性混合型哮喘持续状态。211.肺心病二氧化碳潴留原因:通气不足。212.止血带止血原则不超过:1小时。

213.心搏骤停最简便最可靠的诊断指标:大动脉搏动消失。214.危害最大的电流是:低频交流电。215.电阻最大的组织是:骨。通过人体电流大小取决于皮肤电阻大小。216.锤击复律最好在1.5分钟内。

217.最多见的休克类型:中毒感染性。

218.腹腔内脏器损伤最具有价值:腹腔穿刺。

219.咯血少于100少量咯血,100-500中量咯血;500-1000或一次性100ml大咯血。220.过敏性休克变应反应为I型。

221.成人心外除颤采用:200w.s电能。

222.发作性吸气性呼吸困难:最常见气管异物。主要病变部位:大气道。223.肺性脑病和高血压脑病区别有无发绀。224.两人心肺复苏:比例15挤压:1呼吸 225.超过35度,靠蒸发。常温靠辐射。

226.治疗高血钾引起的心脏骤停,是氯化钙。心脏骤停复苏后最易出现脑损伤。227.心源性休克最好治疗方法:主动脉内气囊反博。228.呼气性呼吸困难:急性左心衰竭。

229.急性呼吸衰竭诊断主要依靠:血气分析。230.心肺复苏最常用:仰头举额法。

231.副溶血性弧菌是食堂食用虾水样泻。鱼虾。232.心肺复苏,潮气量500-600ML。233.腹泻粪便粘液腥臭味是肠阿米巴病。234.一氧化碳中毒表现不对是:贫血。

235.儿科:麻疹传染最强:前驱期。麻疹皮疹一般出现在:发热后3-4天。236.新生儿核黄疸早期指证:嗜睡肌张力下降。237.儿童反复上呼吸道感染指:每年6次以上。

238.21三体综合征:46,XX,T染色体畸病:染色体易位。239.新生儿肺炎最可能出现症状:口吐白沫。240.幼儿急疹:退热后出疹。

241.流行性腮腺炎:腮腺红肿热痛。242.新生儿溶血病:不发热。

243.小儿重度脱水,失水量大于10%。244.健康查体血红蛋白查:大于6个月。

245.支气管肺炎重要体征:鼻翼扇动,三凹征,肺部中小型湿罗音。246.小儿腹泻多发:6月-2岁。

247.小儿肺炎最常见:支气管肺炎。

248.脑脊液:蛋白增高,糖氯化物下降:结核性脑膜炎。249.诱发加重结核病:麻疹。

250.重症肺炎失衡:呼吸酸加代酸。251.APGAR:不属于评分标准:血压。252.麻疹患者应隔离至出院:5天。

253.中度脱水累计损失:50-100ML/KG 大约需8-12小时 254.水痘不对:离心性分布。只发生在皮肤是不对的。255.新生儿肝炎和胆道闭锁鉴别:大便颜色。256.不是小儿腹泻主要原因:血中IGM含量低。257.等渗性脱水,第一天补液1/2张

258.ABO血型不合,换血最适合是:O型血细胞AB型血浆。259.咽结合膜热病原体:腺病毒。

260.4:3:2混合液:4份生理盐水和3份10%葡萄糖和2份1.4%碳酸氢钠。261.猩红热皮疹特点:皮疹细小呈鲜红色。

262.新生儿黄疸:出生后2-5天,10-14天消退。263.治疗新生儿破伤风,首选抗生素:青霉素。264.新生儿破伤风多在出生后5-7天发病。

265.新生儿惊厥最常见原因:新生儿缺氧缺血性脑病。(中度:出现惊厥,肌阵挛)266.急性呼吸道感染约占门诊:60%以上。267.重症肺炎引起病理生理变化:低氧血症。268.小儿易发生间质性肺炎:肺含血丰富。269.脓胸最常见病原体:金黄色葡萄球菌。270.麻疹出疹时全身症状不减轻。

271.青霉素治疗猩红热疗程:7-10天。

272.小儿肺炎室内温度:18-20%湿度:60%。

273.小儿腹泻重度酸中毒:二氧化碳结合力小于9mmol/l 274.引起疱疹性咽夹炎的病原体:柯萨奇病毒。275.对接触麻疹易感儿,隔离:至少21天。276.

第二篇:全科护士考试试题

原文地址:全科护士选择题及答案作者:熊居深山

1.有关社区定义来源的说法,正确的是C.采用我国社会学家费孝通先生的定义

2.有关构成社区的基本要素描述,正确的是D.人是社会生活的主体,是构成社区的第一要素

3.有关社区地域文化内容的陈述,正确的是B.社区成员的价值观体现于社区文化之中 4.下列有关社区功能的描述,正确的是C.社区通过各种管理体制,体现社区的控制功能 5.社区卫生服务与医院服务的不同点是B.服务对象是以群体为中心

6.有关社区卫生服务的特点,下列正确的说法是E.对居民提供的是第一线、最基本的医疗保健服务

7.社区卫生服务的可及性特点,是指居民可以 B.按需要及时获得专科性治疗服务

8.下列有关社区卫生服务特点的描述,正确的是C.社区卫生保健服务是医院治疗服务的补充和延续

9.社区卫生服务的特点,是强调为社区居民提供D.基础性服务

10.下列有关社区医疗特点的描述,正确的是A.是当前社区卫生服务的重点工作 11.有关社区康复的描述,不正确的是B.是居家开展疾病后期治疗的一种方法

12.有关社区卫生服务中心的描述,正确的是B.政府原则上按街道办事处范围设置社区卫生服务中心

13.有关社区卫生服务站的描述,正确的是 A.是作为对社区卫生服务中心因各种原因无法覆盖区域的补充

14.下列有关全科医学提供服务特点的描述,应除外 C.提供阶段性服务

15.下列有关全科医疗特点的描述,错误的是 A.以社区居民为对象提供专科医疗服务

16.在社区卫生服务概念中,强调的服务特点应除外 C.以生活困难者为重点 17.社区卫生服务工作的内容,不包括E.提供危急重症的治疗服务 四.多项选择题

1.下列属于社区卫生服务工作范围的内容是 A.医疗与保健B.健康教育 C.计划生育 D.医疗康复 2.有关社区医疗的陈述,正确的有

B.为居民提供出诊、巡诊、转诊及家庭病房服务 D.社区服务人员要为居民建立健康档案

E.社区治疗中特别强调使用适宜技术、中医、中药等 3.有关社区康复的描述,正确的是 A.社区康复不同于医疗康复

B.社区康复强调采用医学和社会人文科学等综合措施 D.强调通过康复训练恢复残疾人的正常生活能力 E.由康复人员进行生活自理、语言、心理训练等 4.目前我国社区卫生服务发展中存在的主要问题有 A.仍然存在重病轻防的倾向 C.缺乏高素质的社区卫生服务人员

D.常见病、多发病的患者仍然选择去大的综合性医院 E.普遍对社区卫生服务模式及特点认识不足,重视不够

1.甲乙两地年龄标准化死亡率相等,而甲地粗死亡率较低,最可能的原因是E.乙地老年人比重高于甲地 某病的患病率是指 D.某病新旧病例数/同期平均人口数 对某社区10万人进行某病普查,共查出病例30人,因此得出 B.某社区某病患病率为30/l0万 某工厂体检,检查了500人的乙肝表面抗原,结果发现50人阳性,最合适的描述指标为D.感染率

5.罹患率可以表示为

C.(一年内的新病例数/同年暴露人口数)×k 6.某种新疗法可延长患病后寿命,但不能治愈该疾病,可能会出现 C.该病患病率将增加 7下列因素与患病率的变化无关的是C. 人口总数自然增加或减少 8在评价暴发性肝炎临床抢救效果时,最恰当的指标应是B. 发病率

9某区有人口10万人,2000年因各种疾病死亡800人。该年共新发生结核病例300人,原有结核病人400人,当年共有50人死于结核,则结核的病死率为C.50/700 10下列措施属于第一级预防的是B开展社区康复 11下列措施属于第二级预防的是B高危人群重点监测

12、下列有关发病率的描述中不正确的是C分母为人群总数。1.医学伦理学的基本原则不包括B.价值原则

2.我国卫生部制定《医务人员医德规范及其实施办法》的时间是A.1988年 3.《中共中央、国务院关于卫生改革与发展的决定》正式发布的时间是C.1997年 4.以下对社区卫生服务的公正原则的理解,错误的是 D.公正原则强调事实与结果的完全平等

5.医师向病人说明病情、治疗过程及预后状况履行的义务是B.帮助病人知情 6.“人人享有初级卫生保健”的战略目标体现D.公正原则

7.当妊娠危及胎儿母亲的生命时,可允许行人工流产或引产,这符合C.不伤害原则 8.医生在治疗中确诊一名肝癌患者,他妥当的做法应是B.向患者本人宣布病情危重程度 9.某大医院眼科医生第二天要为一位病人做角膜移植手术,当天晚上发现准备的角膜不见了,若病人第二天做不了手术,将有完全失明的危险,于是该医生到医院太平间偷偷摘取了一位刚刚死亡的病人的角膜。第二天,手术很成功。但不久,死亡病人的家属发现角膜不见了,状告了该医生。关于这起案件,下列说法的正确的是A.该医生侵犯了患者和家属的知情同意权

10.不会造成对病人的伤害的情况是D.病人不可逆昏迷用呼吸机维持时允许病人死亡 11.下列描述中,体现有利原则的是

A.对有危险或伤害的诊治措施,通过评价选择利益大于危险或利益大于伤害的行动 12.一个医护行动的双重效应在有害效应上其主观性质是E.间接、不可预见的 13.《城市社区卫生服务基本工作内容(试行)》规定城市社区卫生服务的基本工作内容共有D.13项

14.下列关于我国社区卫生服务的立法概况的描述错误的是C.社区卫生服务工作无法可依 15.《国务院关于发展城市社区卫生服务的指导意见》提出的我国社区卫生服务完成“全国地级以上城市和有条件的县级市建立比较完善的城市社区卫生服务体系”的工作目标的时间是C.2010年

四、多项选择题

1.医学伦理学的基本原则

A不伤害原则B有利原则C D.尊重原则E.公正原则

2.下列关于国务院《关于发展城市社区卫生服务的指导意见》提出的发展社区卫生服务的基本原则的描述正确的是

A.坚持社区卫生服务的公益性质,注重卫生服务的公平、效率和可及性。

C.坚持实行区域卫生规划,立足于调整现有卫生资源、辅以改扩建和新建,健全社区卫生服务网络

D.坚持基本医疗为主,中西医并重,防治结合 E.坚持以地方为主,因地制宜,探索创新,积极推进 3.知情同意的内容包括 A.信息的告知 B.信息的理解 D.病人同意的能力 E.自由同意的程度

4.下列关于病人自主性的前提的描述正确的是 A.医护人员提供足够的、正确的和可理解的信息 B.病人必须有一定的自主能力 C.病人的情绪处于稳定状态

D.病人的自主性决定不会与他人和社会的利益发生严重冲突 5.下列属于侵犯病人隐私权的行为有 B.病人的个人信息与家人分享 C.在公共场合随意散播病人的信息 D.在闲谈中无意泄漏病人的个人隐私 E.在闲谈中故意泄漏病人的个人隐私 第二章

1.下列不属于公共卫生项目的是 E女常见病治疗 2.能评价护理的成效标准是C所设定的成效指标能达到 3.以下那一项不属于小区护理成效指标::小区护士的收入及满意度 4.以下那一项不属于社区护士的职责:管理小区中心的经济及物价

5.国家提出到哪一年要全国地级以上城市和有条件的县级市建立比较完善的城市社区卫生服务体系”的工作目标:2010

四、多项选择

1.卫生信息管理包括 日常疾病预防控制信息收集 健康档案管理 社区卫生诊断

2.评估健康照顾需求的结构标准包括 社区护士拥有相关知识和技巧 有相关评估架构和工具 有相应仪器去执行特定的评估 3.小区护理的特点包括: A.广泛性 C.协调性 E.可及性

4.小区护士所执行的护理过程标准有 健康教育、指导和劝导 治疗和处置能力 个案管理 监测患者健康状况 5.个案管理是指: 向所负责的患者提供身心社灵的整全护理 充分且合理地运用人、财、物和社区资源 协助患者寻求相应的帮助和支持

6.社区护士在监测患者健康状况过程中, 包括

A.正确地应用体查方法及辅助仪器测量健康变化 B.持续监测患者家居康复训练计划的执行情况 C.持续监测患者疾病自我管理的实施情况 D.监测急性发作的症状和体征

7.在推行国家社区护理工程时,那几项工作需要加速发展: 规范化服务的管理 医院与社区双向转诊制度 社区护士的教育

1、小明,男,7岁,同他一起居住的家庭成员有爸爸、妈妈、奶奶和未婚的小姑。小明所在的家庭类型是: B.主干家庭

2、属于家庭健康功能的是: C.寻求卫生服务

3、某家庭,主要家庭成员有夫妇二人,一女,14岁。按照Duvall的家庭周期划分,该家庭所处的发展阶段为:(C.有青少年家庭

4、中年期家庭的主要发展任务是:B.巩固婚姻关系,计划退休后生活

5、挡风玻璃似的调查是指:D.护理人员通过自己观察大体上了解社区的特点

6、在社区护理诊断中,按照北美护理诊断协会护理诊断系统,属于健康性护理诊断的是:(A.有增强社区应对的趋势

7、在下列社区护理目标中,属于书写正确的是:E.至2007年12月31日方庄社区90%居民能够得到安全的饮用水

8、属于结果评价的是:A.所制定的具体目标是完全达到、部分达到还是未达到?

9、按照Duvall的家庭周期划分,中年期家庭是指:()D.从所有孩子离家到退休

10、属于家庭凝聚力的评估内容是:()C.家庭管理是否充分满足身体、精神的需要

11、属于家庭内资源的是:B.疾病照顾能力

12、关于家庭亲密度图正确的陈述是:()E.用于理解家庭成员之间的亲密关系及相互关系

13、关于评估性家访,下列叙述正确的是:()A.目的是对照顾对象的家庭进行评估

14、在默克的社区问题优先顺序决定准则中,除外:()B.对其他社区的借鉴

15、关于社区护理措施的制定,下列陈述错误的是:()D.主要针对那些没有具体健康问题的服务对象

16、不属于社区医疗保健机构的是:D.老年公寓

17、属于社区卫生服务状况的是:()

D.本社区卫生服务人员的数量、构成和结构等状况

18、核心家庭的特点是:A.家庭内部有一个权力和活动中心

19、某夫妇,婚后无子女,一年前收养一男孩,10岁,现在他们组成的家庭类型属于:A.核心家庭

20、某家庭,夫妇两人,有两个孩子,女儿12岁,儿子8岁,他们目前家庭的主要发展任务是:()

D.教育孩子,使孩子社会化

E.注意青少年的教养与沟通,青少年与异性交往

21、在社区评估方法中,系统调查是指:B.护理人员基于评估工具对评估对象进行整体全面的调查

22、在社区护理诊断中,按照北美护理诊断协会护理诊断系统,属于现存性护理诊断的是: B.社区应对无效

23、不属于OMAHA系统护理诊断领域的是:()E.危险性行为

24、下列关于联合家庭,叙述正确的是:()A.结构相对松散,家庭难以做出一致决定

25、不受家庭影响的因素是:C.个人的经济收入

26、不属于家庭生活史评估内容的是:D.家庭保健功能

27、关于家系图正确的陈述是:(A.包括整个家庭的构成及结构 1.书面语沟通的优点是A.信息可以长期保存 2.属于非语言沟通特点的是 B.情景性

3.社区护士在沟通中一般多使用A.公务凝视区域 4.非语言通中最有力最亲密的沟通行为是 B.保持微笑表 5.建立人际关系的基础是C.人际关系的建立是否满足双方的需要 6.人际交往中最受欢迎的个性品质是E.真诚 7.依据功能分类,沟通包括工具沟通与A.情感沟通

8.使用历史最久、范围最广、频率最高的语言沟通形式是D.口语沟通 9.社交距离大约为 C.1.2m~3.5m

四、多选题

1.依据沟通的渠道分类,可以将沟通分成C.正式沟通 E.非正式沟通

2.在人际交往的相似性因素中影响更重要的是 B.学历相似 C.态度相似 D.价值观相似

3.在促进态度转变的因素中与态度的改变呈现倒U形关系的是A.信息的差距 B.信息的可靠程度 C.信息的陈述方式D.信息唤起的恐惧感 E.信息发出者的专业程度

1.个人的健康状态在影响健康的因素中属于 B.生物学因素

2.属于C型行为模式的是E.情绪过分压抑 3.下列属于基本健康行为的是A.平衡膳食 4.与冠心病密切相关的行为模式是A.A型行为模式

5.为了解社区居民对高血压知识的掌握程度,某社区护士抽样调查该社区居民100位。回答正确率达到合格标准者60人,其中包括正确回答所有问题者20人。则该社区高血压知识合格率为D.60%

6.为增加糖尿病患病高危人群对糖尿病的了解,某社区护士计划开展讲座1次。该护士共管理具有高危因素的居民50人,参加讲座者45人,则该次活动的暴露率为 E.90%

7.社区护士成功进行健康咨询的基础是A.建立相互信任的关系 8.健康教育中最常用的教育方法是C.讲授 9.培训操作技能的最重要的方法是E.示教与反示教 10.印刷类宣传品中最常用的是A.标语

11.第一届国际健康促进大会发表《渥太华宪章》是在B.1968年

12.能减少40%~70%的早死,1/3的急性残废,2/3的慢性残废的方法是B.控制不良生活方式

13.直接影响社区居民接受健康教育的动机与愿望的因素是A.家庭经济条件 14.依据制定健康教育具体目标的要求,以下目标中合格的是E.通过1年的健康教育,使社区内体重指数超过28的老年人中有30%体重指数下降到24以内 15.制定健康教育具体目标的基本要求不包括 E.明确方向 16.健康教育具体目标中可以不明确回答的问题是均为正确答案 A.对谁 B.实现什么变化 C.变化程度多大 D.如何测量这种变化

E.在多长时间之内实现这种变化

17.效应评价指标不包括 C.干预活动暴露率

18.健康咨询基本步骤中实际上发生在咨询的每一个步骤当中的是C.接受反馈 19.使用讨论法进行健康教育时应当保证讨论时间不少于D.5~10分钟

四、多选题

1.不良生活方式对健康影响的特点是 A.变异性大 B.广泛存在 C.潜伏期长 D.特异性差 E.协同作用强 2.下列属于基本健康行为的是 A.平衡膳食 C.合理运动

3.依据危险因素的可干预性确认健康问题的依据是 A.优先选择明确的致病因素进行健康教育 B.优先选择可测量可定量评价的项目进行健康教育

D.优先选择社区居民能够接受、操作简便的项目进行健康教育 E.优先选择可以预防控制、有明确健康效益的项目进行健康教育 4.下列指标属于健康教育效应评价指标的是

A.卫生知识合格率 B.卫生知识知晓率 E.行为改变率 5.提问与讨论适于培训

A.知识 B.态度 C.交流技能 D.决策技能

第四章

1.小儿与照顾者建立信任的关键期是A.婴儿期 2.小儿体重生后前半年每月平均增加D.600~800g 3.在学龄期末尚未发展达到成人水平的器官系统是

B.生殖系统

4.小儿生后体重翻倍是在生后A.3~5个月

5.出生体重为3.5kg的小儿,依据公式估算在4个月时体重应当接近C.6kg 6.出生体重为3.5kg的小儿,依据公式估算在10个月时体重应当为E.8.5kg 7.10岁小儿,依据公式估算体重应当为D.28kg 8.小儿一周岁时身长为C.75cm 9.小儿身长在2岁以后平均每年增长B.5~7.5 cm 10.依据公式推算,5岁小儿的身高为A.108cm 11.小儿身长上部量等于下部量是在D.12岁 12.小儿出生时平均头围是A.34 cm 13.6岁小儿使用上臂围评价为营养状态中等,其上臂围可能为D.15cm 14.后囟闭合时间最迟为生后E.6~8周 15.乳牙的萌出时间平均是在生后B.6个月 16.小儿第一颗恒牙萌出的时间是A.6岁 17.女性进入青春前期的年龄是D.11~13岁 18.皮博迪图画词汇试验适用的年龄范围是 B.2.5~18岁

19.麻疹疫苗初种的时间是生后D.8个月

20.出生体重为3g的健康小儿,使用8%糖牛奶人工喂养,8个月时每日奶量应当为B.800ml l 21.全脂牛奶按重量冲调成乳汁时奶粉与水的比例是 E.1∶8 22.3个月小二可以添加的辅食是A.菜汤 23.中度营养不良体重下降超过正常平均值的 D.25%~40% 24.早产儿生理性黄疸消退时间一般持续到生后 D.3~4周

25.母乳性黄疸达到高峰的时间是在生后C.2~3周 26.3个月小儿即将接种的疫苗是B.百白破第一剂

27.未完成百白破疫苗免疫程序,需要使用百白破疫苗补种的儿童是A.6岁小儿 ? 1.小儿生长发育的第一个高峰是在B.婴儿期

29.新生儿期小儿发生疾病和死亡最危险的时间是在生后A.7天以内 30.2岁后到11、12岁前每年体重增加约 B.2kg 31.小儿体重达到平均出生体重四倍的时间是在D.2岁 32.评价小儿脑部发育较有价值的测量头围的时期是E.2岁以内 33.小儿上臂围1~7岁约增加A.1~2cm 34.颅骨缝闭合的时间大约为生后B.3~4个月 35.小儿前囟饱满多提示E.颅内压增高 36.1岁小儿乳牙萌出的数量大约为C.6~8颗 37.新生儿生后心率平均为D.120~140次/min 38.新生儿生理性体重下降恢复的时间一般为生后

D.7~10天

39.新生儿生理性黄疸一般出现于生后C.2~5天

四、多选题

1.幼儿期能够达到的发育水平是

A.前囟闭合 B.乳牙出齐 C.学会控制大小便 E.饮食由乳汁逐渐过渡到普通饮食 2.体重测量的正确方法是

A.在晨起空腹时测量 B.测量前排空膀胱 C.测量前脱去衣裤鞋袜 3.立位身高测量的正确方法是

B.两足后跟、臀部及两肩三点都接触主柱 D.小儿脱鞋、帽,直立,两眼平视前方 E.精确读数到1 cm 4.符合男性生殖系统的发育特点的是 A.12~13岁开始出现阴毛 B.14~16岁发生首次遗精 E.部分男孩乳房发育 5.符合小儿脊髓发育特点的是 B.4岁时下端上移至第一腰椎 C.胎儿时脊髓下端达第二腰椎下缘 D.神经纤维到3岁时才完成髓鞘化

6.受母体雌激素水平影响而形成的新生儿特殊生理变化是 C.乳腺肿大 D.假月经 7.5个月小儿已经接种过的疫苗有A.卡介苗 C.脊髓灰质炎糖丸 E.白百破疫苗第二针 8.母乳喂养的优点是A.成分合理 B.安全卫生 C.喂哺简便D.增强婴儿免疫力 E.利于母亲产后恢复

9.正确的乳房含接姿势是

A.婴儿嘴部大张、下唇外翻、面颊鼓起 B.婴儿嘴上方露出的乳晕比下方多 D.母亲乳房呈圆形

1、正常的胎心率为:D.120~160次/分

2、孕妇出口横径正常值为:C.8.5~9.5cm

3、正常胎动为每小时: C.3~5次

4、适于放置宫内节育器的妇女是:

B.年龄40岁,无子女,要求放置宫内节育器

5、对于接受宫内节育器放置术的妇女,社区护士应告知: E.术后3个月内的经期注意有无节育器脱出

6、适合于采用药物避孕的妇女是: A.月经不规律

7、关于短效口服避孕药的用法,下列叙述正确的是: E.连续闭经3个月以上,应停药检查原因

8、关于女性绝育术的手术时间,正确的是: D.人工流产术后

9、对于行输卵管结扎术的妇女,正确的护理措施是: C.若有发热、腹痛应及时就医

10、药物流产的适应证是: A.妊娠50天以内的健康妇女

11、妊娠期妇女血容量达高峰的时期是: C.妊娠30~32周

12、关于产妇产褥期的生理变化,下列叙述错误的是: A.体内呈高雌激素、高泌乳素水平

13、不属于围绝经期妇女生理特点的是:E.乳房的结构和功能出现退行性改变

14、通过四步触诊,社区护士不能了解到的内容是: D.胎心率

15、孕妇血压属于正常范围的是:B.130/90mmHg

16、在四步触诊中第三步主要目的是:()C.判断胎儿先露部是胎头还是胎臀

17、妊娠晚期孕妇体重增长过多者疑有水肿或隐性水肿,一般每周体重增长不应超过:D.500g

18、产前检查开始的时间是:B.确诊为早孕时

19、关于妊娠期间性生活的指导,下列陈述错误的是:E.在孕期应禁止性生活 20、孕妇检测胎动次数时属于应及时到医院就诊的情况是12小时胎动数为E.16次

21、属于临产先兆的症状是:A.见红

22、当孕妇发生破水时,下列措施中不正确的是: C.嘱孕妇保持蹲踞姿势

23、产妇在产后1日属于正常范畴的生命体征是;

B.体温37.80C,脉搏68次/分,呼吸16次/分,血压130/85mmHg

24、关于产后产妇子宫复旧的评估,叙述错误的是:()D.正常产后的子宫质地软,位于腹部的中央

25、关于产后浆液恶露的叙述,正确的是: B.色淡红,含少量血液

26、若产后产妇会阴伤口感染或愈合不佳,给予高锰酸钾坐浴的时间是: D.产后7~10天起

27、关于产妇乳房的清洁,下列叙述中错误的是:()E.乳头处若有痂垢,可用肥皂擦洗乳头

28、关于母乳的优点,下列陈述中错误的是:()A.母乳中的脂肪含量高于牛乳

29、下列症状不属于围绝经期症状的是:C.腹部包块 30、根据我国计划生育政策规定,属于晚婚的是:()C.男25周岁,女23周岁结婚者

31、妊娠中期是指:C.妊娠13周至27周末

32、关于妊娠期生殖系统的变化,叙述错误的是:()A.妊娠12周后增大的子宫超出盆腔,一般略向左旋

33、产褥期是指产妇全身各器官:D.除乳腺外,从胎盘娩出至恢复或接近正常未孕状态所需的时期。

34、关于产褥期产妇的心理调适,下列叙述正确的是: C. 在依赖期产妇很多需要是通过别人来满足

35、下列不属于婚前健康教育的内容是: B.产前检查

36、在产前检查中,通过肛门检查不能检查到的内容是: B.子宫及其附件

37、当孕妇发生眩晕时,错误的护理指导是:()B.在两餐之间,可适当进食

38、能够增加盆底肌韧性,较好适应分娩需要的运动是: A.缩肛运动

39、在下列情况中,适于按摩孕妇乳头的是 C.乳头扁平

40、产后10天,正常子宫底的位置是:C.在耻骨联合上方扪不到宫底

41、当产妇感觉乳房胀痛,错误的护理方法是:()E.停止哺乳

42、产后落实避孕措施的时间是:D.产后42天

43、采用阴道隔膜避孕,留置时间不宜超过: C.24小时

三、多选题

1、当孕妇发生腿抽痉时,社区护士应指导孕妇采取的措施有:()A.赤脚站立在地面上,弯曲躯体,并加以局部按摩 B.仰卧位,伸直下肢,足背屈曲 C.遵医嘱服用钙剂

E.注意局部保暖,行走时避免脚尖先着地

2、属于临产先兆的症状有:()B.不规律的子宫收缩C.见红

3、属于妊娠期异常症状的有:()A.阴道流血B.腹痛D.重度水肿E.蛋白尿

4、某产妇的乳头发生皲裂,社区护士应指导产妇采取的正确措施是:(ABCDE)A.指导产妇取正确、舒适且松弛的喂哺姿势

B.哺喂前湿热敷乳房和乳头3-5分钟,按摩乳房,挤出少量乳汁使乳晕变软 C.哺喂时先在未损伤的一侧乳房哺乳,让乳头和大部分乳晕含吮在婴儿口内 D.哺喂后,挤出少许乳汁涂在乳头和乳晕上,短暂暴露并使乳头干燥 E.建议产妇增加哺喂的次数,缩短每次哺喂的时间

5、关于宫内节育器的放置时间,正确的叙述是:()A.月经干净后3~7日

B.自然流产者月经复潮,月经干净后3~7日 C.足月产后3个月 E.剖宫产后半年

6、关于宫内节育器的描述,正确的是:()A.是一种安全有效、简便、经济的避孕方法 D.哺乳期放置应先排除早孕可能

7、药物避孕的原理是:(ABCDE)

A.抑制排卵B.改变宫颈粘液性状C.改变子宫内膜形态D.改变子宫内膜功能E.改变输卵管蠕动

8、对于采用药物避孕的妇女,社区护士应进行如下指导:(ABCDE)A.避孕药保存在阴凉、干燥处

B.将用药的目的、方法、可能出现的副反应告知妇女 C.对于长期服药者,建议停药6个月后再受孕 D.使用注射针剂后,应观察15分钟 E.停用长效药者,应服用短效避孕药作为过渡

9、使用避孕药物常见的药物副反应有:()B.类早孕反应C.突破性出血D.色素沉着

10、对于接受人工流产术的妇女,正确的护理措施是:(ABCDE)A.保持外阴清洁

B.术后1个月内禁止性生活和盆浴 C.休息2周,避免重体力劳动

D.术后阴道流血量多,应随时联系医生 E.术后1个月后复查

1.我国老年人的年龄界线是B.60岁

2.老年人口型国家是指≥60岁的老年人占总人口的C.10%以上

3.不属于正常老年人外形特征的是 C.手足震颤 4.关于老年人心血管系统的生理改变,描述错误的是

A.心率变快

5.关于老年人呼吸系统的生理改变,描述错误的是

E.支气管反应性降低

6.关于老年人消化系统的生理改变,描述错误的是

E.食物在胃停留时间缩短

7.关于老年人内分泌系统的生理改变,描述错误的是

B.甲状腺生成增多

8.关于老年人泌尿系统的生理改变,描述错误的是

B.肾小球滤过率增加

9.在日常生活能力量表(ADL)中,属于工具性日常生活能力的项目是 E.服药 10.下列属于老年人特有疾病的是 E.老年性痴呆 11.我国百岁以上老人的首位死因是 B.肺部感染 12.关于老年人的患病特点,描述错误的是 A.病情变化较慢

13.一般不随年龄而下降的认知功能是D.晶态智力

14.对文化程度为小学的老人进行简易智能状态(MMSE)测查时,将其判断为认知障碍的界值分是 C.20分

15.老年痴呆患者早期主要的认知功能障碍是 A.近期记忆力下降

16.某老年人否认自己的衰老,仍努力返聘工作,为自己制定严格的目标,并乐于其中,该老人的人格类型属于 A.防御型

17.下列容易出现离退休综合征的老人是

E.从事管理工作的老人

18.对离退休期老人进行健康指导时,不妥的是 D.避免接触新的事物

19.某老人丧偶3个月,常对着照片中的老伴生闷气,很容易无缘无故地和别人争吵,该老人的心理特点是

D.情绪波动

20.冬季使用暖气时,老年人居室的温度宜调节在C.22℃~24℃

21.有关老年人的营养需求,描述错误的是 C.应多摄入砂糖、红糖等双糖

22.关于老年人的平衡膳食,描述错误的是 D.增加热量摄入

23.关于老年人的饮食原则,描述错误的是 C.食物温度宜高

24.老年人不宜食用的是A.猪油

25.不适于老年人的活动项目是E.打篮球 26.对70岁的老年人来说,活动后最宜心率应为 D.100次/分

27.对老年人来说,根据活动时自我感觉,表明活动强度适中的情况是 C.活动时微出汗或全身有热感

28.关于老年人的活动原则,描述错误的是 D.每次活动不超过20min 29.对老年人来说,可继续进行活动的情况是出现 C.全身热感

30.对老年人来说,要达到活动强度适中,心率恢复到活动前水平的时间应为活动后 B.3~5min 31.对老年人来说,下列存在安全隐患的环境特征是

B.茶几下面铺小块地毯

32.下列易致老年人烧伤的情况是E.持续使用红外线治疗仪1小时 33.关于老年人的药代动力学特点,描述正确的是

C.脂溶性药物血浆半衰期缩短

34.关于药物不良反应,描述正确的是 E.发生在正常用法用量下 35.老年人使用镇静催眠药时,描述正确的是

C.血浆半衰期延长

36.老年人使用降压药时,描述正确的是 D.β受体阻滞剂易诱发和加重心衰

37.常见耳毒性不良反应的抗生素是D.氨基糖苷类 38.老年人使用解热镇痛药时,描述错误的是

A.血药浓度降低

39.老年人睡前不宜进行的活动是E.争论家事 40.对便秘老人进行健康指导时,措施不当的是 E.经常使用开塞露

41.对尿失禁老人进行健康指导时,措施不当的是 A.限制饮水量

42.关于老年人的睡眠特点,描述错误的是 A.睡眠时间增加

(二)多项选择题

43.正常老人记忆力的变化特点是 A.随年龄增长而下降 B.衰退程度有个体差异 D.记忆广度和回忆减退 E.给予提示仍很难回忆起

44.对丧偶老人进行健康指导时,正确的是 A.寻求新的生活方式 B.建立新的依恋关系 C.领悟“人死不能复生”的道理 45.老年人经济虐待的形式包括 B.不承担老人的经济赡养责任 C.未经允许动用老年人的财产 D.拖欠发放离退休老人的养老金 46.我国老年人再婚面临的障碍包括 A.子女的反对 B.居住条件的限制 C.社会舆论的压力 D.老年人自身的旧观念 47.建议老年人摄入的食物有

C.牛奶、鸡蛋、鱼类

D.花生、玉米、小米

E.柑橘、菠萝、菠菜

48.老年人服用抗抑郁药时,描述正确的是 C.治疗作用与血药浓度密切相关 D.毒性反应与血药浓度密切相关 E.可出现口干、便秘、肌肉震颤等 49.有利于促进老年人睡眠的措施是 A.温水泡脚B.背部按摩C.喝热牛奶 50.可促进老年人记忆力的措施是 A.适当运动B.保证规律睡眠

51.护理长期卧床老人时,正确的措施是 B.每天按摩和活动肢体2~3次 D.若骶尾部发红,应避免继续受压 E.在腋窝、腘窝、腹股沟等处涂爽身粉 52.老年人长期便秘可导致

A.腹胀B.肛裂C.痔疮E.排便不尽感 第五章a 1.关于慢性非传染性疾病的定义,错误的是 A.不是一组疾病的概括性总称、而是特指某种疾病

2.WHO指出的慢性非传染性疾病最常见的3种共同危险因素之一是 D 静坐生活方式 3.对慢性非传染性疾病危险因素的干预比治疗任何慢性病的成本-效益都好的是 A 戒烟

4、下列属于合理膳食的是 B 低盐食物

5.体重质量指数(BMI)超重标准的上限是<28,下限是 A≥24 6.慢性非传染性疾病的管理原则是

A 强调降低最常见慢性非传染性疾病的共同危险因素

7.强调在社区及家庭水平上降低最常见慢性非传染性疾病的共同危险因素,所进行的预防应该是 E 生命全程

8.慢性非传染性疾病的管理策略是 A 全人群策略和危重病人策略并重 9.慢性非传染性疾病管理原则是 C加强社区慢性非传染性疾病防治的行动

10.世界卫生组织建议每人每日食盐量不应超过 A 6克 11.社区护士在疾病护理方面的主要工作是A慢性病护理 12.慢性非传染性疾病康复和康复护理强调的目标是 B使病、伤、残者重返社会 13.关于康复护理学,错误的是 C康复护理单独即可完成患者康复任务

14.健康教育目标可以分解成以下几个部分,但不应该包括在内的是E定语—什么样的主语 1.慢性非传染性疾病的定义,错误的是 E有明确的传染性生物病因证据

2.慢性非传染性疾病流行病学的特点,错误的是 D强度减弱

3.卫生部公布的调查结果显示,2006年中国人首位的死因是 C恶性肿瘤

4.卫生部公布的调查结果显示,2006年中国人前4位的死因不包括E营养和代谢疾病 5.WHO指出的降低慢性非传染性疾病最常见的共同危险因素是 A吸烟 6.合理的膳食是 E低胆固醇食物

7.肥胖的危害不包括C低甘油三酯血症的检出率增高 8.关于酒的成分、种类和作用,错误的是 D 仅红葡萄酒对心血管系统有保护作用

9.关于社会心理因素对慢性非传染性疾病的危害,错误的是 C 急性应激状态较慢性应激更易引起高血压

10.社会心理应激不能诱发或促进的疾病是 B阑尾炎 12.慢性非传染性疾病的中间危险因素不包括 A吸烟 13.慢性非传染性疾病的管理原则,错误的是 A二级预防并重

14.降低最常见慢性非传染性疾病的共同危险因素不应强调 D 在个体水平上 15.慢性非传染性疾病三级预防并重,错误的是 C仅对特定的某种慢性非传染性疾病防治 16.新型的慢性非传染性疾病保健模式不强调 B 加强对个体疾病的治疗

17.慢性非传染性疾病综合性社区行为危险因素干预项目错误的是 A通过改变行为危险因素来治愈慢性非传染性疾病

18.慢性非传染性疾病患者康复护理的内容不包括 D完全恢复日常生活能力

20.慢性非传染性疾病患者挛缩畸形的康复护理错误的是E为保持肢体的功能位不必定时改变体位

21.慢性非传染性疾病患者日常生活活动自理训练的康复护理不包括D替患者穿脱衣物 22.社区慢性非传染性疾病患者康复的原则错误的是 E替代康复对象参与

23.社区慢性非传染性疾病患者康复护理的要点错误的是A康复对象仅为伤、残和病者 24.社区健康教育的步骤错误的是E第五步开发领导

26.制定社区健康教育计划的第三个步骤拟定方案中不包括E评价 1.诊断1级高血压的收缩压(mmHg)标准是: C 140-159 2.诊断1级高血压的舒张压(mmHg)标准是: E 90-99 3.诊断2级高血压的收缩压(mmHg)标准是: B 160-179 4.诊断2级高血压的标准(mmHg)是 B 160-179/100-109 5.诊断3级高血压的舒张压(mmHg)标准是: E ≥110 6.诊断3级高血压的标准(mmHg)是 A ≥180/≥110 7.最准确的血压计是 A汞柱式血压计

8.测量血压充气时气囊内压力应达到肱动脉搏动消失后再升高的数值是A 30mmHg 9.我国卫生部已经重新确定测量血压的计数单位是 A mmHg 10.血压的记录方法是 A收缩压/舒张压

11.高血压患者的体育锻炼,错误的是 C可以参加比赛性质的活动

12.高血压患者因用力而使收缩压上升,甚至造成血管破裂的原因可能是A便秘 13.关于高血压的流行病学特点,错误的是:B我国治疗率高于美国 2.中国高血压防治指南指出的三个主要危险因素之一是 A超重 3.高血压损害的靶器官不包括 E胃

5.高血压患者常见的心理社会情况不包括E同时患糖尿病

6.《社区高血压、糖尿病防治管理手册》指出影响高血压预后的因素不包括 E胶质细胞瘤 7.《社区高血压、糖尿病防治管理手册》指出高血压的心血管危险度分度不包括 A 正常层 8.常用的降压药物不包括 D 肾上腺皮质激素 9.血压计的种类不包括 E携带式血压计

10.《社区高血压、糖尿病防治管理手册》指出社区正常人群高血压健康教育的内容不包括 E降压药的疗效和副作用

11.《社区高血压、糖尿病防治管理手册》指出社区的不同目标人群高血压健康教育的共同内容不包括

E高血压长期药物治疗的重要性

12.对高血压高危人群健康教育的内容不包括 E高血压危险分层、分级管理及意义

13.《社区高血压、糖尿病防治管理手册》指出对已确诊的高血压患者健康教育的内容不包括

A高血压危重症的抢救

14.社区人群高血压健康教育的渠道不包括 E 发放健康教育材料

15.高血压高危人群管理原则不包括 A加强卫生消毒 16.以下不属于预防高血压的生活方式是D久坐休息 17.高血压患者的非药物治疗错误的是 A经常发怒

18.以下不属于对超重的高血压患者进行的体重控制措施的是 C吃人造奶油 19.含钙少的食物是 D动物内脏

20.不利于高血压患者平衡心理的是 B 尽可能压抑愤怒 21.高血压管理流程不包括 E特发性高血压的诊断 1.2004年卫生部公布我国成人糖尿病为2.6% 2.诱发糖尿病的遗传因素中错误的是 有“肥胖基因” 3.糖尿病可改变危险因素是 B病毒感染

4.糖尿病的典型症状是多尿、多饮、多食和体重减轻 5.果糖胺反映血糖控制总体水平的时间是 B 2~3周

6.反映近2~3个月内血糖控制总体水平的指标是 C 糖化血红蛋白 7.反映近2~3周内血糖控制水平的指标是 A果糖胺

8.WHO推荐的成人口服葡萄糖耐量试验(OGTT)的口服无水葡萄糖剂量是 B 75克 10.关于糖尿病的治疗目标,错误的是E彻底治愈

11.某糖尿病患者的体型理想,从事轻体力活动,每日每标准体重所需的热能(kcal/kg标准体重)应是C 30 12.《社区高血压糖尿病综合防治管理手册》指出的糖尿病标准体重(kg)计算公式为C身高(cm)-105 13.《社区高血压糖尿病综合防治管理手册指出的糖尿病理想体重与标准体重的关系是B理想体重=标准体重±10%

1.下列属于糖尿病可改变的危险因素的是 B 不良生活方式 2.不属于糖尿病危险因素的是B 性别

3.以下不属于糖尿病常见症状的是D 咳嗽、喘息 4.糖尿病患者的健康问题一般不包括 E呕血黑便 5.不属于糖尿病慢性并发症的是 E糖尿病低血糖 6.糖尿病的微血管病变不包括 ? A多发性周围神经病变 B 出血性脑血管病 C 糖尿病性眼病 D糖尿病肾病 E败血症

7.糖尿病常见的辅助检查不包括 E肺功能测定

8.具有糖尿病症状的患者如欲诊断糖尿病还需要满足的实验室检查标准是 B任意时间血浆葡萄糖水平≥11.1mmol/L 9.关于糖尿病的诊断标准,正确的是 ? A口服葡萄糖耐量试验2小时葡萄糖水平≥11.1mmol/L B口服葡萄糖耐量试验2小时葡萄糖水平≥13mmol/L C任意时间血浆葡萄糖水平≥6.7mmol/L D任意时间血浆葡萄糖水平≥7.0mmol/L E任意时间血浆葡萄糖水平≥11.1mmol/L

12.糖尿病患者必须做到,但不能完全由主观控制的是 E不发生感染 13.不属于糖尿病低血糖反应的处理原则的是

B如果无血糖检测条件时,所有怀疑为低血糖反应的患者不能按低血糖处理 14.不属于糖尿病低血糖反应的诱发因素的是 E过量饮水

15.不属于胰岛素治疗反应时应观察的项目是 A高血糖反应

16.不属于腹部注射胰岛素的优点的是 E减少低血糖的发生 17.糖尿病患者可多吃的食物为 A大米 18.糖尿病患者可适量吃的食物为 C瘦肉 19.糖尿病患者应少吃的食物为B酒精

20.糖尿病患者中适宜运动治疗人群是: E慢性呼吸道感染者

21.糖尿病患者运动的注意事项中错误的是 B 运动中血压如果升高过多,不用担心 22.不属于糖尿病患者运动时避免低血糖的措施是

D运动停止数小时后不会发生迟发性低血糖

23.不属于确定与妊娠分娩有关的糖尿病高危人群的标准的是 A年龄≥25岁的妊娠妇女 24.糖尿病高危人群确定标准中不可改变因素是 C 有糖尿病家族史者

25.糖尿病高危人群群体干预内容不包括 B早期用药预防糖尿病 27.糖尿病管理的评价指标不包括 C糖尿病管理流失率 1.在所有因心脏病死亡中占最大比例的是 A冠心病 2.血脂正常是指B 高密度脂蛋白胆固醇高 3.冠心病可改变的危险因素是 C 吸烟 4.冠心病不可改变的危险因素是A性别

5.以剧烈而持久的胸痛为主要表现的,属于冠心病分类中的 C心肌梗死型 6.以发作性胸骨后疼痛主要表现的,属于冠心病分类中 B心绞痛 7.稳定型心绞痛胸痛的性质,错误的是 E刀扎样痛

8.原为稳定型心绞痛,在1个月内疼痛频率增加、程度加重、时间延长、诱发因素变化、硝酸类药物缓解作用减弱。应考虑: B.不稳定型心绞痛 9.心绞痛发作时含服硝酸甘油片后起效的时间是 A 1~2 min 10.关于冠心病的流行病学特点,错误的是 C女性多于男性 3.不属于冠心病次要的危险因素的是 B C型性格者 4.冠心病危险因素中遗传因素不包括 E C型性格者 5.关于稳定型心绞痛胸痛的时间,错误的 C持续数天 6.心绞痛的诱发因素不包括D肥胖

7.心绞痛和心肌梗死的胸痛特点,相同的是 B部位 8.心肌梗死可出现的症状中错误的是 A短暂胸痛 9.不属于心肌梗死发作特点的是 D常发生于午后 10.心绞痛发作时的院前急救措施错误的是 C服阿斯匹林 11.对不稳定型心绞痛者的处理是: B舌下含服硝酸甘油 12.心绞痛发作时舌下含服硝酸甘油片不应当A首次剂量加倍

13.护士对心肌梗死患者及家属的院前急救指导错误的是 D频繁早搏时给肾上腺素 14.指导病人及家属保存药物错误的是 A药应保存在浅色瓶中

15.在心肌梗死康复护理指导中,活动时的注意事项错误的是 A在饭后半小时开始运动 16.在心肌梗死康复护理指导中,应提倡的活动是: E散步

17.在心肌梗死康复过程中的关于排便指导内容不包括: E连续数日未排便者可用强泻剂 18.冠心病二级预防错误的是 E.大运动量锻炼

1、以下有关脑卒中的流行病学特点描述正确的是 复发率高

2、以下有关脑卒中的危险因素描述正确的是 心房纤颤不是脑卒中的危险因素

3、脑梗死起病多见于 安静休息时

4、脑卒中患者最常见的功能障碍是 A、运动障碍

5.下列用于脑卒中日常生活活动能力评定的是A.Barthel氏指数

6.ADL评定中Barthel 指数总分为70分,结果是 D.生活大部分自理,轻度依赖 7.脑卒中的常见临床表现不包括 B.一侧下腹痛

8.脑卒中偏瘫患者仰卧位时患侧上肢摆放正确的是 D肘关节伸展置于枕上,掌心向上,手指伸展 9.脑卒中偏瘫患者保持仰卧位时,以下哪个不正确 D.膝关节伸直,足底放软枕

10.脑卒中偏瘫患者健侧卧位时患侧上肢摆放正确的是()A. 肘关节伸展,前臂旋前,腕关节背伸

11.脑卒中偏瘫患者患侧卧位时患侧上肢摆放正确的是 A.肘关节伸展,手指张开,掌心向上 12.坐位平衡训练中脑卒中偏瘫患者受到突然的推、拉外力仍保持平衡时可认为完成的平衡训练是他动态平衡训练

13.以下有关脑卒中偏瘫患者进食训练描述正确的C偏盲时食物放在健侧 14.预防肩关节半脱位的正确措施是 E用吊带支撑患侧上肢

15.对长期卧床的脑卒中偏瘫患者,预防皮肤损伤应做到A.应选择合适的床垫,以硬枕、硬垫为好B.定时变换体位C.翻身时动作要轻柔,不可在床上拖拉患者 D.保持床单干燥、平整E.维持患者足够的营养 16.以下有关拐杖的使用方法正确的是 A. 使用单拐时应置于患侧 B. 使用拐杖时仅用腋窝支撑身体 C. 使用拐杖时手应握住中部的把手 D. 步行时的两拐与患足呈一线 E. 步行时的两拐与双足呈一线

17.以下有关轮椅的使用方法不正确的是床到轮椅的转移时轮椅放在护理对象患侧

一、多项选择

1、脑卒中可干预的危险因素包括 A、高血压 B、心脏病 C、颈动脉狭窄

2、社区中处于康复期的脑卒中患者主要健康问题有

A、运动障碍 C、感觉障碍 D、言语障碍 E、日常生活活动能力障碍

3、可由脑卒中患者或其家属在家中进行的检查项目是 B、血压检查 C、面肌检查 D、感觉检查 E、反射检查 4.下列脑卒中偏瘫患者向健侧被动翻身动作正确的是 先旋转上半部躯干

旋转上半部躯干时,护理人员一手放在颈部下方 旋转下半部躯干时,护理人员一手放在患侧膝关节后方 5.以下有关脑卒中偏瘫患者主动向健侧翻身动作描述正确的是 双手十指交叉握手,患手拇指置于健手拇指之上 健腿插入患腿下方

双上肢伸直举向上方做左右侧方摆动

6、以下有关脑卒中偏瘫患者患侧起坐训练动作描述正确的是 健足推动患足,将小腿移至床缘外 健手掌插在患侧腋部支撑 手掌边推动边后撤

7、以下有关脑卒中偏瘫患者上楼梯训练描述正确的是 健足先上

护理人员站在患侧后方

护理人员一手协助控制膝关节,另一手扶持患侧腰部

8、以下有关脑卒中偏瘫患者下楼梯训练描述正确的是 患足先下 护理人员站在患侧

一手置于患膝上方,稍向外展方向引导

9、以下有关脑卒中偏瘫患者从床到轮椅的转移训练描述正确的是 A.轮椅放在护理对象健侧 B.用健手、健腿站起 C.健手扶在外侧扶手上 D.以健腿为轴转动躯干

10.坐位平衡训练中脑卒中偏瘫患者需经过的三阶段平衡训练是:静态平衡训练;自动态平衡训练 他动态平衡训练

11、以下哪些是脑卒中的常见并发症(A、肩关节半脱位 B、肩痛C、关节挛缩畸形 D、吞咽障碍

12、以下有关脑卒中患者日常生活指导正确的是 限制总热量 适当增加蛋白质

长时间乘坐轮椅者佩带无指手套 推乘轮椅护理对象下坡时倒行

1.慢性阻塞性肺疾病病情严重度的分级为 A高危、轻度、中度、重度、极重度

2.有关慢性阻塞性肺疾病患者活动与休息的描述,错误的是E立位时上半身伸直可使腹肌放松

3.以下有关慢性阻塞性肺疾病患者氧疗的目的描述错误的E使肺功能恢复正常 4.慢性阻塞性肺疾病患者家庭氧疗的原则是:A低流量持续 5.慢性阻塞性肺疾病患者家庭氧疗的流量(L/min)是: A1~2 6.慢性阻塞性肺疾病患者家庭氧疗的时间(小时/天)是: A>15 7.协助慢性阻塞性肺疾病患者训练腹式呼吸时,护士一手放在患者的A腹部

8.慢性阻塞性肺疾病患者通过吹蜡烛来训练缩唇呼气,错误的是 B缩唇的大小程度要适合 9.不利于慢性阻塞性肺疾病患者换气的姿势是 E端坐位挺胸 1.不属于慢性阻塞性肺病的流行病学特点的是 C 女性患病率高于男性 2.慢性阻塞性肺疾病的危险因素不包括 E糜蛋白酶含量增高

5.可导致与吸烟无关的慢性阻塞性肺疾病的职业性粉尘和化学物质不包括 C尼古丁 6.慢性阻塞性肺疾病的病史特征不包括: B司机职业 7.可不入院治疗的是 E高龄慢性阻塞性肺疾病患者稳定期

8.对慢性阻塞性肺疾病患者的健康教育错误的是 D 间断高浓度氧疗 9.不属于慢性阻塞性肺疾病患者增加吸氧浓度的目的的是 E减轻左心负荷 10.不能增加腹式呼吸效果的措施是 E呼气时间短、吸气时间长

11.关于慢性阻塞性肺疾病患者缩唇呼气的方法,错误的是 E吸呼比率为1:1 12.慢性阻塞性肺疾病患者缩唇呼气的训练要点错误的是: E吸呼比率为2:1 13.关于慢性阻塞性肺疾病患者的缩唇呼气的重点,错误的是: B呼吸频率加快 14.慢性阻塞性肺疾病患者在日常生活活动中呼吸动作是正确的 E腹式呼吸 15.慢性阻塞性肺疾病患者错误的体位引流方法是 B安排在饭后半小时进行

16.治疗慢性阻塞性肺疾病的叩击法,错误操作是 C以手臂力量叩击

17.不属于慢性阻塞性肺疾病一级预防的措施是 A筛查发现高危人群

18.慢性阻塞性肺疾病的筛查对象不包括 E连续吸烟3年以上

1. 不属于肿瘤初级预防的内容是:积极治疗癌前病变 2.常用于肝癌普查的指标是 C.甲胎蛋白 3.与子宫颈癌发生无关的因素是E.EB病毒感染 4.下面不是常见癌症危险信号的是E.习惯性失眠 5.肠造口病人护理中不正确的做法是 B.尽量少进食 6.下面不属于癌前病变的是C.乳腺增生 7.便秘通常表现为A.大便干结、排便困难

8.预防造口旁疝的措施不正确的是 A.尽量减少活动,卧床休息 9.对肠造口内陷病人的护理措施不正确的是 A.多做增加腹压的活动

10.预防肠造口周围刺激性皮炎的措施不正确的是

D.经常更换造口袋

11.肿瘤病人便秘的护理要点不正确的是A.常规使用开塞露通便 12.乳房自检不正确的做法是A.在月经期间进行 13.肿瘤病人饮食指导不恰当的是 D.多饮水 14.关于肠造口狭窄,不正确的说法

E.长期携带造口的病人必然会发生 15.关于造口用品的选择,不正确的说法

D.回肠造口建议使用闭口袋

三、多项选择题

1.癌因性疲乏由疾病本身及其相关治疗引起,病人常有的主观感觉 A.虚弱 B.活动无耐力 C.不能集中注意力 D.兴趣减低

2.预防化疗后感染的护理措施有

A.注意保暖,预防感冒

B.开窗通风,保持空气清新

C.注意饮食卫生,预防腹泻 E.保持皮肤清洁

3.应对癌因性疲乏的护理措施有

A.适当锻炼

B.加强营养

D.保持良好的情绪状态

E.保证规律的休息和睡眠 4.肠造口病人常见的皮肤问题有

A.刺激性皮炎

B.机械性损伤

C.过敏性皮炎

D.毛囊炎

5.促进肿瘤发生的不良生活方式有

B.饮酒

C.吸烟

E.食用霉变食物 第六章

1.细胞免疫属于体液免疫

2.在感染过程中出现早,消失快,可作为近期感染标志的免疫球蛋白是E.IgM 3.在感染过程中出现晚,持续时间长,是既往感染标志的免疫球蛋白是E.IgM 4.某地区或某单位,短时间内突然出现大量病例的现象称为C.暴发流行

5.体温升高达39 oC以上,而且24小时体温相差不超过1oC,这种热型为A.稽留热 6.24小时体温相差超过1oC,但最低未到正常水平的热型 B.弛张热

7.皮疹的形态、出现时间、部位、先后次序等,对传染病的诊断和鉴别诊断有重要意义。皮疹出现在发病第2日的传染病是E.水痘

8.传染病与其他感染性疾病的主要区别是C.有传染性 9.以下那项不是乙型肝炎的传播途径E.呼吸道传播 10.痢疾的主要传播途径是C.粪--口传播 11.属于甲类传染病的是 A.霍乱 12.属于丙类传染病的是D.流感 13.甲肝的主要传播途径是E.粪--口途径 14.下列不是丙型肝炎传播途径的是 E.呼吸道

15.由人类乳头瘤病毒感染引起的良性增生性疾病是A.尖锐湿疣 16出现硬下疳表现的疾病是D.梅毒

17.对自然污染菌达到消毒合格的杀灭率是E.99.9% 18.卫生部《消毒技术规范》规定,经灭菌处理后,被灭菌处理对象上微生物存活的概率必须减少到C.10-6 19.紫外线消毒是属于B.物理消毒

20.连续使用棉纱口罩的时间不得超过 B.4小时 21.耐热耐湿物品首选的灭菌方法是B.压力蒸汽灭菌 22.戊二醛是属于B.高水平消毒剂 23.过氧乙酸是属于B.高水平消毒剂

24.不耐湿热的精密仪器,灭菌首选D.干热灭菌

25.进入人体自然通道与管腔黏膜接触的内镜,如胃镜、肠镜、喉镜、支气管镜等,用前应达到C.中水平消毒

26.携带病原微生物具有引发感染性疾病传播危险的医疗废物是 A.感染性废物 27.能够刺伤或者割伤人体的废弃的医用锐器是()C.损伤性废物

28.肺鼠疫在大、中城市发生并有扩散趋势的情况属于 A.特别重大(Ⅰ级)突发公共卫生事件 29.下列属于中度危险性物品的是C.压舌板

30.治疗室物体表面的细菌菌落总数应控制在B.≤10 cfu/cm2

四、多项选择题

1.下列需要纳入乙类传染病管理的是()A.流行性出血热、B.狂犬病C.流行性乙型脑炎 2.与传染病潜伏期的长短有关的是()A.病原体的数量B.病原体的定位部位 C.病原体在机体内繁殖的时间

3.诊断重度慢性肝炎的指标包括(A.白蛋白(ALB)≤32g/LB.总胆红素(TBiL)大于正常值的5倍

C.凝血酶原活动度(PTA)为40%~60% D.胆碱酯酶(CHE)〈2500U/L E.低热 4.下列可通过血液传播的传染病有 B.乙肝C.丙肝D.梅毒

5.按临床表现,中毒型痢疾可分为 A.休克型B.脑型C.混合型 6.疫源地消毒分为()A.随时消毒B.终末消毒

7.目前公认的艾滋病传播途径主要有()A.性接触B.经血传播C.母婴垂直传播 8.艾滋病的临床分期有()A.急性感染期B.无症状感染期 D.艾滋病前期E.艾滋病期 9.艾滋病期的临床表现包括 A.出现非特异性的全身症状 B.神经系统症状 C.严重机会性感染 D.继发性肿瘤 E.免疫缺陷继发的其他感染 10.隔离衣的应用指征包括

A.护理病人工作服有可能被病人的分泌物、排泄物、血液、体液污染 B.给腹泻或便失禁的病人换单,处理引流管等 C.进入严重感染性疾病病人的隔离室 D.进入免疫功能极度低下病人隔离室

E.进入具有特殊流行病学意义以及多重耐药菌株感染病人隔离室 11.使用隔离衣需注意的事项有 A.使用前认真检查隔离衣有无破损 B.隔离衣使用中发现有渗漏要立即更换 C.处理大面积烧伤与伤口要穿无菌隔离衣 D.隔离衣使用中被病人血液、体液污染应立即更换 E.脱隔离衣时要注意避免衣袖触及面部或衣领引起的污染

12.物理消毒方法包括()B.热力灭菌 C.紫外线消毒 D.电离辐射消毒 E.微波消毒

13.下列属于洗手的指征有 A.接触病人前后

B.接触干净物品前和处理污染物品后 C.无菌操作前、后 D.脱手套后

E.接触破损的皮肤黏膜和伤口前、后 14.下列属于突发公共卫生事件的有 A.传染病疫情暴发或流行 B.群体不明原因疾病 C.重大食物中毒 D.重大职业中毒

E.其他严重影响公众健康的事件

15.特别重大突发公共卫生事件主要包括()A.肺鼠疫在大、中城市发生并有扩散趋势 B.发生传染性非典型肺炎病例,并有扩散趋势 C.涉及多个省份的群体性不明原因疾病,并有扩散趋势 16.社区医务工作者接到疫情通报后,需要做好的工作有(A.隔离病人

B.协助CDC对密切接触者进行医学观察 C.负责社区卫生服务机构的日常消毒和终末消毒)17.突发公共卫生事件的责任报告单位和报告人包括()

A.县级以上各级人民政府卫生行政部门指定的突发公共卫生事件监测机构 B.各级各类医疗卫生机构、卫生行政部门、县级以上地方人民政府 C.检验检疫机构、食品药品监督管理机构 D.环境保护监测机构、教育机构

E.执行职务的各级各类医疗卫生机构的医疗卫生人员、个体开业医生 18.医源性感染常见的传播方式有()A.经医疗器械和设备传播 B.经血液及血制品传播 C.经药物及各种制剂传播 D.经医疗用品传播 E.接触传播

19.医源性感染的临床特征为()B.病人反应性差 C.并发症多 D.临床表现多变

20.医院感染监测的内容包括()A.医院感染病例监测 B.消毒灭菌效果监测 C.环境卫生学监测 D.合理使用抗菌药物监测 E.法规和规章制度执行情况监测

21.下列属于高度危险性物品的是()A.手术器械和用品 B.输血和输液器材 C.注射药物和液体

1.社区精神卫生的最基本服务形式是B.社区医疗

2.从事精神卫生工作的医护人员应遵循的首要伦理道德规范C.尊重患者的人格与权利 3.精神障碍康复的任务不包括E.强化患者的遵医行为

4.世界精神卫生联盟的《卢克色尔精神病人人权宣言》发布的时间是D.1989 5.世界精神卫生联盟的《卢克色尔精神病人人权宣言》规定,精神病患者享有的基本权利所涉及的条文数是 B.9 6.《中国新刑法》第335条规定:医护人员由于严重失职,造成就诊人员死亡或者严重损害就诊人员身体健康的,处以有期徒刑或者拘役的时间是 A.5年以上 B.5年以下 C.3年以上 D.3年以下 E.2年以上

7.1946年提出心理健康4个标志的组织机构是 A.国际心理卫生大会 B.联合国教科文组织 C.世界卫生组织 D.国际心理学会 E.美国心理学会

8.社区精神卫生服务中的6项主要任务,其中不包括

A.保健 B.护理 C. 工疗 D.康复 E.健康教育

9.1990年通过我国第一部《残疾人保障法》的组织机构是 A.全国残疾人联合会 B.国家卫生部 C.全国人大常委会 D.全国人民代表大会 E.全国精神病学会

10.社区精神卫生服务主要涉及的重点人群,不包括

A.儿童 B.在职人群 C.灾后人群 D.被监管人群 E.重度精神病发作者

11.我国现代学者认为,心理健康标准所涉及的条文数量是A.10 12.社区精神卫生要求为所在辖区提供全方位精神卫生服务的对象是C.易感人群 13.精神科护理涉及的失职行为不包括E.有辱患者尊严

14.有关精神卫生方面的描述,不正确的是E. WHO提示,发展中国家的精神发育迟滞患病率高于发达国家

15.有关精神障碍的描述,错误的是

B.许多精神障碍的病因至今未明,实施三级预防难度

16.精神障碍康复中对患者实施训练的能力不包括C.职业活动 17.有关心理健康的描述,不正确的是

E.尚无具体、普遍适用的心理健康标准的评价体系

18.未进入我国“取得精神障碍的康复与防治工作成功经验”先进省市之列的是 A.上海 B.山东 C.北京 D.辽宁 E.广东

19.1958年在南京召开的全国第一次精神病防治会议制定的“16字”工作方针不包括C.预防复发

20.不属于社区康复服务形式之“基层专科”工作内容的是A.开设职业康复训练课程

二、多项选择

1.社区精神卫生涉及的重点人群包括 A.灾后人群 B.在职人群 C.老年人 D.被监管人群 E.儿童

2.判断某人有无知情同意过程的决定能力的参考标准是 A.能否正确理解相关信息 B.能否明了自身状况 D.能否表达自己的意愿 E.能否正确表达自己的决定

3.下列有关精神卫生工疗站的叙述正确的有

B.接纳对象为本辖区内有一定劳动能力的精神障碍患者 D.康复内容包括:职业康复、医疗康复和心理社会康复 E.根据患者的劳动情况,可定期发给一定补贴 4.下列有关精神障碍康复的叙述正确的是 A.是康复医学的分支学科

B.其基本要求是最大限度地恢复适应社会生活的精神功能 C.多方密切配合,是保证患者顺利康复的关键 D.服务的对象包括精神疾病的残疾者 5.社区精神卫生的任务主要包括 A.培训基层精神卫生保健人员 B.开展精神疾病的流行病学调查 D.开展多种形式的社区精神卫生服务 E.精神卫生宣教

6.属于基层精神卫生保健人员的是 A.厂矿医院医生 B.乡村医生 C.工厂车间卫生员 D.机关单位保健医生 E.街道医院兼职医生

1.社区护士接到一位女士来电,讲述自己的婚姻不顺,情绪低落,只想自杀。这时该护士最主要的任务是 E.说服对方放弃自杀的观念

2.典型抑郁症1天之内情绪波动的规律是A.晨重夜轻 3.重症抑郁发作最常见症状表现为显C.抑郁心境

4.患者,女性,40岁,已婚,近1个月来无诱因出现情绪低落,晨重夜轻,对生活失去信心,食欲减退、精力下降,有自杀意念,睡眠差,有早醒,该患者最可能的诊断是 D.抑郁症

5.社区护士怀疑患者有自杀观念时,下列错误的护理措施是B.不能谈论自杀问题 6.关于单纯型精神分裂症,下列说法不正确的是 E.常伴有幻觉、妄想 7.关于偏执型精神分裂症,下列说法不正确的是 D.常出现显著的人格改变

8.下列关于急性精神分裂症急性发作的处理错误的是 D.介绍具体知识和新的应付方法

9.老年痴呆的后期主要表现为 D.生活不能自理 10.老年痴呆病程的特点为 C.进行性发展加重

11.对疑病观念有较好疗效的心理治疗方法是 A.森田治疗 12.下列不属于癔症性躯体障碍的主要表现的是 A.癔症性遗忘 13.癔症的首选治疗方法是D.暗示治疗 14.恐惧与焦虑的区别在于B.有无特定对象 15.下列关于神经症的叙述为正确的是

D.神经症是一组轻型精神病

16.下列对于抑郁的认识错误的是

E.经过治疗,能100%的恢复

17.社区护士对待精神分裂症患者的态度错误的是

C.以批判的态度对待他们

18.社区老年痴呆患者家庭护理管理最优先考虑应是 A.帮助病人维持最佳的功能状态

19.下列关于神经症患者的特点的描述错误的是

D.有相应的器质性病变基础

20.治疗焦虑症最重要的一步是

A.让患者了解焦虑症的本质和所存在的焦虑症状 21.一般的社区精神卫生护理对象是D.神经症患者

四、多项选择题

1.精神分裂症的临床类型主要包括 A.偏执型 C.紧张型 D.青春型 E.单纯型

2.对广泛性焦虑症及惊恐发作具有较好疗效的药物主要包括 B.地西泮 C.阿米替林 D.丙米嗪

3.下列关于“老年痴呆的正确叙述”包括 B.阿尔茨海默病的发病率女性多于男性 C.血管性痴呆发病率男性多于女性

E.阿尔茨海默病是一组原因未明的原发性退行性脑变性疾病 4.关于“精神分裂症预后”的正确叙述包括()A.第一次发作的精神分裂症患者中75%可治愈 B.25%得到完全康复

C.35%可能变成长期的功能残疾 D.40%反复发

E.约60%的患者可以达到社会性缓解 5.强迫症的特点包括()A.有意识的自我强迫和反强迫并存 B.患者体验到的观念和冲动都来源于自我 C.违反自己的意愿、需极力抵抗,但无法控制 D.明知不必要,但无法摆脱

6.下列关于癔症的描述正确的是()A.没有可以证实的器质性病变 B.症状常具有做作、夸大和夸张色彩

C.有反复发作的倾向D.近年发病率有上升趋势 E.发病主要原因一般是心理社会因素

⒈下列不属于社区急救的优势和自身特点的选项是: A.具有地利优势和时间优势

2.下列不属于社区居民的自救互救网络的选项是: C.单元招募志愿者

3.对急救“白金10分钟”的描述正确的选项是: E.时间就是生命

4.下列不属于在突发公共卫生事件中社区卫生服务机构应采取的措施选项是:B.开展病人初诊、救治和转诊工作

5.下列符合我国社区急救发展方向的社区急救模式是: C.制定突发公共卫生事件社区(乡镇)应急预案 6.下列不能反映建立社区急救重要性的选项是:

E.社区急救具有先进的通讯网络优势和急救仪器设备优势 ⒈关于心源性晕厥,描述正确的选项是:

A.具有较大的危险性;B.严重缺氧的患者容易发生;C.发作时心输出量增加;D.见于老年人;E.不需抢救,可自然缓解。

⒉关于短暂脑缺血发作(TIA),描述正确的选项是:

A.多发于40岁以上的人;B.常于卧位或活动时发生;C.多发生于高血压病、糖尿病患者;D.持续时间超过一小时;E.发作前常有头痛。⒊关于中度昏迷,描述正确的选项是:

A.患者对外界一般刺激无反应;B.各种反射均减弱 ;C.无眼球转动 ;D.可出现病理反射。E.以上都正确。

⒋关于晕厥的现场救护,描述正确的选项是:

A.所有患者都需要立即送医院;B.首先要确认患者是否为心源性晕厥;C.立即让患者口服糖水;D.拨打急救电话;E.所有患者都不需要去医院。⒌关于昏迷的临床判断,描述正确的选项是:

A.口唇皮肤樱桃红色多见于肝胆疾病或溶血;B.烂苹果味见于尿毒症;C.脑膜刺激征多见于蛛网膜下腔出血;D.大蒜味见于肝脏疾病;E.发热多见于酒精中毒。⒍关于导致昏迷的病因,描述正确的选项是:

A.急性脑血管病;B.糖尿病;C.颅脑外伤;D.代谢障碍;E.以上都可以。

四、多项选择

⒈惊厥的临床特征包括:

A.多见于小儿;C.常伴有呼吸不规则;D.紫绀;E.尿便失禁。⒉能够正确描述心源性晕厥的选项是:

A.常见于急性心肌梗死患者;C.常见于病窦综合症患者;D.患者有发生心搏骤停的危险 ⒊关于反射性晕厥,A.也称为神经介导性晕厥;B.发病率占全部晕厥的80~90%;C.危险性较小,预后较好;D.老年人及体弱者容易发生; ⒋窒息性晕厥的病因:

C严重的低氧血症.D.急性一氧化碳中毒;E.阿片类药物中毒。⒌关于低血糖晕厥,描述正确的选项是: B.常见于糖尿病患者;E.补充糖分后即可缓解。⒍关于脑膜刺激征,描述正确的选项是:

A.主要表现之一为颈项强直;C.阳性者可见于蛛网膜下腔出血及脑膜炎;

⒈ 2005年心肺复苏和心血管急救指南建议心肺复苏时气管内注射药物的剂量是静脉用量的: B.2~2.5倍;

2.长时间的按压造成急救者的疲劳可能导致按压幅度不够和质量下降。2005年指南指出如果现场有两名或更多的救助者,需要更换按压者的时间是:E.2分钟 ⒋正确的心脏按压应使:A.成人患者胸骨下陷4~5公分; ⒌面罩气囊正压人工呼吸每次供气约为: A.500~600ml; 6.诊断心搏骤停的主要依据是:C.意识突然丧失,呼吸停止;

7.按压时上臂要垂直,同时肘关节要伸直,操作者上臂与患者双肩的连线的交角应为:B.90° 8.对目击下的心搏骤停,现场复苏时应首先实施: A.人工呼吸,心脏按压

四、多项选择

⒈对心搏骤停患者进行脑复苏的主要作用是:A.采取各种措施尽快恢复向脑部供血供氧;C.改善脑细胞代谢;D.消除导致脑细胞损害的因素;E.减轻脑细胞损伤

⒉复苏时扩容药物的选择是静脉滴注:A.小分子或低分子右旋糖酐;B.706代血浆;C.平衡液;D.林格氏液;

⒊ 2005年心搏骤停的复苏指南建议,人工呼吸和心脏按压的配合是: A.成人心脏按压和呼吸之比为30:2;C.成人和婴儿心脏按压和呼吸之比均为30:2;

⒋容易导致心搏骤停的心血管危险因素有:

A.吸烟;B.高脂血症;C.高血压病;D.糖尿病;E.冠心病家族史 5.可以用于脑复苏的药物是:A.地塞米松;B.甘露醇;D.冬眠一号;

6.生命链的内容是:A.早期呼救;B.早期心肺复苏;D.早期电击除颤;E.早期高级复苏 ⒈食物中毒的重要诊断依据是:A.有食入可疑或不良食物史 ⒉毒鼠强中毒的典型表现是:B.持续抽搐; ⒊急性一氧化碳中毒的最佳治疗方法是:C.高压氧; ⒋阿片类药物和海洛因类毒品中毒的最佳治疗药物E.纳洛酮 5.毒鼠强中毒的关键治疗是:B.止痉治疗;

6.下述哪项是诊断中毒的可靠依据:E.食入同一食物的人群中有多人发病

四、多项选择 ⒈催吐的禁忌征是:

A.口服腐蚀性毒物者;B.孕妇;C.主动脉夹层患者;E.患有食道胃底静脉曲张、胃溃疡者 ⒉昏迷伴有口唇红润的中毒多见于:

A急性一氧化碳中毒;B.氰化物中毒;C.有机磷农药中毒;D.毒鼠强中毒;E.安眠药中毒 ⒊下述导致急性一氧化碳中毒的因素是:

A.环境中有一氧化碳的产生;B.房间内一氧化碳的产生量大于其散发量;C.在通风不良的房间吃烧碳的火锅;E.炉火即将熄灭时

⒋杀鼠剂中毒的院前急救要点是:A.迅速采用催吐洗胃法;C.迅速送患者去医院;D.迅速建立静脉通道;

⒌中毒的现场急救原则是:

A.帮助患者迅速脱离中毒环境;B.减少毒物吸收;C.加速毒物排泄;D.尽快送患者去医院;E.搜集毒物证据

⒍安眠药中毒的主要表现是:C.中枢神经抑制的表现;D.呼吸系统抑制的表现;E.循环系统抑制的表现

⒎表现为昏迷伴有严重的皮肤及口唇青紫的中毒见于: C.亚甲蓝(美蓝)中毒;E.亚硝酸盐中毒 ⒏表现为昏迷伴有双侧瞳孔缩小的中毒见于:

A.阿片类药物;B.有机磷农药;D.某些安眠药;E.海洛因类毒品

9.近年来我国职业性中毒和农药中毒的发生率下降,而生活性中毒的发生率增加,其中主要的中毒原因是:

A.镇静催眠药物中毒;B.一氧化碳中毒;C.酒精中毒;.E.海洛因类毒品中毒 10.昏迷伴有双侧瞳孔扩大见于: A.肉毒杆菌中毒;B.阿托品类药物中毒 ⒈毒蛇的特征:A.头部多呈三角形;

⒉国际救助优先顺序中的第一优先用什么颜色的伤票填写:C白色; ⒊容易导致电击伤的最主要因素有:E.忽略了触电因素 ⒋容易导致室颤的电流频率是:B.50~60赫兹; ⒌结扎止血带部位是伤口的近心端:.C10厘米处;.⒍狂犬病的主要传染源是: E.吸血蝙蝠

四、多项选择

⒈我国危害较大的毒蛇有:A.眼镜蛇和眼镜王蛇;C.金环蛇和银环蛇;D.蝰蛇;E.腹蛇和竹叶青

⒉关于触电的描述内容正确的选项是:A.被闪电击中后,患者的皮肤呈红色网状或树纹样图案;C.触电死亡的机制是心室颤动;D.可出现迟发性假死现象; ⒊可以造成特殊烧伤的原因包括:A.火焰;B.电击;C.强酸;D.强碱;

⒋属于重度烧伤的情况是:C.吸入热气造成的呼吸道烧伤;D.会阴部的烧伤;E.烧伤伴有其他复合伤

⒌狂犬病的潜伏期是:B.10~12小时;E.1~2周。

⒍对蜂毒过敏的患者被蛰后可出现:A荨麻疹;C.喉头水肿; 第九章

1.临终关怀的首要工作

2.当病人诉说的疼痛强度与他得行为不相符时,护士应该B.如实记录病人主诉的疼痛强度并及时处理

3.下面不是病人临近死亡时的常见的表现有E. 抑郁状态

4.对阿片类药物成瘾性的正确理解是D.指为了得到精神上的快感而不择手段地获取药物的行为

5.对止痛药三阶梯用药原则的含义正确的理解是 A.根据病人疼痛强度选择不同阶梯的止痛药物 6.阿片类药物引起的终生不耐受的不良反应是C.便秘 7.不属于常用的促进有效沟通的方法 E.闭合性问题

8.关于临终关怀的目标,不正确的叙述是C.为了减轻痛苦可加速死亡 9.对医护人员开展死亡教育的目的不包括D.促进安乐死的实施 10.对临终病人及家属进行死亡教育的要点不包括

C.经常教育病人要面对死亡

11.关于悲伤的特征的叙述,不正确的是 E.所有悲伤反应都是正常的

12.关于阿片类药物引起的便秘,正确的叙述是

A.服用阿片类药物同时应服用缓泻剂预防 13.关于阿片类药物引起的镇静,不正确的叙述是 E.阿片类药物的镇静可以增加镇痛效果,不必处理 14.关于阿片类药物引起的恶心呕吐,不正确的叙述是 E.如出现呕吐,立即停用阿片类药物 15.使用芬太尼透皮贴剂不正确的叙述是 B.只能贴在身体疼痛部位

16.关于阿片类药物的生理依赖性,不正确的叙述是 E.即阿片类药物的成瘾性

17.关于阿片类药物的耐药性,不正确的叙述是 C.尽量不加量,以免以后不起作用

18.关于疼痛的社会心理干预方法,不正确的叙述是

D.可以代替药物治疗疼痛

19.区分便秘和粪便嵌塞的最有效的方法

B.肛门指诊

20.关于止痛药给药时间,正确的叙述是

E.对持续性疼痛,应按时服用控/缓释制剂保持无痛

四、多项选择题

1.在生命最后48小时常出现的问题包括: A.呼吸道分泌物多 B.疼痛 C.烦躁不安 E.恶心/呕吐

2.引起终末期肿瘤病人腹泻常见原因有 A.由肿瘤本身引起 B.与手术操作有关的 C.放疗引起的放射性肠炎 D.与药物副作用有关

3.告诉临终病人实情,正确的做法是 A.选择合适的时间和地点 B.从病人的角度来考虑他的感受 C.给病人足够的时间来宣泄情绪 D.不要回避病人提出的问

4.常见的阻碍有效沟通的语言和行为有 A.不认为病人的感受是独特的 B.强加自己的观点给病人 E.将问题推给别人去解决 5.关于压疮的护理,正确的叙述是

A.大小便失禁、腹泻、肠瘘、阴道膀胱瘘病人更容易出现 B.定时翻身是最有效的预防措施

C.在骶尾部、骨隆突处及其它受压部位使用减压用品 D.增加营养 E.制动减少摩擦

1.肺炎球菌性肺炎的患者发热时常见的热型是 稽留热

2.下列患者中,不宜进行直肠测体温的是:B.腹泻者 3.为脉短绌的患者,测量心率、脉率的正确方法是:

C.一人测心率,另一人测脉率,同时测lmin 4.正常成人每分钟呼吸次数为: B.12~16次 5.属于节律异常的呼吸是: E.潮式呼吸

6.为患者测量血压时,使测得血压值偏低的原因可能是C.缠袖带过紧 7.易发生体温过低的是A.早产儿

8.物理降温后,为了观察效果,再次测量体温的时间是:E.30min 9.浸泡无菌持物镊的消毒液,其液面应在镊子的2/3处 10.在无菌操作中发现手套破裂应 B.立即更换后,继续操作 11.取用无菌溶液的方法正确的是C.手不可触及瓶口 12.铺好的无菌盘,若未被污染其有效时间为C.4h 13.打开的无菌包,若未被污染其有效时间为D.24h 14.无菌持物钳的正确使用方法是B. 一个容器仅可放置一把持物钳 15.取用无菌溶液时,应首先核对A.瓶签

16.符合无菌操作原则的描述是:C.操作时手臂保持在腰部水平以上 17.有关无菌技术操作的描述正确的是B.将无菌盘盖巾扇形折叠开口边向外 18.使用无菌容器正确的操作是E.手指不可触及容器内面 19.有关服药方法的描述,正确的是D.硝酸甘油置于舌下含服

20.有关取药的方法不正确的是D.服用几种液体药物时,应混合放置在一起 21.用滴眼法给药时,应将眼药滴入:D.眼下穹隆

22.使用手压式雾化吸入器的正确方法是B.使用前充分摇均匀 23.煎煮中药时建议先煎的是C.贝壳类

24.有关中药服药方法的描述正确的是D.补益剂,饭前服 25.预防接种行皮内注射时,最常选用的部位是: D.上臂三角肌下缘

26.关于抽吸药液的方法,正确的是: E.手握活塞,进行抽吸

27.有关臂大肌注射时体表定位联线法的描述正确的是 E.髂前上棘和尾骨联线的外上1/3处

28.为患者进行臂大肌注射时,患者侧卧的正确姿势 B.上腿伸直,下腿稍弯曲

29.静脉注射葡萄糖最佳的注射部位是C.股静脉 30.上臂三角肌肌内注射的部位是 D.上臂外侧,肩峰下2~3指

31.静脉注射过程中,患者局部肿胀、疼痛、试抽有回血,可能原因是:C.针头斜面一半在血管外 32.静脉注射过程中,局部疼痛、肿胀,试抽无回血,可能的原因是: E.针刺入过深,药物注在组织间隙

33.造成滴管内液面自行下降的原因可能是D.滴管有裂隙 34.有关止血带止血法的描述正确的D.每 50分钟放松3-5分钟 35.有关包扎的描述不正确的是C.直接用消毒剂消毒伤口 36.给病人鼻饲,胃管插入长度相当于病人从C.前发际至剑突长

37.为病人插胃管过程中,如病人出现呛咳和呼吸困难,护士应E.立即拔出胃管 38.鼻饲操作方法错误的是E.注入少量温开水证实胃管是否再胃内 39.为病人进行鼻饲时,流质饮食的温度是B.38~40℃ 40.为提高昏迷病人插胃管时,采取的措施是 B.使病人的头贴近胸骨

41.为昏迷病人插胃管时,当胃管插至会厌部时,要将病人头部托起,其目的是D.加大咽喉部通道的弧度

42.为病人鼻饲灌食后,再注入少量温开水的目的 C.冲洗胃管,避免堵塞

44.给男病人导尿时,提起阴茎与腹壁成60o角是使C.耻骨前弯消失 45.成年女性导尿时,导尿管插入长度是B.4~6cm 46.用气囊型导尿管进行留置导尿时,应在见尿后再将导尿管插入 C.5~7cm 47. 长期留置导尿患者需定期更换导尿管的目的是D.防止逆行感 48.长期留置导尿后,尿液出现浑浊、沉淀或结晶时应 B.多饮水,并进行膀胱冲洗

49. 行大量不保留灌肠时,成人每次用液量为D.500~1000ml 50. 行保留灌肠时药量不应超过C.200ml 51.行保留灌肠时,肛管插入直肠深度约C.10~15cm 3.行保留灌肠时,下列操作正确的是

A.灌肠前嘱病人先排便C.液面与肛门距离<30cm

52.肝昏迷的患者灌肠禁用溶液是B.肥皂水 53.肝昏迷病人灌肠时禁用肥皂水溶液,其原因是 C.减少氨的产生及吸收

54.行小量不保留灌肠时,灌肠筒液面距离肛门应在B.30cm以下

55.行小量不保留灌肠配制“1、2、3”溶液时,硫酸镁、甘油和温开水的量是A.30ml,60ml,90ml 56.需采用全血标本的检验项目是 A.血脂

57.同时抽取不同种类的血标本,注入试管内的顺序正确的是

E.血培养瓶—>抗凝管—>干燥管

58.血培养标本的取血量一般为 B.5ml 59.留24小时尿标本作17-酮类固醇检查时,应在标本中加入C.浓盐酸 60.采集便标本作血吸虫卵化检查时应 D.取不同部位的粪便 61.留取中段尿主要是为了检查尿中B.细菌 62.采集痰培养标本时应用的漱口液是 A.生理盐水 63.痰常规标本应何时采集 C.清晨

64.王某,几日来大便次数增加,每日10次左右,便中带血及黏液,呈果酱样,疑阿米巴痢疾,需取便标本,留取的标本应置

D.加温的容器

65.张某,女,2岁,白血病,化疗过程中因口腔溃疡需做咽拭子培养,采集标本部位应选B.两侧腭弓

66.鼻导管给氧,导管插入的长度为: A.鼻尖至耳垂D.鼻尖至耳垂的2/ 67.停止氧气的正确方法是:

E.取下鼻导管,关紧流量表再关总开关,重开流量表 68.漏斗胃管洗胃法是利用C.虹吸原理

69.洗胃时,一次洗胃液灌入量应不超过:E.500ml 70.下列禁忌洗胃的药物中毒是:E.硫酸 71.为昏迷病人作口腔护理时,正确的是: C.从里向外擦净口腔及牙齿的各面

72.用1%-3%过氧化氢溶液进行口腔护理是,正确的是

B.遇有机物时,可释放氧气抗

73.不属于压疮诱发因素的是: D.肌肉萎缩 74.最容易发生压疮的C.全身瘫痪病人 75.预防压疮发生最有效的护理措施

E.协助经常更换卧位,防止局部组织长期受压

76.孙老先生,因心衰卧床,一日,发现骶尾部皮肤红、肿、起小水泡,皮下有硬结,有时有渗液,患者诉疼痛,你判断病人此情况属于:B.炎性浸润期

77.朱先生,诊断为再生障碍性贫血,检查发现唇和口腔粘膜有散在瘀点,轻触牙龈出血,为其进行口腔护理应特别注意:A.动作轻、稳

78.下列有关婴儿抚触的描述正确的是C.进食1h后进行

79.有关伤口换药技术的描述正确的是B.清洁伤口由创缘向外消毒 80.乳房自检的最佳时间B.月经期后一周内实施

81.下列有关指压止血法的描述正确的是:A.手指压迫近心端的动脉 【多选题】

1.影响血压的因素包括 A.心输出量 C.外周阻力 E.动脉管壁直径

2.下列关于无菌技术操作原则的叙述中,正确的是 A.操作前护士应洗手、戴口罩和工作帽 B.无菌物品须用无菌持物钳夹取 4.下列病人可给予鼻饲饮食的是 A.昏迷病人 B.口腔疾患病人 C.拒绝进食的病人 D.食道梗阻的病人

E.早产儿和病情危重的婴幼儿

5.下列临床表现属于压疮淤血红润期的是

C.受压部位红、肿、热、痛

D.皮下有硬结

E.皮肤表面无破损

第三篇:全科医生培训考试试题

1.我国目前卫生资源配置的现状是卫生经费投入C.重临床,轻预防

2.世界卫生组织与世界家庭医生组织倡导全科医生和专科医生的比例是E.1:1

3.我国医疗卫生服务模式的多元化需求是受到何种影响引起的D.人民群众

4.当前我国卫生服务模式改革的方向是A.国家对卫生资源的投入更多放在广大人民群众基本医疗需求上

5.对当前疾病谱和死因谱叙述错误的是C.高血压发病率降低

1.我国社区卫生服务的发展计划分为几个阶段B.3个

2.下列哪项不是我国社区卫生服务的特点E.所承担的医疗任务不同

3.当前我国的医疗卫生服务主要由何种机构承担B.改造后的一、二级医院

4.我国社区卫生服务发展的框架建设的时间是D.2000-2005

5.第一次在中央文件中提出发展社区卫生服务是在A.1997

1.当前我国卫生资源的配置特点是C.重点在县(区)

2.当前我国社区卫生服务人员的构成特点哪种说法是正确的E.专业构成比较合理

3.当前我国卫生服务机构的特点说法错误的是C.不能开展计划生育工作

4.对当前我国社区卫生服务存在问题的叙述错误的是B.政府支持力度逐步加大,社区卫生服务可持续发展

5.我国社区卫生服务工作人员主要由哪部分人构成A.从基层卫生机构转型而来

1.下面对社区的叙述错误的是A.社区是相同民族的人群构成的居住区域

2.社区卫生服务的特点叙述错误的是B.以疾病为中心

3.全科医生和专科医生的区别在于A.全科医生以生物-心理-社会医学模式为基础,专科医生以生物医学模式为基础

4.社区医学的特性不包括E.全面性服务

5.全科医学的“持续性服务”是指C.全科医生对人生各阶段以及从健康到疾病的各阶段都负有健康管理责任

1.定量研究技术包括:A.使用结构化问卷调查表

2.结构性问卷的特点不包括:B.不易统计

3.定量调查设计原则包括:A.目的明确、重点突出;B.调查数量足够;C.随机化;D.在相同条件下比较;E.以上都是

4.人口学信息不包括下列哪项?E.购买力和人

5.定性调查技术不包括下列哪项?B.使用结构化问卷调查表

1.加强质量管理的关键是A.加强全科医疗服务机构的各级各类人员的质量意识

2.医疗卫生服务质量管理的主要方法是A.建立高质量的质量管理队伍

3.全科医疗服务管理水平不断提高的标志是C.积极推行和扩大标准化管理

4.不是社区卫生服务绩效评价指标筛选原则的是A.综合性;B.时间性;C.领导的需求;D.可比性;E.导向性

5.社区卫生服务绩效评价指标中的效果、效益指标不包括A.健康促进指标;B.公共卫生指标;C.信息管理指标;D.健康档案指标;E.妇幼保健指标

第四篇:全科医师考试试题(七)

医学心理学复习题

A1型:

1.健康指的是:

A.身体处于良好状态。

B.身体和道德品质处于良好状态。

C.身体和心理处于良好状态。

D.身体、心理、社会适应和道德品质处于良好状态。

E.身体、心理和社会适应处于良好状态。2.医学心理学的主要任务是:

A.教学

B.科研

C.心理评估

D.心理治疗

E.以上都是 3.关于生物一心理一社会医学模式的特点为:

A.认为任何身体疾病必定有其特殊的生物学原因

B.认为没有躯体疾病就等于健康

C.认为疾病是在生物和心理因素共同作用下形成的,与环境无关

D.认为引起疾病的最主要的因素是心理因素

E.认为临床医学服务的对象是病人而不仅仅是疾病。4.医学模式是:

A. 某一时代各种医学思想的集中反映

B.某一时代各种医学学派的集中反映 C.对医学各门类的总称

D.对医学知识和技术的总称. E.对医学知识和方法的总称 5.生物-心理-社会医学模式是:

A.一种系统论和整体观的医学模式

B.在健康和疾病的问题上关注生物心理社会因素的综合作用

C.认为人是多层次的和完整的连续体

D.顺应时代发展的必然产物

E.以上都是

6.下列不属于精神分析人格理论的内容是:

A.自我B.本我C.潜意识

D.超我E.非我 7.遵循现实原则的人格部分是:

A.超我 B.本我 C.性力

D.自我 E.意识 8.皑皑白雪,在晚霞的映照下,呈现出一片红色,但是我们对雪地的知觉仍然是

A.理解性

B.对比性

C.整体性 D.组织性

E.恒常性

9.先吃糖,后吃西瓜,会感到西瓜不甜。这是因为人的感觉具有:

A.对比性

B.感受性

C.适应性

D.发展性

E.选择性 10.“横看成林侧成峰,远近高低各不同”形象说明:

A.认知的相对性

B.认知的发展性

C.认知的多维性

D.认知的整合性

E.认知的适应性 11.“草木皆兵”主要是以下哪种原因引起的:

A.感觉神经发生病变

B.幻觉

C.想象性错觉

D.感受性错觉

E.情绪型错觉 12.短时记忆的时间是:

A 0.25~2秒

B.2~5秒

C.

5秒~1分钟

D.1分钟~5分钟

E.5分钟~10分钟 13.过去经历过的事物再度出现时仍能认识,称为:

A.再现

B.再认

C.追忆

D.识记

E.保持 14.有关马斯洛五个需要层次由低到高的顺序,以下哪项正确:

A.生理需要,安全需要,爱和归属需要,尊重需要和自我实现需要

B.生理需要,爱和归属需要,安全需要,尊重需要和自我实现需要

C.生理需要,安全需要,尊重需要,爱和归属需要和自我实现需要

D.生理需要,尊重需要,爱和归房需要,安全需要和自我实现需要

E.生理需要,尊重需要,安全需要,爱和归属需要和自我实现需要 15.人格的核心是:

A.能力

B.性格

C.智力

D.气质

E.理想 16.比较微弱又比较持久的情绪状态指的是:

A.心情

B.心境

C.应激

D.激情

E.以上都不是

E.感觉、记忆、比较、分析和概括。17.关于情绪和情感,以下哪项是正确的:

A.情绪具有持久性,情感具有短暂性

B.情绪带有稳定性,情感具有冲动性 C.情绪多与社会需要相联系,情感多与自然需要相联系

D.情绪和情感都是对客观事物态度的主观体验

E.以上都不对 18.按照现代心理学界的标准,四项基本情绪是:

A.快乐、恐惧、悲哀和愤怒

B.忧虑、快乐、悲哀和恐惧 C.忧虑、快乐、恐惧和愤怒

D.快乐、悲哀、恐惧和愤怒 E.忧虑、快乐、愤怒和恐惧

19.有关人格的心理特征的组成,以下哪项正确:

A.能力、智力、气质和性格

B.需要、动机、能力和性格 C.动机、思维、能力和性格

D.能力、智力、思维和性格 E.动机、思维、气质和性格 20.A型行为特征不包括

A.脾气急躁

B.时间紧迫感

C.争强好胜

D.对人有敌意

E.对环境强烈不满

21.与恶性肿瘤有密切联系的行为是:

A. A型行为

B.B型行为

C. C型行为 D. D型行为

E.E型行为 22.一年内LCU累计分为250,则来年患心身疾病的概率为

A. 无可能性

B、30%

C.50%

D.70%

E. 100% 23.“杞人忧天”故事中的“杞人”面对的应激源属于:

A.躯体性应激源

B.文化性应激源

C.心理性应激源

D.社会性应激源

E.沟通性应激源

24.自觉地确定目的,并根据目的支配自己的行为,克服困难以实现目的的心理过程是

A.认识

B.意志

C.情感

D.感知

E.思维 25.关于心理应激,错误的说法是

A.可引起生理反应

B.可引起心理和行为反应

C.对身心健康产生不利影响

D.经过认知评价

E.可能提高工作效率 26.影响挫折感受的主要因素是:

A.抱负水平和个人容忍力

B.情绪因素

C.环境因素 D.心理动机冲突

E.想象能力 27.动机受干扰阻滞,被迫放弃而导致需要不能满足的负性情绪状态,叫做

A.动机冲突

B.挫折

C.抑郁

D.应激

E.心境 28.下列疾病中,哪个不属于心身疾病

A. 十二指肠溃疡

B.抑郁症

C.癌症

D.糖尿病

E.支气管哮喘 29.日常麻烦带来的苦恼所构成的应激源称为:

A.困扰

B.担忧郁

C.焦虑

D.痛苦

E.以上都不是 30.社会支持有多种形式,但不包括:

A. 给预信息

B.给预指导和关怀

C.给预经济资助 D.给预影响和教育

E.提供鼓励与保证 31.效度反映心理测量结果的

A.一致性

B.可靠性

C.真实性

D.代表性

E.客观性 32.心理测量工具好坏的基本标志是

A.常模

B.信度和效度

C.指导语

D.记分方法

E.容易操作 33.导致失眠长久愈的重要心理机制是:

A. 不良的自我暗示

B.抑郁

C.焦虑

D.睡眠规律破坏

E.对健康过度关注 34.目前最常用的智力高低表示法是

A. 比率智商 B.离差智商

C.百分位数

D.智力年龄

E.项目数 35.下列哪项是测量人格的:

A.SAS

B.SDS

C.SCL-90

D.EPQ

TAT 36.应提倡的医患关系模式是

A.主动一被动型

B.指导一合作型

C.共同参与型

D.根据具体情况确定

E.以上都不是 37.病人中最常见的情绪反应是:

A. 抑郁

B.愤怒

C.焦虑和恐惧

D.敌意

E.自怜 38.癌症病人听到癌症的诊断后,出现心理反应的顺序是:

A. 休克恐惧期,否认怀疑期,愤怒沮丧期,接受适应期 B. 否认怀疑期,休克恐惧期,愤怒沮丧期,接受适应期 C. 休克恐惧期,否认怀疑期,接受适应期,愤怒沮丧期 D. 否认怀疑期,休克恐惧期,接受适应期,愤怒沮丧期 E.以上都不对

39.医患关系的特点是:

A. 以医疗活动为中心,以维护健康为目的B.是帮助性的人际关系 C.以患者为中心

D.以上均是 40.非言语交流不包括:

A. 面部表情

B.身段表情

C.衣着打扮

D.人际距离 41.大多数终末期病人最早的心理反应是:

A.

否认

B.抑郁

C.愤怒

D.恐惧

E.焦虑 42.由于家庭、工作、环境变化提出新的角色要求时,病人退出病人角色,这是:

A.角色行为减退

B.角色行为缺如

C.角色行为恐惧

D.角色行为冲突

A2型:

1.小张的性格聪明好动、热情、反应敏捷,而且容易兴奋和激动。但是常常缺乏耐心和毅力。这种气质类型属于:

A.粘液质

B.多血质

C.抑郁质

D.胆汁质

E.混合型 2.某男青年,因心爱的女友提出与其终止恋爱关系时,采取了报复行为,毁了女孩的面容,这种由爱变成恨的情绪变化是因为情绪具有:

A.短暂性

B.复杂性

C.两极性

D.持久性能

E.深刻性 3.D在现代社会中,成年人主要将精力消耗在工作中。他们所在工作的企事业中因政策或政策的执行问题会导致很多有关的应激源,但这些应激源不包括:

A. 组织的结构与气氛

B.职业性人际关系

C.个人职业经历 D.社会环境的意外

E.个体在组织中的角色和负责程度 4.一位中所妇女由于对工作的过度需求促使个体感到心力疲惫,处于无从应对的状态。平时感到头痛、疲劳、失眠、情绪低落、沮丧消沉、自暴自弃十分消极。她的这种心身耗状态称为:

A.失望

B.绝望

C.变态

D.崩溃

E.失落

5.某病人的个性特征表现为爱社交,渴望兴奋,喜欢冒先,行动常受冲动影响,反应迅速,乐观,好谈笑,情绪倾向失控,作事欠踏实。若医生给这位病人填写艾森克人格问卷,其结果可能性最大的是:

A.P分高

B.L分低

C.E分高

D.N分低

E.H分高

6.某男士,46岁,患慢性肾功能衰竭多年,准备进行透析治疗。他的以下心理社会条件哪项不利于透析治疗:

A. 家属能够配合B.较高的智力

C.较高的理解力 D.较多的防御态度

E.较少依赖躯体性防御 7.某病人,由于心肌梗塞进入监护病房。入监护病房后病人得了“ICU综合征”。所谓“ICU综合征”是指病人进入监护病房后所产生的:

A. 意识改变和行为异常

B.意识改变和情绪反应

C.感觉异常和知觉异常 D.情绪反应和意志改变

E.以上都是

8.有位病人在与别人发生口角后突然出现严重的胸闷、呼吸困难。他被送到医院急诊室,医生经检查没有发现任何器质性病变。最后决定给病人静脉推了一针生理盐水,并向病人说明此药对病人缓解症状有帮助。病人没注意用的是何种药,在接受治疗的同时也很自然地接受了医生的解释。结果症状果然很快就消失了。医生给病人治疗的技术措施是:

A.对症治疗

B.暗示治疗

C.鼓励治疗

D.疏泄治疗

E.引导治疗

参考答案: A1:

1.D 2.E 3.E 4.A 5.E 6.C 7.D 8.E 9.A 10.C 11.E 12.C 13.B 14.A 15.B 16.B 17.D 18.D 19.A 20.E 21.C 22.C 23.D 24.B 25.C 26.A 27.B 28.B 29.A 30.C 31.C 32.B 33.A 34.B 35.D 36.D 37.C 38.A 39.D 40.C 41.A 42.B

A2:

1.B 2.C 3.D 4.D 5.C 6.D 7.B 8.B

第五篇:全科医师中级职称考试试题1

一、单选题[A1/A2](每题只有一个正确答案,共2000道题)

1、营养不良患儿,皮下脂肪最早消失的部位是A、胸、背部B、四肢C、头面部D、腹部正确答案:E

2、精神病人中,冲动常见于A、抑郁症B、强迫症C、恐惧症D、智能发育不全E、精神分裂症正确答案:E

3、以下不是全科医生的主要历史使命的是A、实施医疗保险B、承担社区群体与个体的三级预防C、发展照顾医学D、重塑医生形象E、成为社区健康的主要组织、实施与监测者正确答案:A

4、全科医生的目标是A、诊断和治疗疾病B、最大限度地满足社区居民的卫生需要C、促进社区健康可持续发展D、实现人人享有卫生保健E、以上都是正确答案:E 解析:E涵盖了全科医生的各种目标。

5、下列何种属性不是全科医疗与专科医疗的区别A、服务对象的稳定性与流动性B、对服务对象承担责任的持续性与间断性C、处理的健康问题常见与少见D、技术水平要求E、服务质量的考核指标为治愈率与病人满意度正确答案:E 解析:无论全科医生,还是专科医生,其服务质量的考核指标都应该是相似的。

6、下列哪一量表可使用自评或他评方式对被试者的日常生活能力进行测定A、日常生活能力量表B、日常生活功能指数C、Pfeffer功能活动调查表D、qWB量表E、PULSES量表正确答案:A

7、下列不属于初级卫生保健基本任务的是A、增进健康B、预防疾病C、随访患者D、治疗病伤E、康复服务正确答案:C 解析:初级卫生保健的基本任务包括:增进健康、预防疾病、治疗病伤和康复服务。

8、新世纪人们医学模式和健康观念的改变是由于A、传染病病死率太高B、明了治疗传染病的抗菌药物C、环境严重污染D、城市人口增多E、慢性非传染性疾病的发病率和死亡率增加正确答案:E 9、21世纪“人人享有卫生保健”的实施策略是A、将与贫困作斗争作为工作重点B、改善公共卫生基础设施和基本卫生服务、控制阻碍经济发展的疾病C、全方位促进健康D、动员各部门合作E、以上均正确正确答案:E

10、我国卫生发展战略在卫生事业重点选择上,要把A、农村作为卫生工作的长期战略重点B、市作为卫生工作的长期战略重点C、西部作为卫生工作的长期战略重点D、中西部作为卫生工作的长期战略重点E、西部和东北部作为卫生工作的长期战略重点正确答案:A

11、老年人最常见的骨折部位是A、Colles骨折B、肩部骨折C、椎骨骨折D、髋部骨折E、骶骨骨折正确答案:D 记忆题。

12、患者,男性,64岁,重度吸烟,慢性咳嗽、咳痰20余年,活动后呼吸困难2年,因高热2天,意识模糊,呼吸困难来院急诊,为判断病情应立即行下列哪一项检查A、急诊胸部CT检查B、床边超声波胸部探查C、动脉血气分析D、血液查氨茶碱浓度E、肺动脉漂浮导管测定肺毛细血管楔压正确答案:C 患者出现呼吸衰竭和意识障碍,动脉血气分析可诊断高碳酸血症及酸碱失衡,并指导下一步治疗。

13、引起婴儿佝偻病的主要原因是A、饮食中缺钙B、甲状旁腺素缺乏C、食物中钙、磷比例不当D、缺乏维生素AE、缺乏维生紊D正确答案:E 维生素D可促进肠道对膳食中钙的吸收,缺乏维生素D可导致佝偻病。

14、患者,男性,62岁,因心前区压榨性闷痛1小时入院,临床症状不除外心肌梗死,为确定诊断采取下列哪一步骤最重要?A、观察血压变化B、抽血测肌酸磷酸激酶C、严密观察心电图演变D、观察含服硝酸甘油效果E、进行超声心动图检查正确答案:C 心电图对心肌梗死的诊断有重要价值。心肌梗死患者的心电图在发病数小时至3~4天内呈动态改变。

15、社区卫生服务是A、以健康为导向的基层卫生服务B、病人为导向的基层卫生服务C、以需求为导向的基层卫生服务D、以利用为导向的基层卫生服务E、以居民为导向的基层卫生服务正确答案:C

16、患者,男性,74岁,腹泻3天,神志不清1天,伴尿量增多。体温37.5℃,血压90/60mmHg(13.3/9kPa),皮肤黏膜干燥。尿糖(++++),尿酮体(±)。最可能的诊断为A、脑血管意外B、毒性痢疾C、感染中毒性脑病D、糖尿病酮症酸中毒E、高渗性非酮症糖尿病昏迷正确答案:E 老年病人,有脱水的诱因及脱水的表现,尿糖强阳性,酮体弱阳性,高渗性非酮症糖尿病昏迷的可能性最大。

17、关于社区的类型划分正确的是A、居民社区B、企业社区C、城市社区D、农村社区E、以上均正确正确答案:E 解析:在我国一般将社区分为城市社区与农村社区,也可以分为功能社区和生活社区。

18、下列关于基本公共卫生服务均等化的叙述错误的是A、每位中华人民共和国的公民,无论性别、年龄、种族、居住地、职业、收入,都能平等的获得基本公共卫生服务B、居民在需要获取相关的基本公共卫生服务时,机会是均等的C、每个人都必须得到完全相同、没有任何差异的基本公共卫生服务D、均等化并不是平均化E、通过实施国家基本公共卫生服务项目和重大公共卫生服务项目,使城乡居民逐步享有均等化的基本公共卫生服务正确答案:C 解析:基本公共卫生服务均等化是指每个中华人民共和国公民,无论其性别、年龄、种族、居住地、职业、收入水平,都能平等地获得基本公共卫生服务,但并不意味着每个人都必须得到完全相同、没有任何差异的基本公共卫生服务。目前我国提供的基本公共卫生服务很多内容是针对重点人群的,如老年人、孕产妇、0~6岁儿童、高血压等慢性病患者健康管理等,因此均等化并不是平均化。

19、下列药物中抗菌谱最广的是A、青霉素GB、红霉素C、诺氟沙星D、四环素E、庆大霉素正确答案:D 20、社区预防的策略的综合性表现为A、从以群体预防为主转向个体、家庭和群体相结合B、从生物预防扩大到心理行为和社会预防C、从独立的预防服务转向防治结合D、从被动预防转向个体和社会要对健康负责的主动预防E、以上均正确正确答案:E

21、卫生部、民政部、财政部、农业部、中医药局五部门联合下发的《关于巩固和发展新型农村合作医疗制度的意见》指出:2010年开始,全国新农合筹资水平提高到每人每年A、10元B、150元C、200元D、250元E、300元正确答案:B

22、氨基苷类最常见的不良反应是A、胃肠道反应B、肝脏毒性C、变态反应D、头痛头晕E、耳毒性正确答案:E 氨基苷类容易引起前庭功能失调、耳蜗神经损害等,所以孕妇注射本类药物可致新生儿听力受损,禁用。

23、某男,因伤寒服用氯霉素,一周后查血象发现有严重贫血和白细胞、血小板减少,这种现象发生的原因是A、氯霉素破坏了红细胞B、氯霉素缩短了红细胞的寿命C、氯霉素抑制了骨髓造血细胞线粒体整合酶的活性D、氯霉素抑制了免疫系统的功能E、氯霉素加强了吞噬细胞的功能正确答案:C

24、对肝功能不良患者,应用药物时,需着重考虑到患者的A、对药物的转运能力B、对药物的吸收能力C、对药物的分布的影响D、对药物的代谢能力E、对药物的排泄能力正确答案:D

25、药物的过敏反应与A、剂量大小有关B、药物毒性大小有关C、年龄有关D、体质有关E、以上均无关正确答案:D

26、布洛芬主要用于A、冠心病B、心源性哮喘C、风湿性关节炎D、人工冬眠E、镇静催眠正确答案:C

27、吲哚美辛属于A、抗痛风药B、镇痛药C、抗癫痫药D、解热镇痛抗炎药E、抗抑郁药正确答案:D

28、已证实可致畸的药物是A、苯妥英钠B、青霉素C、阿司匹林D、红霉素E、叶酸正确答案:A

29、社区卫生诊断过程中,现有资料收集内容一般不包括A、社区人口学资料B、社区环境资料C、社区卫生资源资料D、居民卫生知识水平E、社区卫生服务相关政策资料正确答案:D 解析:现有资料收集内容包括:社区人口学资料、社区环境资料、社区卫生资源资料以及社区卫生服务相关政策资料、社区死亡、疾病与相关调研资料以及各类文献资料等。居民卫生知识水平通过居民卫生调查获得。

30、社区卫生服务与医院服务在理念方面的不同之处表现在A、服务范围、服务技术、服务模式、服务手段、服务费用与特征B、服务对象、服务能力、服务模式、服务手段、服务费用与特征C、服务对象、服务技术、服务模式、服务手段、服务费用与特征D、服务对象、服务技术、服务方法、服务手段、服务费用与特征E、服务范围、服务能力、服务方法、服务手段、服务对象正确答案:C 解析:社区卫生服务机构以社区居民为服务对象,医院主要以就诊的个体患者为服务对象;社区卫生服务机构强调合理使用社区资源和适宜技术,而医院服务应用的技术更为宽泛;在服务模式上,社区卫生服务机构以上门服务、主动服务为主,医院主要以“坐堂行医”为主;社区卫生服务机构以预防为主,提供预防、医疗、保健、康复、健康教育、计划生育技术服务等为一体的综合服务,医院主要提供以治病为主的专科服务;社区卫生服务具有经济性,服务费用相对医院较便宜;社区卫生服务机构主要为社区健康人群提供预防、保健、健康教育等服务,是连续的健康服务,医院主要是“患病-治愈”或“住院-出院”的过程。

31、社区卫生服务综合性服务特点体现在A、服务对象包括病人和健康人,且不分性别、年龄、疾病类型B、务内容包括医疗、预防、康复保健、健康教育等C、服务范围包括个人、家庭、社区D、服务层面包括生理、心理、社会E、以上都是正确答案:E

32、社区卫生服务提供基本医疗服务的特点是A、运用全科医疗与适宜技术B、调综合性、连续性的健康管理C、关注病人的长期治疗效果和整体费用,还包括疾病预后D、服务方式则强调主动服务、上门服务以及责任制服务等E、以上均正确正确答案:E

33、每个社区卫生服务中心应保证每月至少举办健康知识讲座的次数是A、1次B、2次C、3次D、4次E、5次正确答案:A 解析:根据《国家基本公共卫生服务规范(2011年版)》,每个乡镇卫生院和社区卫生服务中心每月至少举办1次健康知识讲座。

34、社区主要构成要素是A、有集居的一群人和一定的地域B、有一定的生活服务设施C、居民之间发生种种社会关系D、为谋求规章制度具体落实,产生一种社会群体的机构E、以上要素都必须具备正确答案:E 解析:社区是指由若干社会群体(家庭、氏族)或社会组织(机关、团体)聚集在某一地域里形成的一个生活上相互关联的大集体。社区的构成须具备以下5个基本要素:①一定的人群;②一定的区域;③有一定的生活服务设施;④具有特定的文化背景和生活方式;⑤一定的生活制度和管理机构。

35、在家庭评估资料收集过程中,直接面谈法是指A、通过家访收集资料的方法B、过直接观察收集资料的方法C、通过电话收集资料的方法D、通过直接面谈收集资料的方法E、以上均是正确答案:A

36、按药物对胎儿危害的分类标准,仅在动物实验证实对胎仔有致畸或杀胚胎的作用,但在人类缺乏研究资料证实的为A、A类B、B类C、C类D、D类E、E类正确答案:C

37、小儿用药,下列哪项不正确A、体表面积比成人相对大,皮肤角化层薄,所以局部用药要防止吸收中毒B、给药量须经过计算并结合儿童具体情况C、要考虑小儿对药物的敏感性和耐受性D、应避免使用糖皮质激素类药物E、小儿上呼吸道感染或感冒时,可常规应用抗生素正确答案:E

38、“以病人为中心”的服务原则不包括A、建立以全科医生为核心的工作团队,发挥团队合作的功效B、重视疾病的同时,更重视病人的患病感受和价值观C、满足病人提供的各种要求D、尊重病人的权利E、注重提供临床预防服务正确答案:C

39、促使全科医学产生的背景不包括A、人口的迅速增长与老龄化B、人群疾病谱与死因谱的变化C、医疗费用的高涨D、健康观的变化E、环境污染的加剧正确答案:E 40、正常情况下,每小时的胎动为A、5次B、1~2次 BC、6~8次D、9~12次E、13~16次正确答案:B 胎动次数小于3次/小时为异常。

41、在健康问题的描述中,下列哪项不属于评估的内容A、问题诊断B、题鉴别诊断C、问题程度判断D、题预后判断E、疾病诊断正确答案:E

42、下述哪项不是以人为中心应诊中的任务A、确认并处理现患问题B、理慢性活动性问题C、提供机会性预防D、改善病人的就医和遵医行为E、分析病人的社会、文化和经济相关问题正确答案:E

43、哺乳期不适宜采用的避孕措施为A、体外射精B、服用避孕药C、使用避孕套D、放置官内节育器E、使用阴道隔膜正确答案:B 服用避孕药影响乳汁的质量和分泌量。

44、服用短效避孕药致无月经来潮连续几个月后,需停药检查A、2月B、3月C、4月D、5月E、6月正确答案:B

45、一对父母带5岁的儿子看病,该患儿高热2天,体温39℃。经检查为病毒性感冒,一般情况良好,无继发感染,患儿父母不接受医生的意见,坚持要求该患儿使用高级抗生素治疗,全科医生对此情况可能采取的最佳做法是A、坚持不开抗生素B、分的解释教育,说明不开抗生素的理由C、为了避免医患矛盾,顺从患儿父母的要求D、解释不开抗生素的道理,然后由患儿父母决定E、既然要求开好药,又能增加创收,何乐而不为正确答案:B

46、陈女士,早晨来社区卫生服务站诉说,起床后发现昨晚12时睡觉前,漏服短效避孕药,现又忘了补服时间,医生应告知其补服的时间为A、10小时内B、12小时内C、16小时内D、20小时内E、24小时内正确答案:B

47、我国老年人的划分标准是A、50岁以上B、55岁以上C、60岁以上D、65岁以上E、70岁以上正确答案:C WHO规定,65岁以上者称为老年人,欧美及工业发达国家均采用这一标准。亚太地区老年学会议提出:该地区60岁以上的为老年人。我国也以60岁以上作为老年人的划分标准。

48、WHO建议,按照6O岁以上老年人口系数达到多少百位比为老年型社会A、5 %B、15%C、20%D、25%E、10%正确答案:B

49、进行健康评估时关于身体评估的原则,下述正确的是A、应注意调节室内温度,一般要求室温在2~24℃B、不能一次进行较长时间,以避免老人疲乏C、体检时注意刺激应该适当,不要伤害老人D、应让老年人有充足的时间回忆过去发生的事件E、以上都对正确答案:E 50、下列哪项不是四分位数间距的特点A、适用条件同中位数B、映数值变量资料的离散趋势C、较极差稳定D、考虑了每个变量值的变异情况E、不受两端极大值、极小值的影响正确答案:D 解析:四分位数间距是上四分位数和下四分位数之差,可以将该指标理解为中间一半观察值的极差,是没有考虑到每个变量值的变异情况的。

51、某病住院日的频数分布呈单峰分布。平均住院日为10日。中位数为5日,可以看出,住院日的分布是A、正态分布B、正偏态分布C、负偏态分布D、左偏态分布E、对称分布正确答案:B 解析:对称分布时平均数与中住数应该一致,当两侧不对称时分布称为偏态分布,当单峰向左移时称为正偏态分布,当单峰向右移时称为负偏态分布。本题中中位数为5,即中间位置为5日,而平均住院日为10日,即单峰向左移,因为频数位于中间位置的数小于平均住院日,所以是正偏态分布。

52、使用时相对数要注意以下几点,其中哪项是不正确的A、比较时应做假设检验B、要分清离散程度和变异程度C、不要把构成比当率分析D、二者之间的可比性E、分母不宜过小正确答案:B 解析:使用相对数时要注意以下事项:计算相对数时分母不宜过小,即要有一定量的观察例数;注意平均率的计算方法;不要把构成比当率分析;注意资料之间的可比性;比较总体率的大小时应做假设检验。题目中离散程度和变异程度是计量资料的描述指标,和相对数没有关系。

53、下列关于患者享有平等医疗权利的表述错误的是A、公民享有平等的生命健康权B、应满足患者的合理需求C、医务人员应该平等对待患者D、患者的需求应得到完全满足E、患者享有的医疗保健权在实现时是受条件限制的正确答案:D平等医疗权是指病人有权享有同样良好的医疗保健服务和基本的、合理的医疗卫生资源,强调医务人员平等对待患者,医疗卫生资源分配体现社会公正,而不是所有的病人需求都得到满足。

54、审慎的含义是A、医务工作者应履行的职责和使命B、医务工作者应享有的权利和利益C、医务工作者对自己应尽义务的自我认知和评价D、医务工作者表现出行为前的周密思考和行为中的谨慎负责E、医务工作者对周围人、事以及自身的内心体验和感受正确答案:D 审慎是指医务人员在为病人服务的过程中,处事慎重、严谨、周密、准确、无误。

55、拟调查某地人群糖尿病的现患情况,可采用的研究方法是A、病例一对照研究B、队列研究C、实验研究D、现况调查E、理论研究正确答案:D 解析:现况调查用于描述疾病或健康的三间分布情况,因此选D。

56、欲进行吸烟与肺癌的病例一对照研究,以下不能作为对照的是A、胃病病人B、支气管炎病人C、意外损伤病人D、肠道传染病病人E、眼病病人正确答案:B 解析:在病例-对照研究中,应注意对照不应患有与所研究疾病有共同已知病因的疾病,以减少混杂偏倚。

57、在病例-对照研究中,调查对象应是A、病例为可疑患者,对照为未患某病者B、病例为确诊患者,对照也为患者C、病例为确诊患某病的人,而对照应是未患某病的人D、病例和对照都未被确定息某病E、病例为确诊患者,对照为可疑患者正确答案:D 解析:病例-对照研究的基本原理是以现在确诊的患有某特定疾病的病人作为病例,以未患有该病但具有可比性的个体作为对照,通过比较病例组和对照组中各个危险因素暴露比例来探讨危险因素与疾病的统计学关联,所以病例为确诊患者,对照为未患该病的人。

58、流行病学的实验性研究不包括A、临床试验B、病例-对照研究C、人群现场试验D、社区干预试验E、类实验正确答案:B 解析:病例-对照研究属于分析性研究。

59、下列哪个指标不能用于流行病学实验研究A、有效率B、治愈率C、抗体阳性率D、保护率E、患病率正确答案:E 解析:患病率是现况调查的常用指标。

60、下列有关偏倚的说法错误的是A、偏倚属于随机误差B、偏倚可产生于研究的设计、实施、分析等各个阶段C、常见的偏倚有选择偏倚、信息偏倚、混杂偏倚D、选择偏倚是由于研究对象与未选入的研究对象在某些特征上存在差异而引起的误差E、信息偏倚是在收集信息过程中由于测量暴露与结局的方法有缺陷造成的误差正确答案:A 解析:偏倚属于系统误差,可以说偏倚是研究过程中的失误,它会对研究结果产生很大影响,甚至可能得到与事实相反的结论。

61、流行病学研究的观察法与实验法的根本区别在于A、设立对照组B、不设立对照组C、是否人为控制研究的条件D、盲法E、统计学检验正确答案:C 解析:流行病学研究的实验法与观察法最大的区别是有人为的干预措施施加于研究对象。62、以下哪项不属于定量资料A、体块指数(体重/身高2)B、白蛋白与球蛋白比值C、细胞突变率(%)D、血压值E、中学生中吸烟人数正确答案:C 解析:定量资料是连续的,通常有度量衡单位。所以,细胞突变率(%)不是定量资料。

63、搞好统计工作,达到预期目标,最重要的是A、原始资料要多B、原始资料要正确C、整理资料要详细D、分析资料要先进E、统计计算精度要高正确答案:B 解析:统计分析一般包括统计设计、收集资料、整理资料和分析资料四个步骤,无论哪一步,均需要追求真实性,所以原始资料要正确最为重要。

64、变异系数越大说明A、标准差越大B、平均数越大C、标准差、平均数都大D、平均数越小E、以均数为准变异程度大正确答案:E 解析:变异系数是反映离散趋势的,变异系数越大反应以均数为准的变异程度大。

65、某课题组获得某校新入学大学男生腹部皮下脂肪厚度(cm)和身高(cm)资料,现比较这两个变量的离散趋势,最佳的指标是A、变异系数B、全距C、方差或标准差D、偏度系数E、四分位数间距正确答案:A 解析:本题目是比较腹部皮下脂肪厚度(cm)和身高(cm)资料的离散趋势,因为是比较不同单位指标的离散趋势,所以应该用变异系数。

66、测量某项正态分布的医学指标n次,得其均值和标准差S,求得区间(-t0.05,vS/,+t0.05,vS/)所代表的含义为A、99%医学参考值范围B、95%医学参考值范围C、99%总体均数置信区间D、95%总体均数置信区间E、以上都不是正确答案:D 解析:题目中求得的区间(-t0.05,vS/,+t0.05,vS/)为均数±95%界值×标准误,所以是95%总体均数置信区间。67、下列是有关参考值范围的说法,其中正确的是A、参考值范围应根据正常人范围的95 来制定B、如果随机测量某人的某项指标,其值在正常人范围的95%之内,那么应认为此人的此项指标正常C、如果某项指标超出了参考值范围,那么其应为不正常D、求正态资料的参考值范围,精确度越高越好E、所谓的正常和健康都是相对的,在正常人或健康人身上都存在着某种程度的病理状态正确答案:E 68、在假设检验中,P值和α的关系为A、P值越大,α值就越大B、P值越大,α值就越小C、P值和α值均可由研究者事先设定D、P值和α值都不可以由研究者事先设定E、P值的大小与α值的大小无关正确答案:E 69、在样本均数与总体均数比较时,若n=25,t=1.96,则A、P>0.05B、P= 0.05C、P<0.05D、P<0.01E、P值无法确定正确答案:A 70、计算某地儿童肺炎的发病率,现求得男、女童肺炎发病率分别为21.2%和19.1%,可认为A、男童的肺炎发病率高于女童B、应进行标准化后再做比较C、资料不具可比性,不能直接作比D、应进行假设检验后再下结论E、以上都不对正确答案:D 解析:计算某地儿童肺炎求得的发病率只是一个观察值,男女间的发病率是否有差异,不能只看到观察值就下结论,要考虑抽样误差的问题,所以需要进行假设检验才能下结论。71、四个百分率作比较,有1个理论数小于

5、大于1,其他都大于5,则A、只能作校正χ2检验B、不能作χ2检验C、作χ2检验不必校正D、必须先作合理的合并E、要用精确概率法正确答案:C 解析:行×列表χ2检验的应用条件是T<5的格子数不超过1/5和没有任何一个格的T<1。题干中符合行×列表χ2检验的应用条件,所以选C。

72、某医生对一批计量、计数资料实验数据进行假设检验,结果判定如下:进行配对χ2检验时,χ2=2.42则A、P<0.05B、P=0.05C、P>0.05D、P< 0.01E、P=0.01正确答案:C 解析:因配对χ2检验的界值为3.84,而统计量χ2=2.42,所以P>0.05。

73、关于χ2检验,下列叙述正确的是A、χ2检验方法适用于任何类型资料B、χ2检验要求资料符合正态分布C、χ2检验用于计数资料的分析D、χ2检验只能检验两个率有无显著性差异E、以上都不对正确答案:C 解析:χ2检验是用于推断两个或两个以上总体率或构成比之间有无差别最常用的方法,适用于对计数资料的分析。

74、用图表示某地区近30年三种疾病的发病率,在各的动态发展速度情况,宜绘制A、普通线图B、直方图C、百分条图D、半对数线图E、直条图正确答案:A 75、下列哪项丕星健康教育规划实施中的监测和质量控制的内容A、正确评估计划执行者的技能B、建立专家小组,保证计划执行的质量C、建立完整的资料收集和保存体系D、经费使用的审计E、强化健康促进的基本知识和技能正确答案:E 解析:E属于项目技术培训部分。

76、下列行为中,属于危害健康行为的是A、预警行为B、求医行为C、遵医行为D、避开环境危害E、C型行为正确答案:E 解析:A、B、C、D均属于促进健康行为。

77、关于流行病学,下列哪种说法是正确的A、流行病学从整体水平认识疾病B、流行病学从细胞水平认识疾病C、流行病学从群体水平认识疾病D、流行病学从个体水平认识疾病E、流行病学从分子水平认识疾病正确答案:C 解析:流行病学主要从群体水平认识疾病,而不是从个体水平。78、4岁男孩,咳嗽5个月,凌晨及活动后加剧,服用多种抗生素无效,服用博利康尼(特布他林)后有缓解。查体:无发热,面及颈部散在湿疹。两肺呼吸音粗,该患儿最可能的诊断是A、毛细支气管炎B、支气管异物C、咳嗽变异性哮喘D、支气管淋巴结核E、儿童哮喘正确答案:C 根据该患儿的临床表现清晨发作性咳嗽,运动后加重,抗生素治疗无效,支气管扩张剂可缓解,两肺呼吸音粗,考虑为支气管哮喘中的咳嗽变异性哮喘,该病诊断要点是支气管扩张剂’可使咳嗽发作缓解,故选C。

79、不符合轮状病毒肠炎的特点是A、夏季多见B、多见于6~24个月C、大便呈蛋花汤样D、出现脱水E、常伴有发热正确答案:A 轮状病毒肠炎又称秋季腹泻,是婴幼儿秋冬季节腹泻的最常见原因;多发生在6~24个月婴幼儿;大便水分多,黄色水样或蛋花样便;伴发热和脱水,故选A。

80、重度脱水小儿,首批液体选用A、4:3:2液,80ml/kgB、2:1液,20ml/kgC、2:1液,30ml/kgD、5%碳酸氢钠,5ml/kgE、3:2:1液,100ml/kg正确答案:C 对重度脱水伴循环衰竭者,必须迅速扩容以改善循环状态,20ml/kg等渗含钠液快速输入,4:3:2液及3:2:1液均为2/3张液体,2:1液兼有扩充血容量及纠正酸中毒的作用。81、急性肾小球肾炎的常见致病茵是A、溶血性链球菌B、萄球菌C、肺炎链球菌D、柯萨奇病毒E、草绿色链球菌正确答案:A 急性肾小球肾炎病因多种多样,但绝大多数的病例属A组β溶血性链球茵急性感染后引起的免疫复合性肾小球肾炎。故选A。

82、发现患者存在意识障碍的重要标志是A、反应迟钝B、情感淡漠C、定向力障碍D、记忆力障碍E、注意力集中困难正确答案:C 定向力包括时间、地点、人物和对其自身状况的认识。定向障碍一般在脑器质性疾病中较为多见,往往是意识障碍的重要标志。

83、癫痫发作不包括A、强直一阵挛发作B、直性发作C、失神发作D、精神症状发作E、不典型发作正确答案:D 精神症状发作属于局灶性发作。

84、川崎病时皮肤黏膜表现中最具有特征性的改变是A、皮疹B、指(趾)端变化C、眼球结膜充血D、杨梅舌E、唇红干燥、皲裂正确答案:B 川崎病的主要表现是发热、球结合膜充血、唇、口腔黏膜充血、手足硬性水肿和掌趾红宽,多形性皮疹,颈部淋巴结肿大等,其中指(趾)端的变化最具有特征性,恢复期呈膜状脱皮。85、阿尔茨海默病患者经常走错房间,外出不认得回家的路,主要是因为A、行为紊乱B、记忆障碍C、意识清晰度下降D、意志减退E、错觉正确答案:B 阿尔茨海默病的核心症状是记忆障碍,包括短时记忆和视空间记忆障碍。

86、诊断躯体疾病所致精神障碍可依据以下几点A、存在大脑疾病、损害或功能紊乱,或系统性躯体疾病的依据B、精神症状的出现与脑部或躯体疾病的进展有时间关系(几周或几个月)C、精神症状随着脑部或躯体疾病的改善而有所缓解D、无证据提示精神症状的产生有其他病因E、精神症状常有昼轻夜重的节律变化正确答案:E 躯体疾病所致精神障碍的诊断要点包括:存在大脑疾病、损害或功能紊乱,或系统性躯体疾病的依据;精神症状的出现与脑部或躯体疾病的进展有时间关系(几周或几个月);精神症状随着脑部或躯体疾病的改善而有所缓解;无证据提示精神症状的产生有其他病因。

87、认知疗法的基本观点是A、情绪决定认知和行为B、行为决定情绪和认知C、情绪、行为与认知互为因果D、认知决定情绪和行为E、行为与认知关系不大正确答案:D 认知治疗认为人的情绪困扰、行为问题或各种心理障碍均与人的认知和认知过程有关。88、发生谵妄时最常出现的幻觉是A、听幻觉B、味幻觉C、视幻觉D、本体幻觉E、触幻觉正确答案:C 谵妄是一种病因学上非特异性的脑器质性综合征,其特点为意识障碍,意识清晰度水平降低,同时产生大量的错觉和幻觉,以幻视为多见。

89、风疹的出疹特点是A、发热3~4小时开始,自耳后、发际、颌面、颈出疹,渐及躯干及四肢,最后见于手足心B、发热1~2天开始,最早见面颊部,迅速扩展至躯干和四肢,24小时内布满全身,但手足心通常无皮疹C、高热退出疹D、发热数小时到1天出疹,分批出现,向心性分布,以躯干、头、腰部多见E、玫瑰红色融合性斑丘疹,先分布于鼻翼两侧,1~2天蔓延至全身正确答案:B A是麻疹的出疹特点,B是风疹的出疹特点,C是幼儿急疹的出疹特点,D是水痘的出疹特点,E是传染性红斑的出疹特点。

90、风疹的隔离期通常是A、出疹后5天,亦可以不隔离B、疹后7天C、出疹后10天D、并发肺炎者,出疹后10天E、并发肺炎者,出疹后2周正确答案:A 一般皮疹出现5天后即无传染性,除病房内发生病例外,一般可不隔离。

91、幼儿急疹的治疗方法是A、发热期间注意退热治疗,皮疹无须特殊治疗B、隔离,早治疗C、注射特异性高效价免疫球蛋白D、格隔离,加强抗病毒治疗E、注射预防疫苗以预防其发生正确答案:A 本病为自限性疾病,无须特殊治疗,可采用对症治疗处理。92、35岁男性患者,近半年来觉得有人跟踪自己,有人在自己的屋里放了窃听器,并且听见有人议论如何对付自己,曾写信到公安局要求保护。此病人最可能的诊断是A、青春型精神分裂症B、偏执型精神分裂症C、单纯型精神分裂症D、偏执性精神病E、紧张型精神分裂症正确答案:B 偏执型精神分裂症是精神分裂症最常见的一个类型,其临床表现以相对稳定的妄想为主,往往伴有幻觉,情感、意志、言语、行为障碍不突出,起病多在30岁以后。

93、患者,女,26岁,近几年来无诱因出现情绪低落,晨重晚轻,兴趣减退,自觉精力减退,易疲劳,少语,失眠以早醒为主,多次想自杀。最可能的诊断为A、神经衰弱B、抑郁性神经症C、反应性抑郁症D、抑郁症E、失眠症正确答案:B 抑郁症的主要症状为情绪低落、思维迟缓、兴趣减退和躯体症状为主,且症状有晨重晚轻的特点,睡眠障碍以早醒为主。

94、某抑郁症患者,27岁,复发情绪抑郁、悲观厌世,认为自己是历史罪人,只有死路一条,反复自杀未遂。首选的治疗方法为A、心理治疗B、电抽搐治疗C、丙咪嗪D、氟西汀E、舒必利正确答案:B 有严重消极自杀企图的患者及抗抑郁药治疗无效的患者可采用电抽搐治疗。

95、在缺乏相应客观因素的情况下,患者表现为顾虑重重、紧张恐惧,以致搓手顿足,似有大祸临头,惶惶不可终El,伴有心悸、出汗、手抖、尿频等自主神经功能紊乱症状,这属于A、情感不稳定B、焦虑C、易激惹D、情感幼稚E、情感低落正确答案:B 96、对流行性腮腺炎的健康指导不正确的是A、隔离病人至腮腺肿胀完全消退为止B、行性腮腺炎是一种化脓性炎症C、被动免疫可给予腮腺炎免疫γ-球蛋白D、主动免疫可接种单价腮腺炎减毒活疫苗和MMRE、人是唯一的宿主正确答案:B 流行性腮腺炎是一种非化脓性炎症。

97、引起猩红热的病原体是A、A组α链球菌B、B组α链球菌C、A组β链球菌D、B组β链球菌E、金黄色葡萄球菌正确答案:C 引起猩红热的病原体是A组β链球茵。

98、关于社区康复医疗的描述,下列哪项不妥A、是整个康复过程的重要组成部分B、具有转诊的功能C、在社区层次上采取综合性的康复措施D、有利于营造助残的良好社会风气E、以物理治疗师为最基本的康复实施形式正确答案:E 99、不属于ADL评定的内容的是A、运动B、作业治疗C、家务劳动D、社会交往E、生活自理正确答案:B 100、社区康复医疗业务工作由谁来指导A、全科医师B、康复治疗师C、上级医疗机构康复专业人员D、社区护士E、社区卫生服务站正确答案:C 社区康复医疗工作是以全科医师为骨干,由上级医疗机构康复专业人员来指导。

101、目前导致我国人群死亡的前十位疾病的病因和疾病危险因素中,比例最高的是A、行为生活方式B、生物学因素C、环境因素D、卫生保健因素E、以上都不是正确答案:A 在疾病病因构成中,行为生活方式占75%,为最主要因素。

102、社区康复训练与服务不包括A、对康复对象进行初次功能评估、选择适宜的训练项目B、指导康复对象进行康复训练、定期评估康复效果C、选用与制作训练器材,用品用具的信息、供应、维修服务D、心理支持服务、知识普及服务、转介服务E、进行社区康复检查评估正确答案:E 103、真性糖尿病性白内障的临床特征不包括A、年龄较轻,多发生在3O岁以下B、双眼发病、进展迅速C、晶状体呈雪花样混浊或点状混浊D、伴有屈光变化E、术前要控制血糖正确答案:D 104、关于老年性白内障的描述,下列哪项描述不正确A、初发期白内障,可选用药物治疗B、熟期白内障,可选用手术治疗C、白内障患者,突然出现眼胀、头痛,应考虑继发性青光眼D、术前应进行眼部B超和CT检查E、术前应将血压、血糖控制基本正常正确答案:D 105、手外伤后,应尽可能使手处于功能位。手的功能位是A、腕背伸约10°~15°,掌指关节及指间关节呈半屈曲状态B、腕背伸约20°~25°,掌指关节及指间关节呈半屈曲状态C、腕背伸约20°~25°,掌指关节及指间关节微屈D、腕背伸约20°~25°,掌指关节及指间关节呈半屈曲状态E、腕背伸约10°~15°,掌指关节及指间关节微屈正确答案:C 腕背伸约20°~25°,掌指关节及指间关节微屈是手的功能位。

106、视网膜浅层出血多呈线状、条状及火焰状,色较鲜红,位于A、神经纤维层B、丛状层C、外核层D、外丛状层和内核层之间E、内核层正确答案:A 107、以下哪点体现了人类行为的社会性A、儿子仿效父亲抽烟B、看见老虎赶快奔逃C、性行为D、寻找食物充饥E、喜甜食正确答案:A 108、眼的屈光系统包括A、角膜、晶状体、玻璃体B、膜、晶状体、视网膜C、角膜、房水、晶状体、玻璃体、视网膜D、泪膜、角膜、晶状体、玻璃体E、角膜、房水、晶状体、玻璃体正确答案:E 109、白内障摘除后的无晶状体眼的屈光状态是A、高度近视B、高度远视C、高度散光D、复合近视散光E、复合远视散光正确答案:B

110、一位6岁儿童,家长发现其双眼不能同时注视一个目标,右眼向颞侧偏斜,检查发现:双眼球运动无障碍,双眼向各个方向转动时,偏斜程度保持不变,双眼视力正常。你的诊断是A、麻痹性内斜视B、性外斜视C、共同性外斜视D、共同性内斜视E、麻痹性外斜视正确答案:C 111、功能独立性评定(FIM)评定的内容不包括A、自理活动B、活动与运动C、括约肌控制D、交流E、肢体畸形正确答案:E 112、Lovett徒手肌力检查最适宜A、脑卒中患者B、小儿脑瘫C、小儿麻痹症、周围神经损伤患者D、帕金森病患者E、脑外伤后遗症患者正确答案:C 113、慢性咽炎的病理分型错误的是A、慢性单纯性咽炎B、慢性肥厚性咽炎C、萎缩性咽炎D、干燥性咽炎E、以上都是正确答案:E 114、关于CT值的描述,哪项是错误的A、CT值说明组织结构的密度高低B、CT值没有单位C、CT值具有量的概念D、CT值由吸收系数换算而来E、对比平扫和增强扫描的CT值,可了解强化程度正确答案:B CT值是CT图像测量中用于表示组织密度的统一计量单位,其单位名称为亨氏单位(Hounsfield unit,HU)。

115、不可用于治疗感冒的中成药有A、银翘解毒丸B、藿香正气丸C、小柴胡颗粒D、金匮肾气丸E、防风通圣丸正确答案:D 116、慢性化脓性中耳炎描述错误的是A、分为单纯型、骨疡型及胆脂瘤型B、耳骨质的吸收破坏C、有耳道流脓病史D、可并发眩晕E、听力渐进性下降正确答案:B 慢性化脓性中耳炎是中耳黏膜、鼓膜或深达骨质的慢性化脓性炎症,常与慢性乳突炎合并存在,本病极为常见,临床上以耳内长期或间歇流脓、鼓膜穿孔及听力下降为特点,可引起严重的颅内、外并发症而危及生命。其中单纯型慢性化脓性中耳炎不伴有中耳骨质的破坏,骨疡型及胆脂瘤型伴有中耳骨质的吸收破坏。

117、耳源性脑脓肿颅内高压症状包括A、头痛B、吐C、意识障碍D、脉搏加快与体温一致E、以上均是正确答案:E 耳源性脑脓肿病人出现颅内高压症状,主要表现为:头痛始于患侧,后可遍及整个头部,以前额或后枕部最为显著,头痛多为持续性,常于夜间加剧而惨叫不止;呕吐为喷射状,与饮食无关;意识障碍,如表情淡漠,嗜睡,乃至昏迷;脉搏迟缓,与体温不一致;可出现视乳头水肿;其他:如打哈欠,频繁的无意识动作,性格与行为改变等。

118、鼻出血的治疗原则A、镇静B、控制急性出血C、病因治疗D、对症治疗E、以上都是正确答案:E 119、急性会厌炎的处理错误的是A、全身足量抗生素B、全身足量糖皮质激素C、局部足量抗生素D、必要时行气管切开E、严密观察正确答案:C 急性会厌炎是一种危及生命的严重感染,一旦发现应全身足量抗生素及糖皮质激素,密切患者呼吸情况,若呼吸困难无明显改善,应考虑气管切开。

120、慢性鼻窦炎的治疗方案包括A、局部鼻用糖皮质激素B、鼻腔冲洗C、鼻窦负压置换D、鼻腔鼻窦手术E、以上都是正确答案:E 121、窝沟龋的早期表现A、明显龋洞B、探诊有酸感C、损害部位呈白垩色D、损害部位透出墨浸色E、损害位于牙颈部正确答案:C 早期呈现白垩色,进一步发展才是墨浸样改变

122、患者,女,39岁,每次月经前小腹刺痛,伴乳房胀痛,经色紫黑,挟血块,舌紫脉弦。其病机是A、肾气亏损B、气滞血瘀C、湿热蕴结D、寒凝血瘀E、气血虚弱正确答案:B 123、以下哪项是外阴阴道假丝酵母菌病的特点A、黄绿色稀薄泡沫状白带,检查阴道黏膜有散在出血斑点B、分泌物涂片可见线索细胞C、阴道pH>4.5D、氨臭味试验阳性E、白色稠厚豆渣样白带,检查小阴唇内侧及阴道黏膜附着白色膜状物正确答案:E 124、痹病的共同特征是A、肢体沉重B、肢体关节灼热C、肢体关节疼痛,曲伸不利D、肌肤麻木E、以上都不是正确答案:C 125、冠周炎多发于哪颗牙齿A、18,28B、38,48C、44,54D、14,24E、16,26正确答案:B 双侧下颌智齿容易食物嵌塞,易发冠周炎。

126、判断有无牙周炎的重要指征是A、龈袋超过3mmB、附着丧失C、龈红肿D、龈出血E、龈乳头增生正确答案:B 有没有附着丧失是判断有无牙周炎的主要指征。

127、属于口腔癌一级预防的是A、早发现B、早诊断C、早治疗D、病因预防E、防止复发正确答案:D 一级预防即病因性预防,是根本性的预防措施。

128、不属于癌前病变的是A、腺纤维腺瘤B、乳腺纤维囊性病C、结肠腺瘤性息肉病D、慢性萎缩性胃炎E、黏膜白斑正确答案:A 从正常组织到发生癌变的中间阶段称为癌前病变。常见的癌前病变有:黏膜白斑、宫颈糜烂、乳腺纤维囊性病、慢性萎缩性胃炎、胃肠多发性息肉、慢性溃疡、卵巢囊肿、慢性乙肝、肝硬化、色痣等。

129、下面哪一项不是透视的优点A、可转动患者体位,进行多方向观察B、价格低廉C、可实时了解气管的动态变化D、操作简单E、图像对比度和清晰度较好正确答案:E 透视图像的对比度和清晰度较差;A~D均正确。

130、立位片上,腹腔游离气体的典型影像学表现为A、不规则形B、球形C、半球形,有“液一气平面”D、与胸腔间距大于20mmE、新月形正确答案:E 膈下新月状透亮区为立位片腹腔游离气体的典型表现。131、诊断听神经瘤的最佳影像学方法是A、X线平片B、桥小脑池空气造影C、CTD、MRIE、超声检查正确答案:E 诊断听神经瘤首选MRI。

132、女性,50岁,因右上腹痛2个月入院。腹平片示右上腹一类圆形致密影,透视下随膈肌运动上下移动。首先考虑为A、胆囊结石B、淋巴结钙化C、右肾结石D、右输尿管结石E、以上都不是正确答案:A 133、痢疾患者粪便常规检查应取以下哪项标本A、黏液B、血液C、脓血黏液处的粪便D、脓液E、无脓液的粪便正确答案:C 对于痢疾患者,粪便常规检查应用竹签自粪便多处取材,特别是有脓、血、黏液等异常部分。134、尿常规分析标本必须在多长时间内完成检验A、2hB、5hC、10hD、16hE、24h正确答案:A 尿液标本收集后应及时送检并在2小时内完成检查,以免发生细胞溶解、蛋白变性、细菌繁殖等现象影响结果。

135、心电图临床应用最有价值的是A、心肌梗死B、心律失常C、电解质紊乱D、心肌缺血E、心室肥大正确答案:B 136、对血液标本发生溶血影响最小的因素是A、钾离子B、葡萄糖C、LDHD、AsTE、ACP正确答案:B 待测物在红细胞内的浓度高于血浆时,溶血可使测定结果偏高,有些物质如LDH、ACP、AST、K+等在红细胞内的浓度比血浆高22~160倍,轻微溶血都对结果影响很大;血红蛋白可干扰胆固醇的酶法测定,抑制胆红素的重氮反应等;溶血也干扰某些光谱分析。因此应尽量避免溶血。

137、有关尿亚硝酸盐试验,以下说法正确的是A、尿亚硝酸盐试验可以协助诊断尿路感染B、尿亚硝酸盐试验阴性可以排除菌尿的可能C、尿亚硝酸盐试验阳性可以完全肯定泌尿系统感染D、尿亚硝酸盐试验检出率100%E、尿亚硝酸盐试验检出率与尿液标本是否在膀胱停留4小时以上无关正确答案:A 当尿中某些致病茵主要是大肠埃希茵增殖时,可将硝酸盐还原为亚硝酸盐,从而使尿液亚硝酸盐检查试带呈显色反应,因此亚硝酸盐阳性有助于尿路感染的诊断。

138、脓血便多见于A、急性细菌性痢疾B、急性胃肠炎C、上消化道出血D、霍乱E、肛门出血正确答案:A 粪便外观出现脓性及脓血说明下段肠道有病变。急性细菌性痢疾时以黏液及脓为主,脓中带血。

139、可使血糖水平下降的激素是A、肾上腺激素B、胰高血糖素C、生长素D、胰岛素E、甲状腺素正确答案:D 胰岛素由胰岛的B细胞所产生,是主要的降糖激素,其余激素升血糖效果明显。

140、以下哪项酶类不能反映肝细胞损伤A、α-淀粉酶B、转氨酶C、异柠檬酸脱氢酶D、鸟氨酸氨基甲酰酶E、谷氨酸脱氢酶正确答案:A α-淀粉酶是反映急性胰腺炎的指标。

141、HBsAg(-),抗H&(+),HBeAg(-),抗HBe(-),抗HBc(-),表明A、急性肝炎初期B、乙肝恢复并产生免疫力C、阳性活动性肝炎D、既往乙肝病毒感染E、慢性迁延性肝炎正确答案:B 抗HBs为保护性抗体,它的出现表明病毒基本清除;HbsAg疫苗注射后抗HBs出现是免疫成功的标志。142、患者女性,35岁。1个月前因外伤手术输血800ml,近1周出现乏力,食欲不振,尿色加深。实验室检查:ALT 200U/L,抗HCV(+),HCVPCR(+),抗HBe(+),诊断为A、急性丙型肝炎,既往曾感染乙型肝炎病毒B、慢性丙型肝炎C、乙型、丙型肝炎病毒合并感染D、急性乙型、丙型肝炎E、急性乙型肝炎正确答案:A 抗HBc(+)说明既往感染过乙肝,HCVPCR(+),且有输血史,说明感染HCV。

143、下列各项外周血检查结果中,对于诊断急性白血病最有意义的是A、白细胞计数26×109/LB、分叶核粒细胞>85%C、白细胞计数2×109/LD、原始细胞27%E、血小板计数20×109/L正确答案:D 急性白血病时,大多数患者外周血中白细胞数增多,部分患者白细胞数正常或减少;红细胞血小板进行性减少;血象中出现原始及幼稚细胞最有诊断意义。

144、临床进行ABO血型鉴定最常采用的方法是A、间接凝集反应B、试管凝集法C、玻片凝集法D、ELISA法E、间接凝集抑制反应正确答案:C 玻片凝集法常用于ABO血型鉴定。

145、白斑的临床类型分为A、寻常型、增殖型、叶型、红斑型B、滑型、结节型C、斑块型、颗粒型、皱纸型、疣状、溃疡状D、皱纸型、海绵状E、萎缩型、肥厚型正确答案:C 146、治疗白斑的首要措施是A、手术切除B、0.2%维A酸溶液局部涂布C、维生素A口服D、去除刺激因素E、增强机体免疫能力正确答案:D 治疗白斑的首要措施是去除刺激因素,包括烟酒、刺激性食物、残根残冠、不良修复体。147、关于慢性盆腔炎的治疗,下列哪项不恰当A、物理治疗B、药治疗C、透明质酸酶肌注D、广谱抗生素静滴2~3周E、药物治疗无效者行手术治疗正确答案:D 长期或反复多种抗生素联合治疗慢性盆腔炎往往并无显著疗效。

148、输卵管妊娠的子宫内膜可见A/S反应,是由哪些激素过度刺激形成的A、孕酮、β-HCGB、PL、β-HCGC、雌激素,孕激素D、β-HCG,雌激素E、FSH,HPL正确答案:C 本考题考核异位妊娠的病理生理变化,β-HCG是妊娠期分泌的特异性激素,但是子宫内膜A/S反应是由雌激素和孕激素的增加引起的。

149、下列哪项不是急性输卵管妊娠破裂的表现A、高热B、动性浊音(+)C、下腹压痛、反跳痛D、宫颈举痛,子宫漂浮感E、后穹隆穿刺吸出不凝血液正确答案:A 150、青春期功能失调性子宫出血的治疗原则是A、减少月经量B、进子宫发育C、止血、调整月经周期、促排卵D、抗生素治疗E、防止子宫内膜病变正确答案:C 151、24岁女性,一周前因停经41天,妊娠试验阳性,行吸宫流产术,今晨突然晕倒在地,体温37.5℃,血压75/52.5mmHg,脉搏100次/分,下腹压痛及反跳痛明显,外阴少量流血,宫颈举痛明显,宫口闭,子宫稍大,稍软,右侧似有一包块边缘不清,压痛。查WBC 10×109/L,N 0.70。最准确的诊断是A、人工流产不全B、颈粘连C、急性阑尾炎D、流产后右附件炎E、右输卵管妊娠破裂正确答案:E 152、患者,35岁,下腹部肿物3年,逐渐增大,自右下腹部开始如拳头大小,现已如妊娠足月大小,但仍能做轻微的家务劳动。最可能的肿瘤为A、卵巢畸胎瘤B、巢浆液性囊腺瘤C、卵巢黏液性囊腺瘤D、子宫内膜样肿瘤E、卵泡膜细胞瘤正确答案:C 153、患者,女,28岁,G1P0A1,停经38天,阴道出血8天,伴下腹隐痛,查:官颈无举痛,官体增大,质中,附件无明显压痛,HCG(+),要求做人工流产。人工流产吸出物见以下哪一项可排除宫外孕A、蜕膜组织B、绒毛C、A/S反应D、增殖期子宫内膜E、分泌期子宫内膜正确答案:B 154、子宫内膜癌首选的治疗办法是A、手术治疗B、放射治疗C、化疗D、内分泌治疗E、免疫治疗正确答案:A 155、以下关于原发性痛经错误的是A、可用性激素抑制排卵B、无排卵性月经多无痛经C、可试用促排卵药治疗D、可用前列腺素抑制剂治疗E、可用解痉药治疗正确答案:D 156、以下关于过期流产的处理错误的是A、刮宫前作好备血、输液准备B、刮宫前先用雌激素以提高子宫肌肉对催产素的敏感性C、刮宫前先作凝血功能检查,防止术时因血凝障碍而大出血D、力争做到一次刮净,以防因不全流产而出血E、过期流产胎盘机化,刮宫术中谨防穿孔正确答案:D 157、女性,26岁,主诉阴道分泌物恶臭1周。患者有几个性伴侣,不使用避孕套。体检没有发现外阴溃疡、病变或皮疹。内镜检查见阴道穹隆壁上有灰色粘连的恶臭分泌物。显微镜下见多个白细胞和线索细胞。对于此种感染,以下哪种治疗最有效A、甲硝唑(灭滴灵)B、头孢曲松(罗氏芬)C、西环素(如:福多力,伟霸霉素)D、复方磺胺甲唑(复方新诺明)E、酮康唑(里素劳片)正确答案:A 该病人患细茵性阴道病,灰白稀薄的分泌物是此病特征。推荐治疗是甲硝唑500mg每日2次给药,连用7~10天;也可用克林霉素300mg每日2次给药,连用7天;或者用2%克林霉素乳膏,睡前阴道内给药5g,连用7天。不推荐对性伴侣进行常规治疗。

158、以下检查除外哪项对功血病人的诊断均有帮助A、基础体温测定B、宫镜检查C、初潮年龄D、阴道脱落细胞检查E、子宫内镜检查正确答案:C 159、女,28岁,月经频发,经血量正常,婚后4年未孕,前来就诊。妇科检查,子宫正常大小,双附件无异常,基础体温呈双相型,最可能的诊断是A、无排卵性功血B、子宫内膜炎C、子宫内膜脱落不全D、黄体功能不全E、子宫肌瘤正确答案:D 160、由真菌引起的皮肤病为A、疣B、带状疱疹C、脚癣D、白斑E、湿疹正确答案:C 脚癣一般由真茵感染引起,故选C。

161、孕妇口服“反应停”致畸作用强(短肢海豹儿),但该药在皮肤科可用于治疗A、严重囊性痤疮B、严重银屑病C、光敏性皮肤病D、瘙痒性皮肤病E、脂溢性皮炎正确答案:C 反应停是一种曾在现代医学史上造成巨大灾难的药物,已经被停止使用。但其对光敏性皮肤病有治疗作用,故选C。

162、患者,女,26岁,皮肤瘙痒难忍,起病急、发展快,局部出现大小不等的红色风团,呈椭圆形、圆形或不规则形状,用钝器以适当压力划过,可出现皮肤划痕试验阳性,数小时内水肿减轻,应考虑为以下哪一种疾病A、急性荨麻疹B、急性湿疹C、急性接触性皮炎D、带状疱疹E、银屑病正确答案:A 皮肤划痕试验阳性为荨麻疹特征性表现,加之起病急、瘙瘁难忍等表现,故选A。

163、患者女,21岁,面部皮疹两月余,无自觉症状,皮疹为米粒大到绿豆大扁平隆起的丘疹,表面光滑,质硬,浅褐色,圆形、椭圆形或多角形,有的呈串珠状排列,最可能的诊断是A、寻常疣B、扁平疣C、丝状疣D、湿疹E、老年疣正确答案:B 扁平疣是一种病毒性皮肤病,它的病原体和寻常疣一样,是由乳头状瘤病毒HPV3和HPV5感染引起的皮肤赘生物。表现为或分散分布、质地柔软、顶部光滑、粟粒至绿豆大、淡褐色的高出皮肤表面的扁平状丘疹。

164、下列各项有助于新生儿ABO溶血症的诊断,但哪一项除外A、母婴血型不合B、儿血清游离抗体阳性C、抗人体球蛋白试验阴性或弱阳性D、50%的患儿黄疸在生后24小时内出现E、抗体释放试验阳性正确答案:D 新生儿ABO溶血症多在24小时内出现。

165、患者,女,60岁,臀部、双下肢皮肤起脓疱2周,自觉疼痛,损害渐扩大向深部发展呈溃疡而就诊。有糖尿病史5年。体检:一般情况良好,臀部、双下肢可见散在性脓疱,部分破淡结黑色厚痂,如蛎壳状,痂脱后形成边缘陡峭的溃疡。应考虑为A、深脓疱疮B、毛囊炎C、痱子D、疖疮E、二期梅毒正确答案:A 患者老年,有糖尿病,根据典型皮损,符合深脓疱疮诊断要点,故选A。

166、男童,2岁,近l周于口周、鼻孔处出现红色小丘疹,渐成脓疱。疱破后结黄色厚痂,波及耳郭处伴瘙痒。体检:下颌可触及肿大淋巴结,患儿体温正常,血常规示白细胞计数正常。应考虑为A、湿疹伴感染B、寻常型脓疱疮C、单纯疱疹D、疖肿E、毛囊炎正确答案:B 患儿皮损为脓疱、黄色厚痂,伴瘙痒及淋巴结肿大,符合寻常型脓疱疮,故选B。

167、患儿,4岁,在躯干出现多个绿豆大小、略呈纺锤形的红色风团样丘疹,有的皮损为伪足,顶端水疱,内容清,周围无红晕,幼儿自诉剧痒。最可能的诊断为A、荨麻疹B、水痘C、Hdem痒疹D、丘疹性荨麻疹E、传染性软疣正确答案:D 丘疹性荨麻疹又称荨麻疹性苔藓、婴儿苔藓或小儿荨麻疹性苔藓、急性单纯性痒疹,是婴幼儿常见的过敏性皮肤病。临床特点为散在性、性质稍坚硬、顶端有小疱的丘疹。周缘有纺锤形红晕,自觉瘙痒。

168、男性,20岁,因面部皮疹前来就诊。患者额头上有弥漫性炎性红色丘疹和脓疱皮疹,尚未接受任何治疗。适合的治疗方案为A、经常被阳光暴晒B、酮康唑霜C、1%糖皮质激素霜D、每日口服苯海拉明E、局部用甲硝唑正确答案:E 过氧苯甲酰和局部使用抗生素,如甲硝唑,是治疗与缓解玫瑰痤疮的一线用药。壬二酸、维A酸和口服抗生素也有效。异维A酸是重症、持续性玫瑰痤疮最有效的治疗药物,但有严格的治疗适应证。169、5天女婴,旧法接生,昨日起烦躁不安,吸奶困难,今出现四肢、颈、躯干呈强直性痉挛,频繁发作。应考虑为A、新生儿败血症B、新生儿颅内出血C、新生儿肺炎D、新生儿破伤风E、新生儿窒息正确答案:D 新生儿破伤风一般在生后5~7天左右发病,首发症状表现为吸奶困难及肌张力增高,结合不洁分娩史可考虑诊断。

170、一位13岁男孩的母亲在过去3天发现男孩左口周的皮损,因此带他前来就诊。皮损为红斑底、蜜色痂皮,周围有不少水疱。最好的治疗方法是A、口服青霉素B、口服阿莫西林-克拉维酸盐(力百汀)C、萘夫西林D、局部用莫匹罗星(百多邦)E、局部用杆菌肽正确答案:D 对于局限性脓疱,局部用莫匹罗星最为有效。金黄色葡萄球茵和化脓性链球茵是最常见的病因,青霉素很可能因其耐药而无效;力百汀和萘夫西林应用于严重感染;杆茵肽软膏的效果欠佳。若脓疱复发,应考虑鼻孔或皮肤破损处的感染转移。

171、一位28岁初次怀孕的孕妇前来就诊。她怀有双胞胎,正处于妊娠晚期,主诉下腹部瘙痒,皮肤出现水疱状皮损;脸部、手掌和脚底都没有病变。考虑是A、水痘B、疥疮C、妊娠瘙痒性荨麻疹性丘疹斑块D、带状疱疹E、高胆红素血症正确答案:C 妊娠瘙痒性荨麻疹性丘疹斑块,也称为妊娠性多形疹,是妊娠期最常见的皮肤病主诉,160个孕妇中就有1例,多胎妊娠的发病几率增加。该病通常发生于初次妊娠的妊娠晚期,很少在再次怀时复发。妊娠瘙痒性荨麻疹性丘疹斑块有明显的痒症状,与皮损同时出现。皮疹首先出现于腹部,多为妊娠纹周围,并可能蔓延至胸部、大腿和手臂,而脸部、手掌、脚底和黏膜表面则往往没有病变。典的皮损是多形、红宽、非滤泡性丘疹,斑块状,有时候是水疱状。该病常常在接近生产或产后初期消退。局部用保湿霜、中强效的类固醇与口服抗组胺药可以缓解症状。

172、糖尿病患者皮肤疾病应慎用的药物有A、外用皮炎平霜等含激素软膏B、外用达克宁霜C、内服西替利嗪D、外用甲癣酊E、外用硼酸溶液正确答案:A 外用糖皮质激素可经皮肤吸收,特别是当皮损面积大,用药剂量大,或皮肤有红斑、糜烂损时吸收增加,糖皮质激素可降低糖尿病患者对降糖药物的敏感性,使患者血糖升高。另外,一些糖尿病感染性皮肤病,外用各类皮质类固醇激素制剂,甚至还会使局部抵抗力降低,令病情加重。

173、外用皮质类固醇激素对以下哪个疾病无效A、湿疹B、神经性皮炎C、银屑病D、药疹E、单纯疱疹正确答案:E 174、下列疾病中哪一种咳嗽时不伴大量脓痰A、肺脓肿B、气管扩张C、支气管胸膜瘘D、急性支气管炎初期E、空洞型肺结核并发感染正确答案:D 急性支气管炎初期有上呼吸道感染的症状:鼻塞、喷嚏、声音嘶哑、全身不适,部分患者有畏寒、发热、全身肌肉酸痛、咳嗽、咳黏液样痰。

175、患者,男性,57岁,嗜烟约35年。慢性咳嗽咳痰10余年,进行性气促加重2年,被诊断为慢性支气管炎、阻塞性肺气肿,其X线胸片不会出现下列哪项A、早期正常无明显变化,中、晚期可见肺纹理增加、粗乱B、合并感染时有支气管周围炎症改变C、合并感染时有支气管柱状扩张,肺纹理蜂窝状改变D、肺容量增大,肋间隙增宽,膈肌低平E、心脏悬垂并狭长正确答案:C C项为支气管扩张的典型X线胸片表现,而不是慢支炎、肺气肿的X线胸片表现,其余各项均是正确的。A、B两项主要为慢性支气管炎的X线胸片表现,D、E两项主要为阻塞性肺气肿的X线胸片表现。

176、室上性心动过速的主要表现为A、突然发作,突然终止,节律规整,心率140次/分左右B、突然发作,突然终止,节律规整,心率200次/分左右C、节律不齐,强弱不一,心率100次/分左右D、心率时快时慢,心率40~120次/分E、节律略不整,伴有血流动力学改变,100~150次/分正确答案:B 室上性心动过速突然发作,突然中止,节律规则可以与心房颤动鉴别。心率一般为150~200次/分,主要由于房室旁路或由房室结双径路引起房室结折返性心动过速,所以心率快但规整。

177、最早能反映急性心肌梗死发生的酶学检查是A、磷酸肌酸激酶(CPK)B、CPK-MM 亚型:MM3/MM1比值C、CPK-MBD、α-羟丁酸脱氢酶E、血清肌红蛋白正确答案:E 急性心肌梗死发病后,血中肌红蛋白的增高,其高峰比血清心肌酶出现都早。肌红蛋白在起病后2小时内升高,12小时内达高峰。CPK发病后4小时增高,16~24小时达高峰。178、哪种瓣膜病变常常出现晕厥A、二尖瓣狭窄B、主动脉狭窄C、三尖瓣关闭不全D、主动脉关闭不全E、二尖瓣关闭不全正确答案:B 晕厥的发生与一过性脑缺血有关,主动脉瓣狭窄引起脑缺血可发生晕厥,多发生在直立、剧烈运动时。因为运动时周围血管扩张,而狭窄的主动脉瓣口限制心排血量的相应增加,从而引起和加重脑缺血。

179、抢救急性肺水肿伴休克患者,下列哪项措施禁用A、静脉注射快速洋地黄类药物B、下注射吗啡C、酒精湿化吸氧D、静脉注射地塞米松E、静脉缓慢注射氨茶碱正确答案:B 本题考点:急性肺水肿伴休克患者的抢救因为吗啡兼有扩张周围血管作用,减少回心血量,而不利于休克患者血压的逆转与恢复,故禁用于急性肺水肿伴休克患者。

180、哪一种药物不是治疗心力衰竭的常规药A、钙离子拮抗剂B、CEI或ARBC、β受体拮阻滞剂D、利尿剂E、洋地黄正确答案:A 钙离子拮抗剂具有负性肌力作用,一般不作为常规治疗心力衰竭的药物。

181、有关糖尿病的诊断,下列哪项正确A、三多一少症状是诊断糖尿病必须具备的条件B、空腹血糖不一定升高C、全天任何时候血糖>10mmol/L即可诊断D、所有患者都需行葡萄糖耐量试验进行诊断E、尿糖检查一定阳性正确答案:C 182、男性患者,78岁,既往体健,突然出现心悸、气促、咳嗽、咳粉红色泡沫痰,诊断为急性左心衰。其原因最不可能的是A、高血压病B、心肌病C、心肌炎D、急性心包炎E、冠心病,急性心肌梗死正确答案:D 183、一个神志模糊的患者,呼气带有烂苹果味,最可能的原因是A、大量饮酒B、糖尿病酮症酸中毒C、有机磷农药中毒D、肝性脑病E、尿毒症正确答案:B 糖尿病患者并发酮症酸中毒时,因丙酮在体内积聚,呼出气中可出现烂苹果味。

184、下列哪项不是甲亢患者的临床表现A、睡眠时心率快B、大便次数增多C、便秘D、阳痿E、低热正确答案:C 185、患者,男性,36岁。1型糖尿病20年,平时应用胰岛素治疗,血糖控制较稳定,近两年来出现双下肢水肿。空腹血糖5.1~6.3mmol/L,尿蛋白(++~+++),Cr 196μmml/L。在饮食中蛋白含量应为A、每日1.3g/kgB、每日0.8~1.2g/kgC、每日1.5~2.0g/kgD、每日o.6g/kgE、每日0.2g/kg正确答案:D 该患者已出现糖尿病肾病。故此应限制蛋白质的总量。通常主张每日膳食中的蛋白质按照0.6~0.8g/kg标准体重给予,并在限定范围内提高优质蛋白的比例。

186、慢性心力衰竭患者的生活管理治疗哪项不正确A、病情稳定者可作3~5次/周,20~30分钟/次运动训练B、戒烟,避免酗酒C、可去高海拔的地方D、低盐及限制液体入量E、坚持服药,定期复诊正确答案:C 187、下列有关风湿性疾病的描述,哪项是错误的A、属于变态反应性疾病B、与溶血性链球菌感染有关C、风湿性关节炎常导致关节畸形D、可累及全身结缔组织E、心脏病变的后果最为严重正确答案:C 188、类风湿关节炎的诊断标准中不包括A、晨僵B、3个或3个以上关节肿C、腕、掌指或近端指间关节肿D、对称性关节肿E、血沉增快正确答案:E 189、Schober试验:患者直立,在背部正中线髂嵴水平做标记为零,继续如何做标记,让患者弯腰,测量上下两个标记间的距离,若这一距离增加量短于4cm为阳性A、向下3cm做标记,向上6cm再做标记B、向下5cm做标记,向上5cm再做标记C、向下5cm做标记,向上10cm再做标记D、向下5cm做标记,向上15cm再做标记E、向下10cm做标记,向上5cm再做标记正确答案:C 190、患者,女,50岁。反复低热1年,伴四肢大小关节肿痛。WBC 8.0×109/L,Hb 100g/L,ANA(-),RF(+)。经多种抗生素正规治疗无效,可能的诊断是A、风湿性关节炎B、系统性红斑狼疮C、骨关节炎D、类风湿关节炎E、结核菌感染引起的关节炎正确答案:D 病史一年,累及四肢大小关节,RF(+),应考虑类风湿关节炎。

191、患者,女性,27岁。双手近端指间关节痛2个月,有时肿胀,伴不规则低热。体检双手近端指间关节有压痛,肿胀不明显,无畸形。血白细胞3.2×109/L,尿蛋白100mg/dl,血沉35mm/h。本例最可能的诊断是A、风湿性关节炎B、类风湿关节炎C、反应性关节炎D、系统性红斑狼疮E、急性白血病正确答案:D 系统性红斑狼疮常有如下表现:① 颧颊部红斑;②盘状狼疮;③光敏感;④口轻度溃疡;⑤非侵蚀性关节炎;⑥蛋白尿(0.5g/d)或尿细胞管型;⑦癫痫发作或精神;⑧胸膜炎或心包炎;⑨溶血性贫血或白细胞减少或淋巴细胞减少或血小板减小;⑩ 抗双链DNA抗体阳性或抗Sm抗体或狼疮细胞阳性;荧光抗核抗体阳性。以上符合4项即可诊断。

192、病原体侵入人体后,寄生在机体的某些部位,机体免疫功能使病原体局限化,但不足以将病原体清除,待机体免疫功能下降时,才引起疾病。此种表现是A、机会性感染B、潜伏性感染C、隐性感染D、显性感染E、病原携带状态正确答案:B 193、上呼吸道感染的治疗原则不正确的是A、一般治疗,休息,多喝水B、注意呼吸道隔离C、预防并发症D、对症治疗:高热给予退热治疗,高热抽搐,给予镇静、止惊等处理E、早期积极使用抗生素正确答案:E 病毒性上呼吸道感染无论轻重都不需要使用抗生素,更不需要预防使用抗生素,主要以对症支持治疗为主。如果是细茵性上呼吸道感染则应给予抗茵治疗。因此选E。

194、以下对流感疫苗的描述哪一项是正确的A、注射流感疫苗后就不会感冒了B、注射流感疫苗后就不会得流感了C、它可降低流感的发病率和死亡率,但不能减少肺炎的发生率和住院率D、它既可降低流感的发病率和死亡率,又能减少肺炎的发生率和住院率E、以上都不是正确答案:D 流感病毒分为甲、乙、丙三型。其中甲型流感又有很多亚型。流感疫苗是根据近年的主要流行情况研制的,不可能对所有类型的流感病毒均有效。它与感冒的病原学不同,对感冒无预防作用。流感后很易继发下呼吸道的感染,流感疫苗可以降低流感的发病率和死亡率,当然也可以减少肺炎的发生率和住院率,因此本题选D。

195、对重型肝炎的判断下列哪项最重要A、ALT>500U/LB、总胆红素>17μmol/LC、PTA<40%D、电解质紊乱E、乏力、腹胀、腹水正确答案:C 重型肝炎才会出现凝血酶原活动度(PTA)低于40%,其他选项在急性肝炎或肝硬化等类型时也可出现。

196、医院获得性肺炎最常见的致病菌是A、革兰氏阴性杆菌B、肺炎链球菌C、支原体D、真菌E、金黄色葡萄球菌正确答案:A 肺炎链球菌、支原体衣原体是社区获得性肺炎常见病原体。金黄色葡萄球菌、真菌亦是医院获得性肺炎的病原体,但最常见的致病菌是革兰氏阴性杆菌。

197、下列属于肝硬化门静脉高压表现的是A、肝掌B、蜘蛛痣C、脾大D、肝大E、男性乳房发育正确答案:C 198、根据下列HBV感染状况检查结果,体内出现保护性抗体、完全恢复的情况是抗-HBc、抗-HBe、抗-HBs、HBsAg、HBeAg分别为A、+ +—+ —B、+— —+ +C、— —+— —D、— — —++E、++— —+正确答案:C 抗-HBs阳性见于乙肝恢复期、HBv既往感染、乙肝疫苗接种后,是保护性抗体;抗-HBc是HBV感染或曾经感染的标志;抗-HBe是HBV感染的标志。199、HBV感染后产生的保护性抗体是A、抗-HBSB、抗-HBcIgMC、抗-HBcIgGD、抗-HBeE、抗-前S正确答案:A 抗-HBs阳性见于乙肝恢复期、HBV既往感染、乙肝疫苗接种后,是保护性抗体。

200、粪-口途径传播的肝炎有A、甲型肝炎与乙型肝炎B、乙型肝炎与戊型肝炎C、丙型肝炎与丁型肝炎D、丁型肝炎与戊型肝炎E、戊型肝炎与甲型肝炎正确答案:E 201、关于接种卡介苗,下列哪项不正确A、我国规定新生儿出生时即接种卡介苗B、每隔5年左右对结核菌素转阴者补种,直至年满18岁C、可显著降低儿童发病率及严重性D、极度营养不良者不予接种E、患湿疹或其他皮肤病者不可接种正确答案:B 我国规定新生儿出生时即接种卡介苗,每隔5年左右对结核菌素转阴者补种,直至年满15岁,而不是18岁。

202、以下关于慢性支气管炎的叙述哪项正确A、它是起源于支气管及其周围组织的慢性炎症B、它是一种具有气流受限特征的肺部疾病C、它是终末气道、肺泡和肺间质的炎症D、它是近端支气管壁肌肉和弹性组织破坏引起的异常扩张的肺部疾病E、它是一种伴有肺血管阻力增加,产生肺动脉高压的肺组织疾病正确答案:A 203、结核菌的易感人群应除外A、青春期者B、肾移植术后2年者C、青壮年D、胃大部切除术后者E、矽肺正确答案:C 婴幼儿、青春期、老年人和糖尿病患者、矽肺、胃大部切除术后者、艾滋病病毒感染者,以及接受免疫抑制剂治疗者肺结核发病率高。

204、关于慢性支气管炎的体格检查不包括A、早期体格检查可无明显异常B、早期体格检查仅有呼吸音粗糙C、随病情进展,肺底可出现干、湿啰音D、随病情进展,可出现杵状指、趾,气管移位E、晚期可有口唇发绀,下肢水肿正确答案:D 205、慢性支气管炎并发肺部感染的治疗,下列哪项不恰当A、应用敏感的抗生素B、菌苗注射C、应用祛痰药物D、应用支气管扩张药E、雾化吸入稀释痰液正确答案:B 慢性支气管炎合并肺部感染的治疗有:①控制感染:选用敏感抗生素。②祛痰、镇咳。③解痉、平喘。④雾化吸入扩张支气管,稀释痰液。

206、霍乱的治疗主要是A、病原治疗B、纠正酸中毒C、补液D、应用血管活性药物E、纠正低钾正确答案:C 207、关于人类对肾综合征出血热病毒(HFRSV)普遍易感,下列哪种除外A、农民B、野外作业人员C、在城市从事饮食行业者D、在城市居住平房的人E、旅行社工作人员正确答案:E 人类对本病毒普遍易感,各种职业的人群均可患病,以农民及野外作业人员居多,在城市以从事饮食行业或居住平房的人居多。

208、患者,男性,61岁,反复咳嗽2年,伴低热、消瘦。查体:气管左移,左上肺闻及闻及湿啰音。胸片:左上肺多个厚壁空洞,左肺门上移,诊断首先考虑为A、肺脓肿B、癌性空洞C、真菌性脓肿D、慢性纤维空洞型肺结核E、阿米巴肺脓肿正确答案:D 肺结核未及时发现或治疗不当,空洞长期不愈,空洞壁增厚,病灶出现广泛纤维化,随机体免疫力高低波动,病灶吸收、修复与恶化、进展交替发生,成为慢性纤维空洞型肺结核。X线显示一侧或两侧单个或多个厚壁空洞,多伴有支气管播散病灶及明显的胸膜增厚。209、患者,男性,28岁。因咳嗽、咳痰、发热3天就诊。查体:体温39℃,血压80/50mmHg,脉搏140次/分。治疗除控制感染外,首要的处理应该是A、使用强心剂B、补充血容量C、使用血管活性药物D、使用大剂量肾上腺皮质激素E、使用利尿剂正确答案:B 患者诊断为肺炎伴休克,临床应首先考虑为感染中毒性休克。在处理上,使用血管活性药物及肾上腺皮质激素是可以的,但最主要的是应首先补充血容量,因为在感染性休克患者常常出现外周循环血量不足。使用利尿剂及强心剂是错误的。

210、有关细菌性痢疾下列哪项是错误的A、菌痢常见致病菌为福氏痢疾杆菌,宋内痢疾杆菌次之B、志贺痢疾杆菌引起者症状较重,宋内痢疾杆菌引起者较轻C、内毒素主要与全身症状有关D、中毒型菌痢是由于机体对细菌毒素产生异常E、菌痢主要病变部位是直肠、乙状结肠正确答案:D 中毒型茵痢并严重结肠黏膜病变,其发病原理主要是由于机体对细茵毒素产生异常强烈的反应,引起急性微循环障碍等一系列病理生理改变。

211、慢性肾炎的综合治疗措施不应包括A、常规应用糖皮质激素和细胞毒药物B、积极控制高血压C、避免用氨基糖苷类抗菌药物D、肾衰竭氮质血症时限制蛋白和磷的摄入E、避免不必要的造影检查正确答案:A

212、对细菌性痢疾来说哪项是对的A、发病多在夏秋季B、近年来在临床上很少见C、粪便中有大量单核细胞D、通常结肠与小肠均有炎症E、发病前2周有饮食不洁史或与菌痢接触史正确答案:A 细茵性痢疾是一种常见肠道传染性疾病,肠道病变主要在结肠,以乙状结肠和直肠病变最显著。粪便镜检可见大量脓细胞或白细胞(>15个/高倍视野)及红细胞。发病季节多为夏秋季。

213、隐匿性肾炎患者的治疗原则是A、激素治疗B、免疫抑制剂治疗C、使用抗菌药物D、不可用中医药E、保护肾功能、避免肾损伤的因素正确答案:E

214、患者,男性,20岁。因乏力、食欲下降、厌油、恶心1周,尿黄4天就诊。查体:血压120/750mmHg,皮肤及巩膜黄染,无蜘蛛痣,无明显肝掌,浅表淋巴结无肿大,腹平软,肝肋下1cm,轻度触痛,肝区叩击痛阳性,移动性浊音(-)。下列项目中暂不予考虑的检查是A、甲肝抗体B、乙肝两对半C、肝功能检查D、电子胃镜E、腹部B超正确答案:D 患者有乏力、食欲下降、厌油、恶心,尿黄消化道疾病症状,肝大,考虑肝疾病;年轻男性,无蜘蛛痣,无明显肝掌,腹平软,移动性浊音(-),暂无肝硬化失代偿期表现,暂不考虑电子胃镜检查。

215、患者,男性,34岁,5天来发热、咳嗽、咳脓痰,量多,有腥臭味,胸片示:右肺下叶后基底段大片阴影,有空洞和液平,血常规:WBC 18×109/L,以下哪种疾病最有可能A、肺脓肿B、肺囊肿C、肺癌D、真菌性肺炎E、肺结核空洞正确答案:A 肺脓肿是多种病因所引起的肺组织化脓性改变。早起为化脓性炎症,继而坏死形成脓肿。临床特征为高热、咳嗽、咳大量脓痰。血常规白细胞、中性粒细胞升高。多发生于壮年,男性多于女性。

216、关于尿路感染临床表现的描述,不正确下列哪项A、急性膀胱炎B、急性肾盂肾炎C、慢性肾盂肾炎D、无症状细菌尿E、前列腺炎正确答案:E

217、尿细菌定量培养时,发生假阴性的主要原因不包括A、近7天内使用过抗菌药物B、尿液在膀胱内停留时间不足6小时C、消毒药混入尿标本中D、病灶和尿路不相通E、取清晨第一次尿作为送检标本正确答案:E

218、急性肾盂肾炎正确的治疗措施是A、口服环丙沙星3天B、口服复方磺胺甲基异唑7天C、根据细菌药物敏感试验选用有效的抗菌药物治疗2周D、联合应用2种以上抗菌药物进行治疗E、应用中药治疗正确答案:C

219、除下列哪项外尿路感染患者应转上级医院进一步诊疗A、男性性泌尿系感染或反复发作患者B、高热不退,出现少尿或无尿C、原有肾盂肾炎症状加重,并有明显的单侧腰痛和压痛D、出现突然寒战、高热及休克者E、尿急、尿频、尿白细胞增多正确答案:E 220、关于二氧化碳潴留临床表现的描述,下列哪项不正确A、二氧化碳潴留轻度升高容易引起呼吸加快、心率加快B、二氧化碳潴留持续升高则引起交换抑制作用C、二氧化碳潴留容易引起呼吸性酸中毒D、二氧化碳潴留容易引起呼吸性碱中毒E、二氧化碳潴留容易出现高钾血症正确答案:D 二氧化碳潴留的病理改变是二氧化碳轻度升高可引中枢神经系统、呼吸系统、循环系统兴奋,容易引起呼吸性酸中毒、代谢性酸中毒和高钾血症。

221、患者男性,45岁。1周前因进食不洁饮食出现腹泻、腹痛,自行服药后腹泻减轻。近3天出现发热,腹痛、腹胀明显;尿黄;尿量明显减少。有慢 性乙肝病史15年。近1年来自感体力下降,时感腹胀,消瘦。入院体检:意识尚清,但烦躁多语;肝病面容,巩膜轻度黄染,明显肝掌,可见蜘蛛痣;心肺未见明显异常;腹部膨隆,下腹压痛,轻度反跳痛,腹水征阳性。急诊化验:血常规:WBC 10.5× 109/L,中性0.89;电解质K+3.4mmol/L,Na+137mmol/L,Cl-98mmol/L,血氨为96μmol/L。该患者目前的并发症不包括A、自发性腹膜炎B、肝肾综合征C、结核性腹膜炎D、腹水E、肝性脑病正确答案:C 45岁男性,有慢性乙肝病史15年,近1年来自感体力下降,时感腹胀、消瘦,有可能合并肝硬化,明显肝掌,可见蜘蛛痣支持肝硬化诊断。1周前因不洁饮食出现腹泻、腹痛,服药后腹泻好转,为诱因;近3天出现发热,明显腹痛、腹胀,小便发黄,尿量明显减少,有感染症状、肾功损害症状。腹部膨隆,脐下腹有压痛,轻度反跳痛,腹水征阳性,有腹膜炎、肝肾综合征的可能。患者意识尚清,烦躁多语,血氨高,支持肝性脑病。而患者无结核病症状,暂不支持结核性腹膜炎诊断。

222、急性肾衰竭(或称急性肾损伤)病程中,无尿是指A、24小时尿量少于1000mlB、24小时尿量少于800mlC、24小时尿量少于400mlD、24小时尿量少于100mlE、24小时尿量等于10ml正确答案:D 223、引起急性肾衰竭的肾前性因素不包括A、创伤、外科手术、消化道出血、产后出血B、呕吐、腹泻、过度利尿、高热、烧伤、胰腺炎、挤压综合征、低蛋白血症C、心肌病、心包疾病、肺动脉高压、心力衰竭及休克D、过量应用扩张血管的药物、败血症;应用去甲肾上腺素E、尿路梗阻正确答案:E 224、患者,女性,30岁。因发热伴咳嗽3天入院治疗。后经检查诊断为疑似SARS。应于多长时间内进行网络直报?A、2小时B、3小时C、4小时D、12小时E、24小时正确答案:A 各级各类医疗机构和卫生人员发现非典型肺炎病例或疑似病例后,应于2小时内进行网络直报。

225、急性肾衰竭的处理要点不恰当的是A、详细查找可引起急性肾衰的原因,纠正血容量不足B、维持机体水、电解质平衡C、对所有患者均可使用呋塞米等强利尿剂D、尽可能终止使用肾毒药物E、病情需要者及早转有关医疗单位专科处理正确答案:C 对有效血容量减少导致肾脏血流灌注不足引起的急性肾衰竭,在有效血容量减少未纠正之前不宜使用强利尿荆。

226、患者,男性,30岁。左上肺浸润性肺结核。治疗2个月余。复查血常规:WBC 5.5×109/L,PLT 75.5×109/L。考虑与服用以下哪种药物有关?A、异烟肼B、乙胺丁醇C、吡嗪酰胺D、链霉素E、利福平正确答案:E 利福平可引起血小板减少。227、下列哪项提示患者的肾衰竭为慢性、不可逆性A、Hb 85g/LB、高磷低钙血症C、BUN 25mmol/LD、SCr 705μmol/LE、B超示双肾大小为85mm×40mm正确答案:E 228、院前急救人员应熟练掌握以下哪项技术A、快速控制伤情,对伤情分类B、止血、包扎C、固定、搬运D、特殊类型伤情的处理E、以上都是正确答案:E 一个合格的院前急救人员,应快速控制患者的致命伤情,并准确进行伤情分类,迅速包扎伤口以止血,对骨折进行固定后搬运,防止搬运途中骨折刺破血管,损伤神经,也可减少疼痛,有些特殊的伤情如颈椎损伤等亦应如此处理。

229、鼻出血时以下哪项处理合适A、有休克时取坐位或半坐位B、无休克时应取平卧头低位C、嘱患者将流入口中之血液尽量吐出,以免咽下刺激胃部引起呕吐D、不应予镇静剂E、不管有无休克,均应取坐位或半卧位正确答案:C 鼻出血时,尽量不要让它流入胃部,以免刺激胃黏膜,有休克时应平卧头低位,以保持大脑供血,必要时应予镇静剂,以免心情过度紧张。

230、患者,女性,65岁。因腹痛、腿抽筋而急诊。患者午餐曾进食较多海鲜。傍晚出现呕吐腹泻,大便初为黄色稀水便,量多,进而变为水样便;无里急后重;一直无尿。1小时前出现腹直肌和腓肠肌痉挛。体温:37.7℃,P 100次/分,BP75/45mmHg,脉搏细速,皮肤极干燥。粪便悬滴镜检可见到运动力强、呈穿梭状运动的细菌,革兰氏染色阴性。确诊为霍乱,其临床分期应是A、泻吐期B、脱水虚脱期C、少尿期D、多尿期E、恢复期正确答案:B 脱水虚脱期:由于持续而频繁的腹泻和呕吐,患者迅速出现失水和循环衰竭。常有腹直肌和腓肠肌痉挛,可出现少尿、无尿等肾功能障碍的表现。

231、鼻出血现场简易止血方法错误的是A、可嘱患者用手指紧捏紧两侧鼻翼10~15B、指压期间用冷水袋或湿毛巾敷前额及后颈C、松指后如仍出血,可用浸以1%麻黄碱生理盐水的棉片塞入鼻腔止血D、指压期间用热水袋或热毛巾敷前额及后颈E、松指后如仍出血可用浸以0.1%肾上腺的棉片塞入鼻腔止血正确答案:D 鼻出血现场,可嘱患者用手指捏紧两侧鼻翼以止血,如松指后仍出血,可用麻黄碱或肾上腺的棉片塞入鼻腔止血,因麻黄素和肾上腺素会收缩血管,从而止血。同时,可用冷水敷前颈及后颈,不应用热水毛巾,因冷的毛巾可收缩血管,从而止血。

232、下列慢性肾衰竭时用药应掌握的原则哪项不恰当A、了解常用药物的药代动力学和药效学特点;患者的肾功能情况及其他病理生理状况B、熟悉肾衰竭及其他病理生理状况时用药方法,按肾功能减退的程度调整某些药物特别是以原形经肾排泄的药物剂量C、首先选用肾毒性相对较小的药物,如确需应用某些有肾毒性的药物,应根据相应的方法减少药物剂量,或延长用药间隔D、对某些特殊治疗药物,如有条件可测定其血中药物浓度,如地高辛、氨茶碱、苯妥英钠、氨基糖苷类抗菌药物等E、已经开始维持性血液透析治疗的慢性肾衰竭患者不需要调整用药剂量正确答案:E 233、患者,男性,45岁。12周前无明显原因腹泻腹痛。大便呈脓血样,10余次/日。经当地用黄连素、痢特灵治疗后症状好转。现因外伤入院。体检:轻度贫血,巩膜无明显黄染,肝脾未触及,左下腹轻压痛。大便常规:红细胞40个/HP,白细胞20个/HP,最可能的诊断是A、伤寒B、急性阿米巴痢疾C、慢性菌痢D、急性菌痢E、慢性溃疡性结肠炎正确答案:C 临床有腹痛、腹泻、排脓血样大便等茵痢临床症状。大便常规示:红细胞40个/HP,白细胞20个/HP,病程超过两个月,考虑为慢性茵痢。

234、下列哪个不是休克的诊断条件A、意识障碍B、高热C、四肢湿冷D、收缩压小于10.64kPa(80mmHg)E、脉搏快,超过100次/分或不能触及正确答案:B 以上几个方面,高热仅反映机体的温度情况,并不能反映机体的组织灌注量,故不能成为休克的诊断条件。

235、下列哪项不是艾滋病急性期的临床表现A、发热B、口腔真菌感染C、咽痛D、淋巴结肿大E、恶心正确答案:B 艾滋病急性期通常发生在初次感染HIV后2~4周左右,临床主要表现为发热、咽痛、盗汗、恶心、呕吐、腹泻、皮疹、关节痛、淋巴结肿大及神经系统症状,故选择B。

236、患者,女,20岁。作静脉胆道造影检查时,出现意识淡漠,心悸、出冷汗,口唇发钳,面色苍白,HR 110次/分,R 25次/分,BP 60/4mmHg,应首先选用A、多巴胺B、地塞米松C、异丙嗪D、肾上腺素E、钙剂正确答案:D 患者考虑为过敏性休克,首选肾上腺素。

237、农村妇女,40岁。8年前其丈夫(长途车司机)病重,渐出现消瘦,体重下降明显,高热1月余,当时因家境困难未能医治病故。现该妇女出现与其丈夫类似症状,近1月余出现不规则发热,近2周出现高热,曾予抗菌药物治疗无效。查体:T 39.5℃,R 30次/分,BP 100/65mmHg,无皮疹,皮肤无黄染,颈部、腋下、腹股沟可扪及多个1cm×cm至1cm×1.5cm淋巴结,心律齐,腹平软。下列哪一项检查最有确诊价值A、胸片B、B超C、心电图D、ANAE、HIV特异性病原学检查正确答案:E 8年前其丈夫为长途车司机,有消瘦,体重下降明显,高热症状,病故,现患者出现类似症状,全身淋巴结肿大,须考虑艾滋病的可能。

238、诊断和鉴别诊断慢性前列腺炎哪项不适合A、尿常规检查B、列腺液常规镜检C、前列腺直肠指诊D、前列腺液细菌培养E、前列腺穿刺活检正确答案:E A、B、C、D都是诊断J隧性前列腺炎的方法,通过上述检查方法就可以明确诊断。前列腺穿刺活检只在前列腺癌的诊断中应用。239、高温环境下,气温超过35℃ 时,人体主要通过哪种方式进行散热A、辐射B、蒸发C、对流D、传导E、呼出正确答案:B 此状态下,除蒸发之外,其余各项作用很小。240、24小时咯血量超过多少为大咯血A、50mlB、100mlC、2004D、300mlE、500ml正确答案:E 咯血是指喉及其以下呼吸道或肺组织出血,经口腔咳出。一般认为,24小时内咯血量少于100ml为少量咯血,100~500ml为中量咯血,24小时咯血量大于500ml或一次咯血量大于100ml为大量咯血。

241、有关尿动力学检查,哪一项是错误的A、尿流率为其中一项无创伤性检查B、病史提示有逼尿肌功能改变可行此项检查C、压力流量测定是膀胱出口梗阻的最准确方法D、可准确诊断支配膀胱神经损伤的部位及严重程度E、压力流量测定有一定创伤性应慎重选择适 应证正确答案:D 尿动力学检查是观察单位时间内膀胱排尿功能变化而检测排尿时的动态功能状态,即通过测定排尿时膀胱内、尿道内和直肠内的压力,同时测定产生排尿感觉时膀胱内的容量和尿流率,来判断排尿功能障碍的原因。此检查方法主要用于鉴别膀胱出口梗阻症状和各种原因引起的逼尿肌功能障碍所致的排尿困难,但不能判断支配膀胱神经的损伤部位及严重程度。

242、有关良性前列腺增生症的药物治疗,以下哪一项是最正确的A、适用于轻、中度症状的前列腺增生症的患者B、α受体阻滞剂作用于前列腺腺细胞上,抑制前列腺增生C、5α还原酶抑制剂抑制而降低前列腺内平滑肌张力D、5α还原酶抑制剂抑制睾酮生成而降低前列腺内平滑肌张力E、5α还原酶抑制剂抑制双氢睾酮生成而使前正确答案:E 治疗前列腺增生症的2个主要药物是α受体阻滞剂和5α还原酶抑制剂。α受体阻滞剂是通过作用于前列腺组织中平滑肌细胞的受体,使其松弛,降低尿道阻力而起到缓解症状作用;5α还原酶抑制剂是通过抑制体内睾酮向双氢睾酮转化,而起到使前列腺体积缩小,缓解症状。故只有答案E正确。

243、溶血性黄疸的尿液检查结果是A、尿胆原阴性,尿胆红素阴性B、尿胆原阳性,尿胆红素阴性C、尿胆原阴性,尿胆红素阳性D、尿胆原阳性,尿胆红素阳性E、尿胆原阳性,尿胆红素强阳性正确答案:B 溶血性黄疸属于肝细胞前性黄疸,故尿液检查结果是尿胆原阳性、尿胆红素阴性。

244、现场诊断伤员心跳停止的指标是A、呼吸停止B、瞳孔散大C、大动脉搏动消失D、血压测不到正确答案:C 245、再生障碍性贫血的发病机制中,下列哪项是错误的A、发病机制未完全明了B、干细胞存在内在缺陷C、造血微环境存在缺陷D、遗传性疾病E、干细胞损伤正确答案:D 遗传因素可能是再生障碍性贫血的发病机制之一,但再生障碍性贫血并不是遗传性疾病。246、《2010版心肺复苏指南》中成人心肺复苏时胸外按压的深度为A、2~3cmB、4~ 5cmC、至少5cmD、6~7cmE、胸廓前后径的一半正确答案:C 2010版心肺复苏指南建议胸外按压的深度为至少5cm。

247、患者,女性,18岁。咽痛、咳嗽、头痛、乏力、食欲减退2周后出现混浊红棕色尿,无尿频、尿急、尿痛,眼睑、颜面部水肿。血压150/90mmHg,血红蛋白102g/L,血沉45mm/h,血肌酐、尿素氮轻度上升,抗链球菌溶血素明显升高,补体C3低下,诊断为急性肾小球肾炎。混浊红棕色尿是A、肌红蛋白尿B、血红蛋白尿C、肉眼血尿D、铁卟啉尿E、乳糜尿正确答案:C 248、患者,男性,25岁。因肾病综合征入院。查体:无贫血,肾功能正常,有严重低蛋白血症和高度水肿,尿圆盘电泳示高分子蛋白尿。下列哪个治疗方案是错误的A、优质蛋白饮食B、严格控制水和钠的摄入C、补充血制品D、应用激素和细胞毒药物E、使用利尿剂正确答案:C 249、成人心肺复苏时打开气道的最常用方式为A、仰头举颏法B、双手推举下颌法C、托颏法D、环状软骨压迫法E、以上均可正确答案:A 250、支持重型再生障碍性贫血诊断的项目是A、Hb<60g/LB、织红细胞绝对值<25×109/LC、中性粒细胞绝对值<1.0×109/LD、血小板<25×109/LE、多部位穿刺显示骨髓增生重度低下正确答案:E 多部位穿刺显示骨髓增生重度低下支持重型再障的诊断,其余各项均未达到诊断标准。251、患者,女性,35岁。发现晨起眼睑水肿2年,腰酸。体格检查BP 167/93mmHg,双踝部凹陷性水肿。血红蛋白101g/L,尿常规:蛋白(++),红细胞10~15个/HP,自细胞0~3个/HP,24小时尿蛋白定量1.8g,血浆白蛋白34g/L,血Cr133.8μmol/L,BUN 10.5mmol/L。最可能的诊断是A、缺铁性贫血B、急性肾小球肾炎C、慢性肾小球肾炎D、肾病综合征E、原发性高血压正确答案:C 252、治疗非重型再障时应首选A、造血干细胞移植B、联合应用造血刺激因子C、免疫抑制剂D、山莨菪碱+一叶秋碱+硝酸士的宁E、以雄性激素为主的综合治疗正确答案:E 以雄性激素为主的综合治疗是非重型再障的首选。

253、患者,女性,23岁。产后第5天出血寒战、高热、腰痛,尿白细胞30个/HP,且伴有夜尿增多、下腹痛,肾区叩击痛(±),血压轻度升高。血象:白细胞18×109/L。应该如何用药A、先观察体温热型,查出病因后再作处理B、首先用广谱抗菌药物C、抗菌药物治疗时少饮水D、抗菌药物治疗应在取尿标本送检后立即进行E、暂不用抗菌药物,待细菌培养结果、药敏结果出来后再用抗菌药物正确答案:D 254、患者,男性,37岁。因突然右侧剧烈腰痛伴肉眼血尿就诊,既往有风湿性心脏病、心房颤动病史。尿常规示:蛋白(++),红细胞满视野,白细胞5~8个/HP。临床首先考虑A、肾动脉栓塞B、肾动脉血栓形成C、肾静脉血栓形成D、输尿管结石E、肾静脉栓塞正确答案:A 255、以下哪项与特发性血小板减少性紫癜的健康指导无关A、预防感染B、制活动C、避免外伤D、保持乐观情绪E、不滥用解热镇痛类药物正确答案:D 保持乐观情绪不是特发性血小板减少性紫癜健康指导的相关内容。

256、患者,女性,30岁。反复颜面及双下肢水肿5年,血压升高3年,近半年反复牙龈出血,2天前出现柏油样稀大便,并感口渴,呼吸困难,2小时前出现昏迷。儿童时患过急性肝炎已治愈。为尽快确诊,应首选下列哪项检查A、血尿素氮测定B、血肌酐测定C、肝功能检查D、血糖及尿酮检查E、血常规检查正确答案:B 257、患者,女性,38岁。查:BP 160/95rmnHg。Hb80g/L,尿蛋白(+),颗粒管型2~3个/HP,BUN 10mmol/L,Cr 220μrnol/L。对该患者不宜采取A、低蛋白饮食B、高蛋白饮食C、低钠饮食D、根据尿量多少适当限水E、低磷饮食正确答案:B 258、贫血伴骨髓细胞外铁缺少及铁粒幼细胞减少最可能的诊断是A、溶血性贫血B、铁粒幼细胞性贫血C、再生障碍性贫血D、巨幼细胞性贫血E、缺铁性贫血正确答案:E 缺铁性贫血时会出现小细胞低色素性贫血,骨髓检查发现骨髓细胞外铁减少或消失,铁粒幼细胞减少。

259、患者,男性,25岁,建筑工人,跌倒时会阴部撞击于硬物上,欲尿不能,很快出现阴茎、阴囊、会阴和下腹壁肿胀瘀斑,其肿胀原因最可能为A、局部挫伤B、炎性肿胀C、局部水肿D、血肿尿外渗E、阴茎海绵体破裂正确答案:D 260、患者,男性,52岁。左输尿管上段结石伴肾积水5个月余,结石大小1.8em×0.8cm,左肾区痛伴发热一周。T:39℃,血常规:白细胞14×109/L,中性粒细胞92%,尿常规:红、白细胞满视野。经抗炎等药物治疗无效时,采取下列哪种措施是最正确的A、抗炎同时,立即输尿管切开取石+肾造瘘B、膀胱镜下逆行插管引流C、中西医结合总攻排石D、体外冲击波碎石术E、继续保守治疗正确答案:A 261、在先后解决了()等问题后,于19世纪40年代形成了现代外科学A、体外循环、麻醉、止血、抗生素、引流B、麻醉、止血、生物医学工程、无菌术C、体外循环、断肢再植、生物医学工程式、抗生素D、无菌术、麻醉、止血、输血E、无菌术、麻醉、断肢再植、止血、抗生素正确答案:D 262、缺铁性贫血的实验室检查哪一组正确A、血清铁降低,总铁结合力增高,转铁蛋白饱和度降低B、清铁降低,总铁结合力增高,转铁蛋白饱和度增高C、血清铁降低,总铁结合力降低,转铁蛋白饱和度降低D、血清铁正常,总铁结合力降低,转铁蛋白饱和度降低E、血清铁降低,骨髓铁粒幼细胞降低,转铁蛋白饱和度增高正确答案:A 缺铁性贫血的实验室检查中生化检查应该提示:血清铁降低,总铁结合力增高,转铁蛋白饱和度降低。

263、测定血清铁(SI)、血清铁蛋白(SF)、总铁结合力(TIBC)及转铁蛋白饱和度(TS)可了解机体铁代谢情况,缺铁性贫血时A、SI↓,SF↑,TIBC↓,TS↑B、SI↑,SF↓,TIBC↑,TS↓C、SI↓,S↓,TIBC↓,TS↑D、SI↑,SF↑,TIBC↑,TS↑E、SI↓,SF↓,TIBC↑,TS↓正确答案:E 缺铁性贫血时SI↓,SF↓,TIBC↑,TS↓。

264、缺铁性贫血治疗的目标为A、自觉症状改善B、细胞数及Hb量恢复正常C、红细胞形态恢复正常D、血清铁恢复正常E、血清铁蛋白恢复正常正确答案:E 缺铁性贫血治疗的目标是使血清铁蛋白恢复正常,亦即是补足储存铁量。

265、关于消化性溃疡的临床症状不正确的是A、胃溃疡多为餐后痛B、十二指肠溃疡多为空腹和夜间痛C、消化性溃疡患者均有上腹痛D、十二指肠后壁溃疡可发展为穿透性溃疡E、球后溃疡易发生大出血正确答案:C 约15%的消化性溃疡者无症状,称为无症状性溃疡。

266、下列哪种疾病不适宜作膀胱镜检查A、输尿管肿瘤B、尿道肿瘤C、尿道狭窄D、前列腺癌E、膀膀肿瘤正确答案:C 267、患者,男性,44岁。反复发作右肾绞痛1年,2年来常于进食肉类尤其是动物内脏后,出现脚趾关节红肿疼痛。泌尿系统平片检查未发现异常。对患者应进行下列哪项检查以明确诊断A、血尿酸化验和B超检查B、反复复查泌尿系统平片C、24小时尿液分析和血钙、磷、尿酸化验检查D、尿常规检查和尿细菌培养E、小关节摄片正确答案:A 该患者有肾绞痛病史,临床提示为上尿路结石,但泌尿系统平片检查又未发现异常,说明结石在X线片上不显影,结合患者有大量进食动物蛋白时出现脚趾关节红肿疼痛,临床高度怀疑为尿酸代谢障碍引起的痛风。为明确痛风的诊断和上尿路结石是否存在应选择A。268、患者,男性,30岁。右肾结石,行ESWL治疗,排出碎石颗粒成分分析为磷酸镁铵结石。为预防结石复发采取的措施哪一项是错误的A、抗感染治疗B、碱化尿液C、酸化尿液D、限制食物中磷酸的摄入E、口服氯化铵正确答案:B 磷酸镁铵结石的形成往往与尿路感染有关,通常在碱性尿液中容易生成尿路结石,在治疗中抗感染和酸化尿液是常规治疗方法。口服氯化铵可以起到酸化尿液的作用,而碱化尿液则起到相反的作用。

269、关于中枢神经系统白血病(CNSL)的叙述,错误的是A、可发生在疾病的任何阶段B、发生在治疗后缓解期C、急性淋巴细胞白血病最常见D、以急性非淋巴细胞白血病最常见E、也可见于慢性白血病正确答案:D CNSL可发生在疾病的任何阶段,但常发生在治疗后缓解期,这是由于化疗药物难以通过血脑屏障,隐藏在中枢神经系统的白血病细胞不能被有效杀灭,因而引起CNSL。以急性淋巴细胞白血病最常见,其次为M4、M5和M2,也可见于部分慢性淋巴细胞白血病和慢性粒细胞白血病。临床上轻者表现为头痛、头晕,重者有呕吐、颈项强直,甚至抽搐、昏迷。270、关于急性白血病,正确的是A、血象白细胞肯定增多B、象肯定有贫血C、血象肯定有血小板减少D、骨髓象肯定显著增生E、骨髓象原始细胞比例肯定增高正确答案:E 大多数急性白血病患者白细胞增多,也有白细胞计数正常或减少。患者常有不同程度的正常细胞性贫血,但部分患者因病程短,可无贫血。多数患者有血小板减少,但部分患者在早期可正常。约50%的患者血小板低于60×109/L,晚期血小板往往极度减少。骨髓象原始细胞占全部骨髓有核细胞≥20%为急性白血病的诊断标准。多数病例骨髓象有核细胞显著增生。少数骨髓增生低下但原始细胞仍占20%以上,称为低增生性急性白血病。因此,正确答案是E。

271、患者,52岁,1个月来,左肘部疼痛,向前臂外侧放射,肘关节活动正常,肱骨外上髁部局限性压痛,最好的治疗方法是A、手术B、理疗C、痛点封闭注射D、按摩E、石膏固定正确答案:C 272、以下为溃疡性结肠炎的治疗原则,但哪项除外A、控制急性发作B、缓解病情C、减少复发D、防止并发症E、早期手术切除全结肠可根治本病正确答案:E 273、Crohn病最常见的并发症为A、吸收不良综合征B、腹腔内脓肿C、肠梗阻D、中毒性巨结肠E、癌变正确答案:C 274、患者,女性,22岁。近3个月来月经增多,因皮肤及牙龈间断出血来诊。体检:齿龈及皮肤瘀斑,肝脾未触及,胸骨无压痛。Hb 105g/L。骨髓示:巨核细胞明显增多,成熟型减少。最可能的诊断是A、慢性再障B、急性再障C、急性淋巴细胞白血病D、特发性血小板减少性紫癜E、DIC正确答案:D 从病史资料反映年轻女性出血倾向、血小板减少、骨髓巨核细胞增多伴成熟障碍,最可能的诊断是特发性血小板减少性紫癜。

275、下列哪项不是急性出血坏死型胰腺炎的主要并发症A、消化道出血B、成人呼吸窘迫综合征C、急性肾衰竭D、心力衰竭E、胰原性腹泻正确答案:E 276、在幽门螺杆菌根除治疗之后,以下的幽门螺杆菌韵检查方法中,哪项不能用于判定治疗是否成功A、胃黏膜组织染色B、快速尿酸酶试验C、胃黏膜组织幽门螺杆菌培养D、尿素呼气试验E、血清学检查正确答案:E 即使幽门螺杆菌已经根除治疗成功,血清学检查在其后较长一段时间中都维持阳性。

277、克罗恩病发生剧烈腹痛和腹肌紧张提示A、进餐过多B、局部肠痉挛C、急性肠穿孔D、不完全性或完全性肠梗阻E、肠蠕动增加正确答案:C 腹痛是克罗恩病最常见的症状,多位于右下腹或脐周,间歇性发作,常为痉挛性阵痛伴腹鸣。如合并肠梗阻的症状提示出现不完全性或完全性肠梗阻;出现持续性腹痛和明显压痛,提示炎症波及腹膜或腹腔内脓肿形成。全腹剧痛和腹肌紧张,可能是病变肠段急性穿孔所致。278、男性,30岁,车祸伤30分钟。体格检查:发绀,烦躁不安,呼吸困难,左侧大块胸壁软化,两肺湿哕音。首要的处理是A、紧急剖胸手术B、吸氧及雾化吸入C、清除呼吸道分泌物D、对软化胸壁牵引固定E、左侧胸腔闭式引流正确答案:C 患者发绀,烦躁不安,呼吸困难,左侧大块胸壁软化,两肺湿啰音。患者有呼吸道梗阻因素,因此首先的抢救措施是清除呼吸道分泌物和异物,保持呼吸道通畅,保证肺通气。

279、下列关于溃疡性结肠炎的描述哪一项不正确A、是一种病因不明的直肠和结肠慢性炎症性疾病B、其发病可能涉及免疫因素C、其发病可能涉及遗传因素D、与环境因素无关E、可发生于任何年龄正确答案:D 溃疡性结肠炎是一种病因不明的直肠和结肠慢性非特异性炎症性疾病,病因和发病机制尚未完全明确,目前认为是由多因素作用所致,主要包括环境、遗传、感染和免疫因素。

280、Charcot三联症反复发作最大的可能是A、壶腹部癌B、肝细胞癌C、胆总管结石D、黄疸型肝炎E、细菌性肝脓肿正确答案:C Charcot(腹痛、高热寒战、感染)三联症是胆管炎的典型症状,胆总管结石常导致胆道梗阻,继发感染,出现Charcot三联症。

281、下列哪项对肝癌的诊断最有价值A、肝区疼痛B、进行性肝大C、肝质硬伴压痛D、肝质韧伴压痛E、肝缘钝伴压痛正确答案:B 肝癌时肝脏是进行性肿大。282、癌常常发生于乳腺的哪个部位A、外上象限B、乳腺内上侧C、乳腺外下象限D、乳腺尾叶E、乳腺内下象限正确答案:A 283、患者,男性,38岁。间歇性上腹痛2年,受凉后加重,嗳气,近2天疼痛加重,突然呕血500ml,继而排黑便200ml,出血后腹痛缓解,其最可能的出血原因是A、胃癌B、消化性溃疡C、慢性胃炎D、急性胃黏膜病变E、食管静脉曲张破裂正确答案:B 284、患者,男性,40岁。间歇性上腹痛3年,近日出现呕吐,吐后自觉舒适,吐物有酸臭味。查:上腹饱满,有振水音,诊断可能为A、消化性溃疡并幽门梗阻B、十二指肠淤滞症C、胃癌D、急性胃炎E、神经性呕吐正确答案:A 285、普查原发性肝癌最简单有效的方法为A、B超B、AFP定性检查C、肝脏CTD、肝脏MRIE、同位素肝扫描正确答案:B 286、患者,女性,50岁。既往无胃病史,近半年来,出现上腹痛,食欲减退,体重下降,一般状态较好。内镜发现胃角溃疡,约1.0cm×0.8cm,病理诊断为早期胃癌,首选的治疗方法是A、内镜下切除B、手术治疗C、介入动脉化疗D、内镜下局部化疗E、免疫治疗正确答案:B 患者无手术禁忌证及远处转移,应尽可能手术切除。由于早期胃癌可能有淋巴结转移因此内镜下治疗不如手术可靠。

287、关于M蛋白,错误的是A、反应性浆细胞增多症不伴有M蛋白B、多发性骨髓瘤均伴有M蛋白C、多发性骨髓瘤也可无M蛋白D、有M蛋白不一定是多发性骨髓瘤E、某些肿瘤可有反应性单克隆免疫球蛋白增多正确答案:B 绝大多数多发性骨髓瘤伴有M蛋白,但约1%的患者血清或尿中不能分离出M蛋白,称为不分泌型骨髓瘤。反应性浆细胞增多症不伴有M蛋白。有M蛋白不一定是多发性骨髓瘤,因为M蛋白还可见于巨球蛋白血症、意义未明的单克隆免疫球蛋白血症及反应性单克隆免疫球蛋白增多,如淋巴瘤可有反应性单克隆免疫球蛋白增多。

288、甲状腺大部切除术后并发症中,下列哪项是错误的A、手足抽搐B、膈肌麻痹C、阻塞性呼吸困难D、声音嘶哑E、误咽正确答案:B 289、患者,58岁,男性,因上腹饱胀隐痛5年,钡餐胃肠造影和B超检查腹部均未见异常,胃镜检查示胃窦黏膜有充血和糜烂,活检病理报告黏膜呈慢性炎症,中度肠上皮化生,重度不典型增生,最佳治疗是A、外科手术B、前列腺素类药物或硫糖铝C、对症治疗D、吗丁林和胃得乐联合治疗E、低脂易消化饮食,忌烟酒正确答案:A 病理报告是重度不典型增生者,首先考虑手术治疗。

290、患者,男性,50岁。慢性肝炎30年,2个月来出现恶心、食欲不振、乏力伴肝区疼痛,超声波显示肝右叶有一直径5cm的实性肿物,肝穿刺活检支持肝细胞癌的病理变化是A、假小叶形成B、肝细胞结节状增生C、肝细胞嗜酸性变D、肝细胞异型性明显形成实性团E、肝细胞出现双核正确答案:D 291、患者,男性,55岁。半年前曾因原发性肝癌而做右半肝切除术,手术后在门诊定期随访,下列哪项检查最能早期预测癌肿复发A、ALPB、B型超声检查C、ALTD、选择性肝动脉造影E、动态观察甲胎蛋白的变化正确答案:E AFP可以用于预测肝癌复发。

292、患者,男性,35岁。饮高度白酒450ml后第2天出现上腹及左上腹持续性疼痛,阵发性加重,并向腰背部放射,抱膝前屈位缓解,应首先考虑哪项诊断A、急性胃炎B、十二指肠穿孔C、急性胆囊炎D、急性胰腺炎E、急性阑尾炎正确答案:D 有酗酒的诱因,起病急,持续腹痛,向腰背部放射,抱膝前屈位缓解,符合急性胰腺炎症状。293、患者,女,26岁。3个月来面色苍白、巩膜黄染。脾肋下3.5cm。Hb 75g/L,网织红细胞0.09,Coombs试验阳性,酸溶血试验阴性。治疗应首选A、输注红细胞B、脾切除C、肾上腺皮质激素D、丹那唑E、保肝治疗正确答案:C 从病例资料反映应诊断为自身免疫性溶血性贫血,故肾上腺皮质激素是治疗的首选药。294、患者,男性,65岁。有高血压及冠心病多年,某晚参加宴会后次晨发现已死亡,检查发现床上有少量咖啡色渣液,左侧腰腹部有大片青紫斑,可能的死因是A、急性心肌梗死B、重症急性胰腺炎C、高血压脑出血D、胃溃疡出血E、高血压脑梗死正确答案:B 有饱餐的诱因,左侧腰腹部有大片青紫斑考虑是Grey-Turner征,有上消化道出血,考虑为重症急性胰腺炎引起的猝死。

295、患者,男性,65岁。反复上腹隐痛半年伴纳差、消瘦,近2个月开始出现黄疸伴瘙痒。大便陶土样色,尿黄,胆囊大。首先应考虑诊断为A、胆总管结石B、胆囊结石C、胰头癌D、壶腹部癌E、胆总管癌正确答案:D 持续性上腹痛或不适,胃纳差,消瘦,进行性加深的黄疸应考虑为胰腺癌,可扪及无痛性肿大的胆囊,称为Courvoisier征,是诊断胰腺癌的重要体征。296、24岁妇女诉脐周疼痛。在9个月前刚从艺术学校毕业后开始她的第一份工作,这些症状就出现了。在此期间,她已经多次便秘持续4~5天。随后是典型的3~4天的频繁腹泻。她否认血便,发热,体重减轻,或食欲改变。她的症状通常是周末较轻。她的体格检查正常。血细胞计数是正常的,红细胞沉降率是4mm/h,血清白蛋白和肝功能试验是正常的。以下哪项是最有可能的诊断A、克罗恩病B、憩室病C、贾第鞭毛虫感染D、肠易激综合征E、溃疡性结肠炎正确答案:D 本患者有典型的腹痛症状伴腹泻和便秘交替。她是一个年轻人,有慢性症状,这是非常典型的肠易激综合征。此诊断的患者一般都有与膳食或压力有关的疼痛症状,且有排便习惯改变。这些患者都没有查体证据或化验结果表明炎症过程。使饮食变化,并减轻压力通常是治疗目标,加用解痉药物治疗令症状减轻。克罗恩病是与一些炎症相关,包括腹部检查疼痛,黏液脓性或血便,体重减轻,伴或不伴白细胞增多与血沉(ESR)升高。憩室痛很少发生在这么年轻的患者身上,通常会便秘而没有腹泻症状。贾第鞭毛虫感染可能会导致腹部慢性疼痛,但通常为上消化道的症状,如恶心、呕吐和嗳气,因为往往贾第鞭毛虫生活在上段小肠。溃疡性结肠炎表现为血便且有炎症证据,如血沉升高。

297、预防急性胰腺炎的措施中,哪项不正确A、积极治疗胆道疾病B、戒酒C、常用抑制胰酶活性的药物D、避免服用引起急性胰腺炎的药物E、避免暴饮暴食正确答案:C 298、患者,男,40岁。面色苍白,疲乏无力1个月。血象:Hb 60g/L,WBC 3.6×109/L,分类可见少量幼粒、幼红细胞,血小板65×109/L。此患者可排除A、骨髓增生异常综合征B、骨髓纤维化C、骨髓转移瘤D、再生障碍性贫血E、巨幼细胞贫血正确答案:D 血象分类可见少量幼粒、幼红细胞不应该是再生障碍性贫血的血象特点,而其余均有可能出现。

299、下述哪项不是溃疡性结肠炎的常见并发症A、中毒性巨结肠B、直肠结肠出血C、癌变D、多发性瘘管E、急性肠穿孔正确答案:D 300、患者,男性,18岁。挫伤后左膝关节逐渐肿大,关节腔抽出血性积液,下一步最重要的检查是A、血常规B、BTC、APTTD、PTE、PAIg正确答案:C 该男孩最可能的诊断为血友病,血友病包括血友病A(FⅧ:C缺乏)、血友病B(FⅨ缺乏症)及遗传性FXI缺乏症,因此APTT会延长,而其余各项检查不会异常,所以最重要的检查是APTT。

301、肌力测定的分级描述中,壁堡的是A、1级:肌完全不能收缩,为完全瘫痪B、2级:肌收缩可使关节活动,但不能对抗重力C、3级:肌仅有抗重力,无抗阻力收缩D、4级:肌有抗重力和抗阻力收缩E、5级:肌有对抗强阻力收缩正确答案:A 脑、脊髓和周围神经疾病,常需测定肌肉的瘫痪程度与治疗中肌力的恢复情况。肌力分六级:0级:肌肉完全不收缩,为完全瘫痪;1级:肌肉稍有收缩,不能带动关节活动;2级:肌肉收缩可使关节活动,但不能对抗重力;3级:肌肉仅有抗重力,无抗阻力的收缩;4级:有抗重力和抗阻力的收缩;5级:有对抗强阻力的收缩,为正常肌力。

302、诊断再生障碍性贫血的必备条件为A、全血细胞减少B、网织红细胞绝对值<15×109/LC、骨髓有核细胞增生低下D、骨髓非造血细胞增多E、骨髓巨核细胞数量明显减少正确答案:E 骨髓巨核细胞数量明显减少或缺乏是诊断再障的必备条件。

303、患者,男性,42岁。10天前右大腿外伤,当时X线摄片无骨折,超声检查未见血肿,3天后右腿疼痛加重并有发热38.5℃,2天后体温上升到39℃,并伴有寒战,拟诊为右大腿深部脓肿。下列表现中哪项不符合患者病情A、局部红肿不明显B、有全身症状C、局部压痛明显D、局部有波动感E、穿刺有脓正确答案:D 304、患者,男,65岁。3年前右股骨颈骨折,三翼钉内固定,1年前右髋关节疼痛,近来加重,X线片可见右股骨头明显变形,可诊断为A、创伤性关节炎B、股骨头缺血性坏死C、继发髋关节结核D、骨折畸形愈合E、髋关节退行性变正确答案:B 305、最常用的体温测定方法是A、腋测法B、口测法C、腋测法和口测法D、肛测法E、以上都不是正确答案:A 临床上最常用的体温测定方法是腋测法。

306、胸骨后绞榨样痛并有重压窒息感,常见于A、肺梗死B、急性心包炎C、夹层动脉瘤D、胸膜炎E、心绞痛正确答案:E 胸痛的程度可呈剧烈、轻微和隐痛。胸痛的性质可有多种多样。心绞痛呈绞榨样痛并有重度窒息感。

307、干咳或刺激性咳嗽常见于下列哪种疾病A、急性或慢性咽喉炎B、支气管扩张症C、慢性支气管炎D、肺脓肿E、肺炎正确答案:A 干咳或刺激性咳嗽常见于急性或慢性咽喉炎、喉癌、急性支气管炎初期、气管受压、支气管异物、支气管肿瘤、胸膜疾病、原发性肺动脉高压以及二尖瓣狭窄。308、咳浆液性粉红色泡沫样血痰的疾病是A、肺梗死B、性肺水肿C、白血病D、肺结核E、肺炎正确答案:B 咳浆液性粉红色泡沫样血痰的疾病是急性肺水肿。

309、肝源性水肿的特点是A、软而移动性大B、多从眼睑开始继而遍及全身C、主要与水钠潴留有关D、失代偿期肝硬化主要表现为腹水E、多伴有淋巴回流受阻正确答案:D 肾源性水肿早期出现于眼睑及颜面,以后发展为全身水肿,常见于各型肾炎及肾病。主要机制是多种因素使肾排泄水钠减少,导致水钠潴留。

310、引起前庭障碍性呕吐的疾病是A、癔症B、震荡C、梅尼埃病D、颅内血肿E、胃神经官能症正确答案:C 引起前庭障碍性呕吐的疾病是梅尼埃病、晕动病等。

311、引起精神性呕吐的疾病有A、癔症B、脑震荡C、颅内血肿D、胃肠疾病E、梅尼埃病正确答案:A 引起精神性呕吐的疾病有胃肠神经症、神经性厌食、癔症等。

312、肥胖是指体重超过标准体重的A、10%B、20%C、55%D、25%E、15%正确答案:B 体内中性脂肪积聚过多,主要表现为体重增加,当超过标准体重的20%以上者为肥胖。

313、上消化道出血部位一般是指A、食管B、贵门C、幽门D、十二指肠球部E、屈氏韧带以上的消化器官正确答案:E 上消化道出血部位一般是指屈氏韧带以上的消化器官,包括食管、胃、十二指肠的出血。

314、一患者.体温渐上升达39℃ 或以上,数天后又逐渐下降至正常水平,持续数天后又逐渐上升,如此反复多次。此种热型为A、稽留热B、弛张热C、间歇热D、波状热E、回归热正确答案:D 波状热体温在数小时内逐渐上升至39℃或以上,经数天逐渐降至正常,持续数天后又开始发热,如此反复多次,常见于布氏杆茵病。

315、男性,37岁。饮酒后突发上腹部剧痛20分钟,伴恶心、呕吐、腹胀。查体:强迫体位,上腹部带状压痛,轻度肌紧张,无反跳痛,诊断首先考虑A、消化性溃疡穿孔B、急性胰腺炎C、急性胆囊炎D、急性胃肠炎E、急性阑尾炎正确答案:B 暴饮暴食可诱发急性胰腺炎。该患者饮酒后突发上腹部剧痛,上腹部带状压痛多考虑急性胰腺炎。

316、男性,57岁。突然头痛剧烈,持续不减,伴有不同程度的意识障碍。T:36.2℃,颈强(+),提示A、感染性疾病B、偏头痛C、蛛网膜下腔出血D、肌紧张性头痛E、神经官能症正确答案:C 蛛网膜下腔出血起病急骤,发病后可出现剧烈头痛,因为大量的血液进入蛛网膜下腔,使脑脊液循环发生障碍,使颅内压增高;由于血液刺激脑膜可产生颈部肌肉痉挛,使颈部活动受限,严重时出现颈项强直,神经系统检查克氏征阳性,布氏征阳性。部分患者还可出现烦躁不安、谵妄、幻觉等精神症状,或伴有抽搐及昏迷等。

317、男性患者,28岁。反复上腹正中疼痛,进食后疼痛缓解,4小时前突发全腹剧烈疼痛。查体:全腹肌紧张,压痛及反跳痛。考虑消化性溃疡合并穿孔。下述哪一项支持此诊断A、腹痛前饮酒B、腹痛部位不固定C、腹痛伴肝浊音界缩小或消失D、腹痛伴黄疸E、腹部受外部暴力正确答案:C 肝浊音界缩小或消失代之以鼓音者,多由于肝表面覆有气体所致,是急性胃肠穿孔的一个重要征象。

318、男性,12岁。阵发性剑突下钻顶样疼痛,辗转不安,缓解时无不适,查体:腹软,无压痛及反跳痛,Murphy征(-),肝区叩击痛(-),可能诊断为A、急性胰腺炎B、消化性溃疡穿孔C、急性肠梗阻D、胆道蛔虫症E、结核性腹膜炎正确答案:D 阵发性剑突下钻顶样疼痛是胆道蛔虫症的典型症状。

319、男性患者,21岁。昨夜会餐,5小时后开始出现恶心,呕吐胃内容物,伴腹痛、腹泻。查体:体温37℃,心肺听诊正常,腹软、无压痛,最可能的诊断是A、梅尼埃病B、急性胃肠炎C、急性胰腺炎D、幽门梗阻E、肠梗阻正确答案:B 急性胃肠炎是胃肠黏膜的急性炎症,临床表现主要为恶心、呕吐、腹痛、腹泻、发热等。其发生多由于饮食不当,暴饮暴食,或食入生冷腐馊、秽浊不洁的食品所致。

320、脑出血急性期血压在什么范围内应使用降压治疗A、收缩压≥140mmHg,或舒张压≥90mmHgB、缩压≥160mmHg,或舒张压≥95mmHgC、收缩压≥180mmHg,或舒张压≥105mmHgD、收缩压≥200mmHg,或舒张压≥110mmHgE、收缩压≥220mmHg,或舒张压≥120mmHg正确答案:C 脑出血后血压升高是对颅内压增高情况下为保持相对稳定的脑血流量的脑血管自动调节反应,通常可不用降压药。收缩压180~230mmHg,或舒张压105~140mmHg,宜口服降压药治疗,使血压维持在略高于发病前水平;收缩压180mmHg以内,或舒张压105mmHg以内,可观察而不用降压药。

321、患者,男性,65岁。既往有高血压病病史,与家人吵架后突然出现一侧肢体偏瘫、该侧深浅感觉均有障碍,上视不能,其最有可能的诊断为A、丘脑梗死B、脑出血C、壳核梗死D、壳核出血E、尾状核出血正确答案:B 患者为老年患者,既往有高血压病史,情绪激动后起病,因此多为脑出血,由于查体见上视不能,出血部位考虑为丘脑的可能性大。

322、男性,22岁。有癫痫病史,频繁抽搐持续二小时并意识不清。应诊断为A、晕厥B、单纯部分性发作C、癫痫持续状态D、自动症E、失张力发作正确答案:C 癫痫持续状态是癫痫连续发作之间意识尚未完全恢复又频繁再发,或癫痫发作持续30分钟以上未自行停止。选择答案为C。

323、患者,男性,44岁。Graves病史多年。平时不规则治疗。两天前因感染后出现甲亢危象。下列治疗哪项是错误的A、大剂量丙硫氧嘧啶B、口服复方碘或静滴碘化钠C、应用利血平或心得安D、大剂量阿司匹林降温E、氢化可的松静滴正确答案:D 水杨酸盐可与T4、T3竞争与甲状腺结合蛋白的结合,使游离甲状腺激素增加,且大量水杨酸盐也增加代谢率,故在甲亢危象中不宜使用。

324、男性患者,65岁。无明显诱因于晨起后突感胸骨后闷痛,持续2小时不缓解,伴有胸闷、气短。查体:血压125/80mmHg,双侧肺底有细小啰音。心电图示Ⅱ、Ⅲ、aVF导联ST段弓背向上抬高,T波倒置,该患者的心功能A、NYHA分级ⅢB、NYHA分级ⅣC、NYHA分级ⅡD、Killlip分级ⅠE、Killlip分级Ⅱ正确答案:E

325、女性患者,65岁。高血压病史10余年,平时血压在170/100mmHg左右,突然出现胸闷,气短,咳嗽,端坐呼吸,血压200/110mmHg,治疗应首选A、毛花苷CB、呋塞米C、氨茶碱D、美托洛尔E、硝普钠正确答案:E

326、关于“全科医生”,以下说法最恰当的是A、全面掌握各科业务技术的临床医生B、提供“六位一体”全部服务内容的基层医生C、专门为社区群众提供上门医疗服务的基层医生D、经全科医学专业培训合格,在社区提供长期负责式医疗保健的医生E、以公共卫生服务为主的医生正确答案:D

327、全科医学与社区卫生服务的关系可以体现为A、发展全科医学教育,培养全科医生B、全科医疗是社区卫生服务的最佳服务模式,全科医生是社区卫生服务的主力军C、全科医生在工作中应了解替代医学的知识,也应看到替代医学的局限性D、发展社区护理E、全科医生在工作中充分掌握替代医学的方法,为患者提供及时的服务正确答案:B

328、全科医生在临床诊断过程中,概率是主要的判断依据之一,这里的概率是指A、该病的现患病率B、该病的发病率C、该病的病死率D、医生根据症状判断患该病的概率E、该患者在服务人群中所占有的比例正确答案:D

329、由家庭所在的社会文化传统而来的权威,属于以下哪种类型权力结构A、传统权威型B、工具权威型C、分享权威型D、感情权威型E、以上都不是正确答案:A 330、社区卫生服务计划的主要内容不包括A、时间安排和经费预算B、服务对象的人数C、活动地点和指标D、本活动的长期健康效益E、质量控制措施正确答案:D 331、衡量一个国家是否进入老龄化社会的标准是A、60岁及以上的老年人口比例>7%B、65岁及以上的老年人口比例>6%C、60岁及以上的老年人口比例>10%D、65岁及以上的老年人口比例>8%E、以上都不是正确答案:C 332、下面哪项不属于临床预防医学的特征A、是在临床场所执行的B、是一种群体预防C、主要针对慢性病D、对象是健康人和“无症状”患者E、是一级和二级预防的结合正确答案:B 333、以下哪项不是临床预防医学的方法A、免疫接种B、随诊随访C、筛检D、病人教育E、病例发现正确答案:B 334、关于活疫苗的说法正确的是A、容易保存B、需多次注射C、局部和全身反应比较明显D、可以长时间保存E、接种后免疫效果持续时间长甚至终生正确答案:E 解析:活疫苗是指原病毒的毒力很强,经人工变异,成为车较弱的、毒力很轻的活病毒,接种人体后只能引起轻微反应,但免疫效果较为可靠。

335、健康教育要给人们提供A、卫生知识、经验与物质支持B、信息、技术和经验支持C、卫生知识、技能与服务D、卫生宣传、经验和反馈E、技能、卫生知识和经验正确答案:C 336、下述哪项不是以人为中心应诊中的任务A、确认并处理现患问题B、管理慢性活动性问题C、提供机会性预防D、改善病人的就医和遵医行为E、分析病人的社会、文化和经济相关问题正确答案:E 337、下面哪项不是社区卫生服务“六位一体”的综合功能A、疾病预防B、专科医疗C、保健D、康复E、健康教育正确答案:B 338、“初级保健管理、以人为本、特殊问题的解决技能、综合性服务、社区为导向、整合性模式”是A、实施初级卫生保健的基本原则B、实施社区卫生服务的原则C、实施人人享有卫生保健的原则D、全科/家庭医师的能力要求E、社区卫生人员的能力要求正确答案:D 339、可指导和组织健康促进干预实施的理论是A、健康信念模式B、创新扩散理论C、社会认知理论D、“知-信-行”理论E、格林模式正确答案:E 340、下列大于老年人运动时间和环境的说法哪一项是不正确的A、老年人不宜饱餐后运动B、下午和傍晚是老年人最佳的运动时间C、清晨树林空气清新最适合老年人运动D、清晨树林不适宜老人运动E、雾天不适宜户外运动正确答案:C 341、老年冠心病的膳食保健中以下哪项观点是错误的A、建议每日钠盐的摄入量少于6gB、适量增加糖的摄入C、增加膳食纤维的摄入D、严格限制脂肪类食物E、提倡适量摄入富含不饱和脂肪酸的植物油正确答案:D 342、老年冠心病的一级预防中被提到的首位药物是A、硝酸酯类B、降脂剂C、钙离子拮抗剂D、阿司匹林E、血管紧张素转换酶抑制剂正确答案:D 343、上题中,该男子较其他人更容易得A、冠心病B、糖尿病C、关节炎D、耳聋E、胃癌正确答案:A 解析:国内外的许多调查已经证明,A型行为者的冠心病发病率明显高于B型行为者。344、某家庭,父亲及儿子都抽烟,母亲肥胖,从不进行体育锻炼;女儿上大学,在学校居住,每天坚持锻炼,饮食合理。这家父亲、母亲及儿子具有A、日常健康行为B、避开环境危害行为C、预警行为D、预防性行为E、不良生活方式正确答案:E 345、上题中,她儿子的行为属于A、预防性行为B、病人角色行为C、A型行为D、D型行为E、预警行为正确答案:A 346、下列有关定量凋查描述错误的是A、定量调查常采用问卷的方式进行B、访谈法一般在样本较大或调查对象较集中的调查中使用C、现场自填法是现场把问卷发给被调查对象,填好后收回D、信访法的回收率较高E、定量研究收集资料的过程称为定量调查正确答案:D 解析:信访法是将问卷邮寄给调查对象,填好后再寄回给调查者,这种方法回收率比较低。347、选择10所幼儿园的儿童进行某疫苗的效果观察,随访3年结果表明83%的疫苗接种者未发生该病,由此可认为A、该疫苗预防效果欠佳,仍有17%儿童患病B、该疫苗预防有效,因可保护83%儿童不患病C、不能下结论,因为3年观察时间不够D、不能下结论,因为未没对照组E、不能下结论,因为所选对象覆盖面小正确答案:D 解析:流行病学实验研究的基本原则是设立对照,目的是保证实验的效应是真实可靠的,故选D。

348、某高校测得1100名20岁健康男大学生的身高,经检验资料服从正态分布,其均值为172cm,标准差为4cm,求得的区间(172-2.58×4,172+2.58×4)称为身高的A、99%医学参考值范围B、95%医学参考值范围C、99%总体均数可信区间D、95%总体均数可信区间E、以上都不是正确答案:A 解析:本题目是考察对医学参考值范围和总体均数可信区间的理解。题目中求得的区间(172-2.58×4,172+22.58×4)为均数±2.58×标准差,所以是99%医学参考值范围。349、对于一组正态分布的资料,样本含量为n,样本均数为,标准差为S,该资料的医学参考值范围为A、B、C、D、E、正确答案:A 350、对于一组正态分布的资料,样本含量为n,样本均数为,标准差为S,该资料的总体均数可信区间为A、B、C、D、E、正确答案:C 351、在两组正态分布资料比较的t检验中,结论是P<0.05,差别有统计学意义,则P越小,说明A、两样本均数差别越大B、两总体均数差别越大C、两样本均数有差别的可能性越大D、越有理由认为两样本均数不同E、越有理由认为两总体均数不同正确答案:E 解析:在假设检验中,得出的P值越小越有理由认为两总体均数不同,而不能说P值越小两均数差别越大。

352、关于统计资料的列表原则,错误的是A、横标目是研究对象,列在表的左侧;纵标目是分析指标,列在表的上方偏右侧B、线条主要有顶线、底线及纵标目下面的横线,分析指标后有斜线和竖线C、数字右对齐,同一指标小数位数一致,表内不宜有空格D、备注用“*”标出,写存表的下面E、标题在表的上端,简要说明表的内容正确答案:B 解析:制作统计表时,一般用3~4条基本横线,不应有任何纵线和斜线。

353、知信行理论(KABP)中,A代表A、知识B、态度C、信念D、决心E、行为正确答案:B 解析:A是Attitude,代表态度。

354、当吸烟者吸烟时,给予讲解吸烟致癌等事例,从而助矫正吸烟行为,这种个体行为矫正方法为A、脱敏法B、示范法C、厌恶法D、强化法E、消除法正确答案:B 355、知识、信念、态度、价值观属于以下哪种影响行为的因素A、倾向因素B、促成因素C、强化因素D、危险因素E、核心因素正确答案:A 356、“近期和中期效果评价”义称为A、形成评价B、过程评价C、效应评价D、结局评价E、总结评价正确答案:C 357、政府规定禁止向未成年人销售香烟,这属于健康促进策略中的A、环境策略B、社会策略C、健康教育策略D、监测评价策略E、行为干预策略正确答案:B 解析:政策、法规、奖惩办法等属于社会策略。

358、社区健康促进的核心策略是A、加强社区领导B、建立组织网络C、开展人员培训D、开发利用社区资源E、社会动员正确答案:E 359、不属于老年人合理膳食的是A、合理营养B、低盐膳食C、高脂膳食D、多样化E、三餐合理搭配正确答案:C 360、学龄期是指A、从出生脐带结扎开始到生后满32天B、从出生到满5周岁前C、1周岁到满7周岁前D、7周岁到入小学前E、从入小学起到青春期之前正确答案:E 361、幼儿开始能区别各种形状,对图片感兴趣时,已有A、1~3个月B、5~7个月C、11~18个月D、1周岁E、2周岁正确答案:C 解析:小儿在5个月时能看自己的手,注视物体,7~11个月能看到小物体,11~18个月能区别各种形状,对图片感兴趣。

362、含有特异性抗体的免疫制剂接种于人体后,可立即获得免疫力,这种免疫称为A、基础免疫B、加强免疫C、自动免疫D、被动免峻E、应急接种正确答案:D 解析:接种丙种球蛋白、胎盘球蛋白等特异性抗体,使人体立即获得免疫力,称为被动免疫。363、预防佝偻病发生的措施错误的是A、母孕期应多晒太阳B、按时添加辅食C、坚持母乳喂养D、婴儿出生后2个月开始补充预防剂量的维生素DE、母孕期末3个月应补充预防剂量的维生素D正确答案:D 解析:婴儿应在出生后2周开始服用维生素D。每日400IU,同时补充钙剂,其余选项均为正确的预防措施。364、婴儿4天,于生后第2天出现黄疸,体查:皮肤黄染,精神好,哭声大,血清胆红素150μmol/L,该患儿可诊断为A、生理性黄疸B、新生儿败血症C、新生儿肝炎D、新生儿溶血症E、新生儿胆道闭锁正确答案:A 解析:生理性黄疸是新生儿出生后2~14天内,单纯由于新生儿胆红素代谢特点所致,无临床症状,肝功能正常,血清胆红素不超过220.6μmol/L。

365、让孕妇直接方便地自我监测胎儿宫内情况的方法为A、测体重B、数胎动C、测腹围D、听胎心音E、测子宫高度正确答案:B 解析:胎动减少或者胎动剧烈均应考虑胎儿在宫内是否缺氧。一般胎动消失后数小时,甚至1~2天后胎心音消失。因此,在发现胎动消失后及时终止妊娠,尚有挽救重危胎儿的可能。366、对妇女更年期的描述,不正确的是A、通常发生存45~55岁B、卵巢功能逐渐衰退到丧失C、为每位妇女必经的过程D、为正常的生理过程E、均会出现更年期综合征正确答案:E 367、用电子胎心监护仪监测胎心率时,提示胎儿缺氧的指标是A、早期减速B、变异减速C、晚期减速D、变异的频率≥6次/分E、胎心率的波动范围在10~25次/分正确答案:C 解析:晚期减速特点是子宫收缩开始后一段时间出现胎心率的减慢,但是下降缓慢,持续时间较长,回复也缓慢,一般认为是胎儿缺氧的表现。早期减速一般认为是宫缩时胎头受压,脑血流量一时性减少的表现。变异减速一般认为是子宫收缩时脐带受压兴奋迷走神经所致。而变异的频率≥6次/分和波动范围在10~25次/分是正常胎心率基线变异的频率和振幅。368、老年人的思维特点是A、不易集中精力思考问题,思维迟钝B、计算速度减慢,计算能力减退C、对语言的理解速度减慢,讲话变缓D、思维的敏捷度、流畅性比中青年时差E、以上都是正确答案:E 369、患者,男性,70岁,因发热、胸痛,X线片可见肺炎浸润征象,治疗过程中出现夜间阵发性呼吸困难,需要坐起来或半卧位可缓解,此时要考虑并发A、呼吸道通气不畅B、肺衰竭C、右心衰竭D、左心衰竭E、冠心病正确答案:C 370、恶性胃溃疡常表现为A、周期性胃痛明显,无上腹包块B、粪便隐血持续阳性C、龛影直径<2.5cm,壁光滑,位于胃腔轮廓之外D、胃液分析胃酸正常或偏低,但无真性缺酸E、胃镜检查见溃疡圆或椭圆形,底平滑,边光滑,白或灰白苔,溃疡周围黏膜柔软,可见皱襞向溃疡集中正确答案:B 371、患者,女性,67岁,既往肺心病病史20年,此次患大叶性肺炎,咳嗽,咳痰2周,在家自己服用抗生素,未见明显效果,并出现呼吸困难,昨天出现烦躁不安,接着神志恍惚,送至医院就诊,查体:口唇发绀,颈静脉充盈,两肺底闻及细湿啰音,体温37.4℃,脉搏110次/分,血压13/9kPa,双下肢水肿,尿蛋白阳性,大便隐血阳性,此病人最可能出现了下述哪个并发症A、呼吸衰竭B、上消化道出血C、急性脑出血D、肾衰竭E、急性心力衰竭正确答案:A 372、乙胺丁醇主要不良反应是A、肾盂肾炎B、心肌炎C、中毒性肝炎D、膀胱炎E、视神经炎正确答案:E 解析:乙胺丁醇的不良反应主要是球后视神经炎,胃肠道反应,偶见过敏反应、肝功能损害及神经精神症状等。

373、第三代头孢菌素的特点,叙述错误的是A、体内分布较广,一般从肾脏排泄B、对各种β-内酰胺酶高度稳定C、对G-菌作用不如第一、二代D、对铜绿假单胞菌作用很强E、基本无肾毒性正确答案:C 解析:第三代头孢菌素对革兰阳性菌作用弱,对革兰阴性菌的作用强。

374、某男,因伤寒服用氯霉素,一周后查血象发现有严重贫血和白细胞、血小板减少,这种现象发生的原因是A、氯霉素破坏了红细胞B、氯霉素缩短了红细胞的寿命C、氯霉素抑制了线粒体整合酶的活性D、氯霉素抑制了高尔基体的功能E、氯霉素加强了吞噬细胞的正确答案:C 375、可引起亚急性脊髓-视神经病的抗阿米巴病药是A、甲硝唑B、氯喹C、喹碘方D、二氯尼特E、依米丁正确答案:C 376、实施临床试验前,对无行为能力的病人要获得其家属的同意,这属于A、知情同意B、代理同意C、无效同意D、家属同意E、间接同意正确答案:B 解析:特殊病人(婴幼儿病人、智残病人、休克病人等),因本人不能、不宜行使知情同意权,而由其家属或其他适合的代理人代行此权。

377、以下哪项与自然流产无关A、经产妇B、子宫畸形C、甲状腺功能低下D、宫颈重度裂伤E、黄体功能不全正确答案:A 378、以下哪项是外阴阴道假丝酵母菌病的特点A、黄绿色稀薄泡沫状自带,检查阴道黏膜有散在出血斑点B、分泌物涂片可见线索细胞C、阴道pH值>4.5D、氨臭味试验阳性E、白色稠厚豆渣样白带,检查小阴唇内侧及阴道黏膜附着白色膜状物正确答案:E 379、以下哪项是急性盆腔炎的病理变化A、输卵管积水B、输卵管卵巢囊肿C、卵巢很少单独发炎,常与输卵管伞端粘连形成卵巢周围炎D、黏液脓性宫颈炎E、以上均不是正确答案:C 解析:输卵管积水及输卵管卵巢囊肿是慢性盆腔炎的病理表现;黏液脓性宫颈炎是最常见的急性宫颈炎类型。

380、青春期功能失调性予宫出血的治疗原则是A、减少月经量B、促进子宫发育C、止血、调整月经周期、促排卵D、抗生素治疗E、防止子宫内膜病变正确答案:C 381、关于子宫内膜异位症的临床表现,下列叙述错误的是A、因子宫内膜异位的部位不同,症状差别大B、异位的子宫内膜面积大,则症状明C、痛经的特点为继发性和进行性加重D、不育是因盆腔粘连、子宫后倾、卵巢功能失调及性交疼痛造成的E、体征随部位不同而改变正确答案:B 382、患者,35岁,下腹部肿物3年,逐渐增大,自右下腹部开始如拳头大小,现已如妊娠足月大小,但仍能做轻微的家务劳动。最可能的肿瘤为A、卵巢畸胎瘤B、卵巢浆液性囊腺瘤C、卵巢黏液性囊腺瘤D、子宫内膜样肿瘤E、卵泡膜细胞瘤正确答案:C 383、患者女性,18岁,使用口服避孕药后发现部分面部皮肤变暗。最可能的诊断是A、黑斑病B、冻疮样狼疮C、恶性黑素瘤D、皮脂腺增生E、肾上腺增生正确答案:A 解析:部分服用口服避孕药的女性会出现黑斑病。这种病的典型症状就是出现黑色素沉着,可能影响面部。此种情况也出现于妊娠期,日光照射会使情况恶化。通常,当妊娠结束或停止口服避孕药后,色素沉着会自行消失。口服避孕药不增加恶性黑色素瘤的风险。

384、患者女性,48岁,不规则阴道流血前来就诊。合适的初步处理措施为A、子宫内膜活检B、口服避孕药试验C、注射甲羟孕酮D、转外科E、骨盆检查和巴氏涂片检查正确答案:A 解析:生殖系统成熟的女性,若大于40岁,不规则阴道流血,应进行子宫内膜活检来排除子宫内膜增生症或癌症。另外,也应考虑其他检查来排除甲状腺功能不全和出血性疾病。如果青春期女性出现不规则阴道流血,在排除怀孕和感染的可能后,可以使用口服避孕药物来进行调整。小于9岁和大于52岁没有进行激素替代治疗的女性,若阴道流血,需要慎重考虑和进一步检查。

385、患者女性,25岁,有多位性伴侣,因骨盆痛、发热、阴道分泌物异常和恶心呕吐前来就诊。体检发现Chandelier征阳性。最可能的诊断是A、异位妊娠B、肾盂肾炎C、盆腔炎(PID)D、细菌性阴道炎E、假丝酵母菌阴道炎正确答案:C 解析:盆腔炎是输卵管、子宫颈、子宫内膜或卵巢的感染,常见于30岁以下、性生活活跃或近期有生育的年轻妇女,症状包括骨盆痛、发热、阴道分泌物、性交困难和恶心呕吐。体检发现附件、子宫和宫颈摇摆痛(阳性Chandelier征)。化验检查发现白细胞计数增加,血沉增高。盆腔炎需与以下疾病相鉴别:阑尾炎、异位妊娠、尿道感染和腹膜炎。盆腔炎的并发症有慢性盆腔炎、不育和异位妊娠。治疗使用头孢曲松(罗氏芬)和多西环素,大部分门诊治疗的盆腔炎病人可以用口服抗生素治疗。在接下来的72小时内必须随访,以了解治疗反应。盆腔炎病人入院治疗只有在某些特定情况才推荐。使用避孕套能降低盆腔炎的危险,使用宫内节育器和多个性伴侣则增加其危险。386、36岁女性患者,引产1周后发热伴腹痛2天。体查:全腹软,少量阴道血性分泌物,宫颈举痛(+),子宫增大如孕3个月,宫体压痛,双附件增厚压痛,T39℃,WBC18×109/L,中性粒细胞90%。如本病例妇科捡查发现宫腔左侧有一6cm×6cm×5cm类圆形囊性包块,与子宫壁粘连紧密,压痛阳性,考虑最可能的诊断是A、败血症B、输卵管卵巢脓肿C、急性盆腔腹膜炎D、急性子宫内膜炎E、急性子宫肌炎正确答案:B 解析:结合患者病史、实验室检查,体查全腹软,盆腔有类圆形囊性包块与子宫壁粘连紧密,压痛阳性,最可能为输卵管卵巢脓肿。

387、足月新生儿生理性黄疸多发生于A、生后笫1~2天出现黄疸,10天左右消退B、生后第24小时出现黄疸,3天内进行性加重C、生后第4~7天出现黄疸,10天后消退D、生后第2~5天出现黄疸,10~14天消退E、生后第7天后出现黄疽,呈进行性加重正确答案:D 解析:约60%的足月儿出生后2~3天出现黄疸,黄疸程度较轻,4~5天为高峰,7~10天消退。

388、引起咽结合膜热的病原体是A、流感病毒B、腺病毒C、鼻病毒D、链球菌E、柯萨奇病毒正确答案:B 解析:属记忆题,考查各种特殊类型上呼吸道感染的病原体。上述病原体分别可引起下列疾病:流感、咽结合膜热、急性鼻炎、化脓性扁桃体炎、疱疹性咽峡炎。

389、以下哪项不是小儿肺炎的高危因素A、病毒感染B、营养不良C、先天性心脏病D、生活环境拥挤E、免疫缺陷者正确答案:A 390、关于支气管哮喘,下列叙述哪项是错误的A、儿童期最常见的慢性呼吸道疾病B、多基因遗传性疾病C、与Th1/Th2细胞功能失衡有关D、气道高反应性导致不可逆气道阻塞性疾病E、24小时PEFR变异率大于20%正确答案:D 391、小儿腹泻重型与轻型的主要区别点是A、发热、呕吐B、每天大便超过10次C、有水、电解质紊乱D、大便有黏液、腥臭E、镜检有大量脂肪滴正确答案:C 解析:小儿腹泻重型与轻型的主要区别点有水、电解质紊乱以及全身中毒症状。

392、怀疑肠套叠的病人,可考虑选择以下哪项检查确诊A、腹部CTB、腹部MRIC、腹部B超D、腹部X线平片E、肠镜正确答案:C 解析:腹部B超是诊断肠套叠最重要的无创性方法,肠管横断扫描显示同心圆。

393、急性肾小球肾炎治疗初期使用抗生素的目的是A、治疗疾病本身B、预防复发C、预防并发感染D、彻底清除体内残存细菌E、治疗并发感染正确答案:D 解析:急性肾小球肾炎无特异性治疗,有感染灶时用青霉素彻底清除体内残存细菌,故选D。394、下列哪项不是肾病综合征的并发症A、肾静脉栓塞B、原发性腹膜炎C、低钙抽搐D、低血容量E、循环充血正确答案:E 解析:肾病综合征的并发症包括:感染、电解质紊乱和低血容量、血栓形成、急性肾衰竭、肾小管功能障碍,而循环充血是急性肾小球肾炎的严重表现之一,故选E。

395、下列哪项不支持脑瘫诊断A、运动发育落后B、肌张力异常C、姿势异常D、反射异常E、病变呈进行性加重正确答案:E 解析:脑瘫的定义是非进行性脑损伤。

396、典型化脓性脑膜炎的脑脊液特点A、外观混浊,可呈脓样,蛋白显著增高,糖减少B、清亮至微浊,白细胞正常至数百,以淋巴细胞为主C、糖和氯化物正常,压力升高D、脑脊液墨汁染色阳性E、毛玻璃样,糖和氯化物减少正确答案:A 397、癫痫发作不包括A、强直-阵挛发作B、强直性发作C、失神发作D、精神症状发作E、不典型发作正确答案:D 解析:精神症状发作属于局灶性发作。

398、先天性髋关节脱位复位的正确体位是A、髋关节保持伸直、外展、外旋位B、髋关节保持伸直、内收、内旋位C、髋关节保持屈曲、外展、外旋位D、髋关节保持屈曲、内收、内旋位E、自然体值正确答案:C 解析:先天性髋关节脱位复位的正确体位是髋关节保持屈曲、外展、外旋位,其余体位均为错误,故正确的答案选择应当为C。

399、我国流行性脑脊髓膜炎流行的主要菌群是A、A群B、B群C、C群D、D群E、E群正确答案:A 解析:脑膜炎球菌分为13个血清群,以A、B、C三个群最常见。我国以A群为主,B群有上升趋势。欧美以B、C群为主,A群极少。

400、麻疹前驱期最有诊断价值的表现是A、皮肤红色斑丘疹B、上呼吸道卡他症状C、口腔麻疹黏膜斑D、结膜炎E、高热正确答案:C 解析:麻疹前驱期的表现有发热,上呼吸道炎症,口腔麻疹黏膜斑,而口腔麻疹黏膜斑为该期特别的体征,最有诊断价值。

401、风疹的出疹特点是A、发热3~4小时开始,自耳后、发际、额面、颈出疹,渐及躯干及四肢,最后见于手足心B、发热1~2天开始,最早见面颊部,迅速扩展至躯干和四肢,24小时内布满全身,但手足心通常无皮疹C、高热退出疹D、发热数小时到1天出疹,分批出现,向心性分布,以躯干、头、腰部多见E、玫瑰红色融合性斑丘疹,先分布于鼻翼两侧,1~2天蔓延至全身正确答案:B 解析:A是麻疹的出疹特点,B是风疹的出疹特点,C是幼儿急疹的出疹特点,D是水痘的出疹特点,E是传染性红斑的出疹特点。

402、以下实验室检查结果可以不属于流行性腮腺炎的是A、外周血白细胞计数降低B、外周血白细胞分类检查淋巴细胞相对增高C、血淀粉酶测定轻度至中度增高D、尿淀粉酶测定轻度至中度增高E、脑脊液检查细胞数增高正确答案:A 解析:流行性腮腺炎外周血白细胞计数大多正常或稍高。

403、下列哪项不是猩红热的皮疹特点A、皮肤普遍充血,上有鲜红斑点疹B、皮疹先见于面部C、可有大块脱皮D、可见口周苍白圈E、帕氏线正确答案:B 解析:猩红热的皮疹先见于颈、胸部,面部无皮疹。

404、患儿,女,2岁,因发热2天,咳嗽1日伴腹痛而来诊。查体:T40℃,神志清,扁桃体Ⅱ度肿大,颈软,颈部可及黄豆大淋巴结3~4枚,活动,有压痛,心肺(-),腹稍胀,质软,满腹痛,以脐周为主,无腹肌紧张和同定压痛点,血象Hb128g/L,WBC8.9×109/L,L78%,N22%。可能的诊断是A、急性阑尾炎B、肠套叠C、急性胰腺炎D、梅克尔憩室炎E、上呼吸道感染伴肠系膜淋巴结炎正确答案:E 解析:婴幼儿起病,发热、咳嗽为主要症状,伴腹痛。查体扁桃体肿大,颈部淋巴结肿大,心肺正常,脐周疼痛,但无腹肌紧张和固定压痛点。血象白细胞计数正常,中性粒细胞减少,淋巴细胞计数相对增高。根据病史、体检及实验室检查诊断为上呼吸道感染伴肠系膜淋巴结炎。

405、患儿,女,2岁,因发热2天,咳嗽1日伴腹痛而来诊。查体:T40℃,神志清,扁桃体Ⅱ度肿大,颈软,颈部可及黄豆大淋巴结3~4枚,活动,有压痛,心肺(—),腹稍胀,质软,满腹痛,以脐周为主,无腹肌紧张和同定压痛点,血象Hb128g/L,WBC8.9×109/L,L78%,N22%。进一步治疗措施为A、给予静滴病毒唑+对症处理B、静滴激素+病毒唑C、口服中药D、口服雷尼替丁+颠茄E、只给退热药即可正确答案:A 解析:根据患儿血象考虑病毒感染可能性大。可试用病毒唑治疗,另需及时对症处理:高热给予退热治疗,如高热抽搐,给予镇静、止惊等处理。

406、患儿,男,7岁,反复咳嗽和喘息发作6个月,夜间为甚。体检:肺内哮鸣音和粗湿音,余无异常发现。胸部X线片示肺纹理增多,外周血WBC7×109/L,N0.50,L0.38,E0.12。最可能的诊断是A、免疫功能低下B、肺型肺吸虫病C、支气管异物D、支气管哮喘E、慢性支气管炎正确答案:D 407、13个月龄女孩,腹泻4天入院,每日大便10次左右,蛋花汤样,水分多,伴有呕吐、尿少、轻咳。体检:T38.5℃,前囟、眼窝凹陷,皮肤弹性差,四肢稍凉。血白细胞6.0×109/L,血Na+128mmol/L,K+ 3.6mmol/L,BE-15mmol/L。该患儿在失水、代谢性酸中毒纠正后突然抽搐,此时应做哪项检查A、血钙B、血镁C、脑电图D、脑脊液E、脑部MRI正确答案:A 解析:该患儿在失水、代谢性酸中毒纠正后突然抽搐,此时应查血钙。因为当脱水、代谢性酸中毒纠正后血游离钙可降低,如此时出现抽搐,应考虑低钙血症可能。

408、患儿,女,2岁,流涕伴发热1天,发热38℃左右,今起出疹,由面颊迅速扩展到躯干四肢,到院就诊:T37.8℃,P112次/分,R50次/分。皮疹为红色斑丘疹,颜面、四肢皮疹较稀疏,背部皮疹较密集,有部分融合。咽充血,扁桃体稍大,充血。耳后及颈部数个淋巴结黄豆大小,有压痛,活动。心肺腹无特殊。四肢,关节无异常,神经反射正常。本患儿诊断是A、水痘B、麻疹C、风疹D、幼儿急疹E、风湿热正确答案:C 解析:风疹诊断有发热及卡他症状,耳后、颈后淋巴结肿大,弥漫的斑疹24小时遍及全身。409、关于先天性白内障下列描述中哪项不正确A、其病因可能与遗传、病毒感染、药物有关B、晶状体混浊可表现为点状、冠状、纺锤状C、治疗目的是恢复视力、减少弱视D、都应陔进行手术治疗E、术后应该进行屈光矫正和视力训练正确答案:D 解析:先天性白内障,如为静止性且对视力影响不大者,一般不治疗。

410、并发性白内障的病因不包括A、葡萄膜炎B、视网膜脱离C、青光眼D、低眼压E、远视正确答案:E

411、按晶状体混浊的部位分类,正确描述应当是A、分为点状、冠状和板层白内障B、分为皮质性、核性和囊膜下白内障C、分为老年性、并发性和后发性白内障D、分为外伤性、代谢性和中毒性白内障E、分为先天性和后天获得性白内障正确答案:B

412、下述关于睑腺炎的治疗,错误的描述是A、外睑腺炎的切口须与睑缘垂直,内睑腺炎切口与睑缘平行B、脓肿形成时,应及时切开排脓C、眼睑局部和结膜囊涂抗生素眼膏D、未化脓前可热敷和其他物理治疗E、反复发作及伴有全身反应时可口服抗生素类药物正确答案:A

413、角膜病致盲原因中占首位的是A、单纯疱疹病毒性角膜炎B、细菌性角膜C、真菌性角膜炎D、棘阿米巴性角膜炎E、衣原体性角膜炎正确答案:A

414、耳前淋巴结肿大常见于A、过敏性结膜炎B、急性卡他性结膜炎C、急性流行性出血性结膜炎D、慢性结膜炎E、春季角膜结膜炎正确答案:C

415、高血压视网膜病变2级的眼底表现是A、明显的小动脉狭窄及局部管径不规则B、视网膜小动脉普遍变细,管径均匀,无限局性缩窄C、视网膜出血、渗出和棉絮斑D、视乳头水肿和视网膜水肿E、视网膜新生血管正确答案:A

416、糖尿病性视网膜病变Ⅴ期的表现是A、白色软性渗出或并有出血斑B、黄白色硬性渗出或并有出血斑C、眼底有新生血管、纤维增生和视网膜脱离D、眼底有新生血管和纤维增生E、眼底有新生血管或并有玻璃体积血正确答案:D

417、紫外线损伤主要造成A、虹膜损伤B、视网膜灼伤C、晶状体损伤D、角膜上皮脱落E、视网膜色素上皮损伤正确答案:D

418、眼的屈光系统包括A、角膜、晶状体、玻璃体B、角膜、晶状体、视网膜C、角膜、房水、晶状体、玻璃体、视网膜D、泪膜、角膜、晶状体、玻璃体E、角膜、房水、晶状体、玻璃体正确答案:E

419、屈光不正包括A、近视、远视、散光、老视B、近视、远视、屈光参差C、近视、远视、散光D、近视、远视、散光、屈光参差E、近视、远视、斜视、屈光参差正确答案:D 420、患者女性,48岁,因视力模糊、头痛和l右眼相关的疼痛前来就诊。患者告知能看见光源周围的光圈、恶心、腹痛和呕吐。体检发现右眼充血,瞳孔放大。最可能的诊断是A、闭角型青光眼B、甲亢C、地高辛中毒D、眼前房出血E、阿托品中毒正确答案:A 解析:青光眼分为两种:开角型和闭角型。开角型出现于房水流出率减低和眼压持续上升的情况下,导致视神经萎缩和视力受损。检查可见视盘凹陷和眼内压增加(正常:10~21mmHg)。诊断不能只依靠一次检查读数。治疗为使用β受体阻滞剂如噻吗洛尔和毛果芸香碱。若病人对药物治疗反应不佳,需进行外科手术治疗。闭角型青光眼往往起病较急,由前房狭窄和瞳孔扩大导致房水正常流动受阻引起。这种情况构成眼科急症,和压力、暗室、利用药物致瞳孔扩大以进行眼科检查等相关。大部分病人自觉疼痛、视力模糊、看见灯光周围的光圈,可能出现腹痛和呕吐。体检发现跟睛充血、瞳孔放大、对光没有反应。若不处理,这种情况很快会在2~5天后发展为失明。治疗包括药物(缩瞳药和碳酸酐酶抑制剂)和激光虹膜周边切除术。大于65岁的病人应该每1~2年筛查青光眼;如果有青光眼家族吏,应每年筛查。

421、扁桃体手术适应证正确的是A、慢性扁桃体炎反复发作或多次并发扁桃体周围脓肿者B、单纯扁桃体肥大C、扁桃体恶性淋巴瘤D、慢性扁桃体炎合并严重风湿性心脏病活动期E、以上都对正确答案:A 解析:扁桃体手术适应证包括:①慢性扁桃体炎反复发作或多次并发扁桃体周围脓肿者;②扁桃体重度肥大,妨碍吞咽、呼吸者;③慢性扁桃体炎已成为引起其他脏器病变的病灶,或与邻近器官的病变有关联;④白喉带菌者,经保守治疗无效时;⑤各种扁桃体良性肿瘤,可连同扁桃体一并切除,对恶性肿瘤则应慎重,而严重全身性疾病,如活动性肺结核、风湿性心脏病、关节炎、肾炎、高血压、精神病等是扁桃体手术禁忌证之一。

422、按国际标准,阻塞性呼吸暂停是指在7小时睡眠中,呼吸暂停在A、35次以上B、30次以上C、25次以上D、平均每小时3次以上E、上述都不是正确答案:B 解析:按国际标准,阻塞性睡眠呼吸暂停是指睡眠时鼻腔气流停止流通达10秒或10秒以上,并伴有血氧饱和度下降等,成人每晚的7小时睡眠中呼吸暂停30次以上或平均每小时呼吸暂停5次以上。

423、“星烁征”可见于A、急性化脓性中耳炎B、慢性化脓性中耳炎C、耳硬化症D、大疱性鼓膜炎E、上述都不是正确答案:A 解析:急性化脓性中耳炎初期表现为鼓膜松弛部充血,锤骨柄及紧张部周边可见放射状扩张的血管,渐发展为鼓膜弥漫性充血、肿胀、膨隆,正常标志难以辨识,鼓膜穿孔前局部出现小黄点,然后出现针尖样穿孔,脓液自穿孔处呈搏动样流出,检查时可见闪烁之亮点,称“星烁征”也叫“灯塔征”。

424、某患者,男,15岁,右耳闷,耳聋,查右耳听力为传音性聋,耳镜检查提示右耳鼓室积液,经鼓膜穿刺抽出淡黄色液体1.5ml,初步诊断为分泌性中耳炎,行鼓膜穿刺术更符合下列治疗原则中哪一项A、改善中耳通气引流B、改善咽鼓管功能C、清除中耳积液D、病因治疗E、上述都不是正确答案:C 解析:分泌性中耳炎的治疗原则是:改善中耳通气引流,清除中耳积液和病因治疗。

425、某患者,女,32岁。双侧鼻塞,呈持续性,渐加重,鼻镜检查,见双下鼻甲肥厚,呈桑椹样改变,触之不凹陷,最可能的诊断是A、慢性肥厚性鼻炎B、慢性单纯性鼻炎C、萎缩性鼻炎D、变应性鼻炎E、血管运动性鼻正确答案:A 解析:鼻镜检查为慢性肥厚性鼻炎表现。

426、一位65岁、有长期2型糖尿病病史的病人前来就诊,主诉左耳疼痛并流脓。检查发现,患者左耳耳廓压痛,外耳道肿胀和水肿,无发热,外周白细胞计数正常。最可能导致其流脓的微生物是A、铜绿假单胞菌B、金黄色葡萄球菌C、白色念珠菌D、流感嗜血杆菌E、黏膜炎莫拉菌正确答案:A 解析:一位年长的糖尿病病人耳痛和流脓必须考虑恶性外耳道炎,外耳道肿胀和炎症非常支持这个诊断。这种感染往往发生于年长的糖尿病病人,而且多由铜绿假单胞菌(绿脓杆菌)所致。流感嗜血杆菌和黏膜炎莫拉菌常常导致中耳炎,而非外耳炎。

427、患者女性,54岁,主诉眩晕、耳胀满感、咆哮样或吹气样杂音耳鸣,波动性低音听力损失。最可能的诊断是A、内耳炎B、梅尼埃综合征C、良性体位性眩晕D、血管性偏头痛E、中耳炎正确答案:B 解析:梅尼埃综合征或内淋巴积液的症状有眩晕、耳鸣(就是低音、咆哮或吹气性质)、耳胀满感、低频听力损失。内耳炎常源自病毒感染引起的前庭神经炎症,眩晕往往持续几天,在几周内自行缓解。良性体位性眩晕是由半规管内的耳石引起的,眩晕发作一般持续不到1分钟。中耳炎在成年人中少见,并且通常伴随耳痛,而非眩晕。

428、去除牙齿邻而菌斑的最佳方法是使用A、牙签B、牙线C、牙刷D、漱口水E、以上效果相同正确答案:B

429、正常龈沟深度是A、≤0.5mmB、≤1.0mmC、≤2.0mmD、2.0~3.0mmE、≤3.0mm正确答案:C 430、下列哪项不是慢性龈缘炎的病因A、牙菌斑B、牙石C、食物嵌塞D、不良修复体E、牙齿慢性磨损正确答案:E 解析:牙齿慢性磨损有可能引起牙髓炎,而其他几项均为慢性龈缘炎的病因。

431、与慢性牙周炎发生相关的因素有A、菌斑和牙石B、牙列不齐C、食物嵌塞D、糖尿病E、以上均是正确答案:E

432、复发性口腔溃疡的确切病因是A、细菌感染B、病毒感染C、免疫因素D、精神因素E、尚不清楚正确答案:E

433、下列哪项不是口腔白斑病的病因A、抽烟B、白色念珠菌感染C、嚼槟榔D、维生素A、B族维生素缺乏E、遗传因素正确答案:E 解析:目前尚未发现口腔白斑病有遗传现象。

434、根据临床表现,可将口腔白斑病分为以下几种,除外A、斑块状B、颗粒状C、网格状D、疣状E、溃疡状正确答案:C

435、带状疱疹的病原微生物为A、水痘病毒B、水痘-带状疱疹病毒C、带状疱疹病毒D、人类乳头瘤病毒E、麻疹病毒正确答案:B

436、性传播疾病的预防措施为A、对高危人群主动监测,推广使用安全套B、性病患者应及时、正规治疗,按照要求复查C、同时对患者性伴侣检查及治疗,治愈前做好预防措施D、重视宣传教育E、以上均是正确答案:E

437、青年男性,因左额部及左上眼睑出现疼痛即自涂“驱风油”治疗2天没减轻,之后局部起皮损,疼痛加剧3天而就诊。体检:上述部位可见5小片群集性小水疱,基底淡红,皮损间皮肤正常,排列呈带状,不过中线,局部淋巴结肿大压痛。主要考虑下列哪种疾病A、湿疹B、带状疱疹C、接触性皮炎D、单纯疱疹E、脓疱疮正确答案:B 解析:接触性皮炎发病部位与接触部位应基本一致,与本病表现不符可排除。根据临床特点符合带状疱疹,其余A、D、E均不符合,故选B。

438、患者,女,30岁,阴道分泌物增加,白带呈豆腐渣样,伴有外阴瘙痒,应考虑为以下哪一种疾病A、阴道念珠菌病B、衣原体性宫颈炎C、阴道毛滴虫病D、细菌性阴道病E、淋菌性宫颈炎正确答案:A 解析:白带呈豆腐渣样是阴道念珠菌病特征性表现,其余均无,故选A。

439、一男性患者,43岁,鼻部红斑5年,渐加重。嗜酒。体检:鼻尖部红斑、表面毛细血管扩张、丘疹、脓疱和结节。以下哪项诊断正确A、酒渣鼻红斑期B、结节性痤疮C、毛囊炎D、洒渣鼻丘疹脓疱期E、酒渣鼻鼻赘期正确答案:D 解析:根据临床特点,诊断为酒渣鼻丘疹脓疱期,故选D。

440、一男性患儿,8岁,咽部疼痛5天,自服头孢拉定3天后咽痛有好转,之后全身皮肤出现散在的红色斑丘疹,部分融合成斑片,自觉微痒。体检:上述可见绿豆大小红色斑丘疹,表面可见银白色鳞屑,鳞屑易刮除,刮除鳞屑后可见半透明薄膜,点状出血。应考虑为A、药疹B、急性点滴状银屑病C、湿疹D、玫瑰糠疹E、扁平苔癣正确答案:B 解析:根据特征性皮损(银白色鳞屑、蜡滴现象、薄膜现象、点状出血)及诱因(上呼吸道感染),诊断为急性点滴状银屑病,故选B。

441、社交恐惧症不具有的临床特征是A、好发年龄17~30岁B、回避会暴露于陌生人的场景C、出现脸红或发抖等自主神经反应D、思维奔逸、散漫E、时常伴有抑郁的情绪正确答案:D 解析:思维奔逸、散漫为躁狂症的临床特征。社交恐惧症除回避、自主神经紊乱外,可以伴有抑郁的情绪。

442、患者在听到脚步的声音的同时听到有人议论他的声音,这种症状属于A、思维化生B、听错觉C、反射性幻觉D、功能性幻觉E、心因性幻觉正确答案:D 解析:功能性幻觉是伴随现实刺激而出现的幻觉。即当某种感觉器官处于功能活动状态时出现涉及该器官的幻觉,正常的知觉与幻觉并存。

443、一老年男性患者,高热后出现反应迟钝,注意力、记忆、理解都有困难,有环境定向障碍,能简单回答问题,但对复杂问题则有些茫然不知所措,吞咽、角膜、对光反射尚存在,这种症状属于A、嗜睡B、意识混浊C、昏睡D、昏迷E、朦胧状态正确答案:B 解析:意识混浊是意识清晰度轻度受损,表现为似醒似睡,缺乏主动,行动迟缓。如果缺乏刺激,常像要睡着一样,但还能有一些简单的动作和反应,例如张口、伸舌等,他都能做,只是缓慢些。跟环境有一些接触,能回答简单问题,但对复杂问题则茫然不知所措,语音低,语速慢。整个精神活动均显得十分呆滞。见于癫痫、颅内压增高、颅脑创伤或感染中。444、在缺乏相应客观因素的情况下,患者表现为顾虑重重、紧张恐惧,以致搓手顿足,似有大祸临头,惶惶不可终日,伴有心悸、出汗、手抖、尿频等自主神经功能紊乱症状,这属于A、情感不稳定B、焦虑C、易激惹D、情感幼稚E、情感低落正确答案:B 445、消炎镇痛药阿司匹林最常见的不良反应是A、胃肠道反应B、凝血障碍C、过敏反应D、水杨酸反应E、Reye综合征正确答案:A 解析:胃肠道反应是阿司匹林最为常见的不良反应。

446、下列哪个部位不是压疮的好发部位A、坐骨结节B、骶部C、足跟D、背部E、股骨大转子正确答案:D 解析:压疮的好发部位是坐骨结节、骶部、足跟、股骨大转子。

447、受压局部深层皮肤有破坏,累及真皮的压疮属于A、Ⅰ度B、Ⅱ度C、Ⅲ度D、Ⅳ度E、Ⅴ度正确答案:B 解析:累及真皮的压疮属于Ⅱ度压疮。448、目前公认的预防和治疗肥厚性瘢痕最有效的方法是A、压力治疗B、硅胶治疗C、手术治疗D、放射治疗E、激光治疗正确答案:A 解析:压力治疗是目前公认最有效的预防与治疗肥厚性瘢痕的方法。

449、感觉消失的部位对温热治疗应A、禁忌温热疗法B、温度稍低,避免烫伤C、升高温度,增加治疗效果D、升高温度稍超过其痛阈E、控制在正常皮肤能耐受的温度正确答案:B 解析:感觉障碍的部位血液循环减慢,局部不易散热,容易导致烫伤。因此,选择的温度应较一般治疗时偏低。

450、足下垂伴足内翻最多见于A、脑卒巾偏瘫患者B、高位脊髓损伤患者C、低位脊髓损伤患者D、脑外伤患者E、脊髓灰质炎后遗症患正确答案:A 解析:足下垂伴足内翻是最常见的神经瘫痪步态异常。其中最常见的是脑卒中后。

451、男性,45岁,右手远侧指间关节疼痛、肿胀、有晨儡感,活动后可逐渐消失,此患者临床诊断可能性最大的是A、类风湿关节炎B、骨性关节炎C、牛皮癣性关节炎D、关节结核E、梅毒性关节炎正确答案:B 解析:远侧指间关节是骨性关节炎的好发部位。

452、老年男性,脑卒中后右侧肢体瘫痪,查体:右上肢肌张力轻度增加,肘关节被动屈伸时,在关节活动范围之末时出现最小的阻力,按修订的Ashworth痉挛分级标准,患者应坪为A、0级B、1级C、2级D、3级E、4级正确答案:B 解析:按修订的Ashworth痉挛分级标准,被动屈伸时,在关节活动范围之末时出现最小的阻力或出现突然卡住和释放,为1级。

453、女性患者,40岁。车祸伤致右:赶腿肿、痛、活动障碍2小时,急诊X线检查示右股骨中段斜行骨折,入院后立即行骨牵引治疗。牵引时不可以进行A、患侧髌骨的被动运动B、患侧跟腱的被动牵拉练习C、患侧股四头肌的等张收缩练习D、患侧踝关节的主动背屈和跖屈练习E、腹式呼吸运动练正确答案:C 解析:股骨中段骨折时,若行殷四头肌等张收缩练习,由于产生关节活动,可能引起骨折断端的位移,从而加重骨折的伤情。454、“解表法”又称为“汗法”,其功效不包括A、祛湿B、解表C、消肿D、温中E、透疹正确答案:D 解析:解表法即运用解表发汗药物,通过开泄腠理,使表证随汗出而解的治法。通过发散,可祛除表邪、透发疹毒、祛风除湿、祛水外出而消肿。

455、下列哪种病证可用“下法”A、燥屎停结B、邪在表者C、津枯便秘D、阳虚便秘E、久病正虚正确答案:A 解析:下法是通过通便、下积、泻实、逐水,以消除燥屎、积滞、实热及水饮等证的治法。但是,凡邪在表、老年津枯便秘、阳气衰微而大便艰难者,不宜用下法。456、薄荷入汤剂宜A、先煎B、后下C、包煎D、烊化E、冲服正确答案:B 解析:薄荷含挥发油,其主要成分是薄荷醇和薄荷酮,后下以防止其挥发油过分损失。457、咳嗽初期,最易“闭门留寇”的药是A、苦寒药B、温补药C、收涩药D、镇咳药E、通下药正确答案:C 458、吐酸寒证的治疗宜用A、泻热清肝B、泻热和胃C、温养脾胃D、调和脾胃E、补益心脾正确答案:C 459、治疗行痹的首选方是A、桂枝芍药知母汤B、桂枝汤C、防风汤D、麻黄汤E、越婢汤正确答案:C 460、患者男性,4岁,食少泄泻,面黄肌瘦,腹胀,睡时露睛,予肥儿丸口服,其作用是A、化痰导滞B、健脾消食C、化湿和胃D、养阴生津E、疏肝健脾正确答案:B 解析:小儿脾胃虚弱,消化不良,则食少泄泻,面黄肌瘦,腹胀,睡时露睛。肥儿丸可健脾、消食、杀虫,故本题正确答案是B。

461、患者,女,40岁,发热恶寒,肢节酸痛,头痛,咳嗽轻微,咳吐白稀痰,苔薄白,脉浮。临床上最可能的诊断为A、风寒感冒B、风热感冒C、时行感冒D、气虚感冒E、阴虚感冒正确答案:A 462、肿瘤相关抗原的特点是A、只存在于肿瘤细胞B、细胞癌变时含量明显增高C、有严格的肿瘤特异性D、肿瘤相关抗原仅存存于血清中E、仅以膜结合形式存在正确答案:B 解析:肿瘤相关抗原是指非肿瘤所特有的、正常细胞和其他组织上也存在的抗原,只是其含量在细胞癌变时明显增高,此类抗原只表现为量的变化而无严格的肿瘤特异性。

463、导致白细胞计数生理性减少的因素包括A、运动B、妊娠C、吸烟D、情绪激动E、休息正确答案:E 解析:白细胞计数在安静和休息时数量较低,在运动、疼痛、情绪激动时可增高,妊娠时白细胞可增高达15×109/L以上,吸烟者平均自细胞计数可高于非吸烟者30%。

464、中性粒细胞减少可见于A、急性化脓性胆囊炎B、伤寒C、心肌梗死D、消化道大出血E、败恤症正确答案:B 解析:某些革兰阴性杆菌如伤寒、副伤寒杆菌感染时,如无并发症,中性粒细胞计数均减少,在其他病理性情况如再生障碍性贫血,接触放射线等,中性粒细胞也可减低。465、下列血红蛋白(Hb)检测结果符合女性中度贫血的是A、30g/LB、50g/LC、80g/LD、95g/LE、105g/L正确答案:C 解析:根据血红蛋白(Hb)浓度,成人贫血程度划为4级。轻度:Hb参考值下限至91g/L;中度:Hb90~60g/L;重度:Hb60~31g/L;极重度:Hb≤30g/L。

466、原发性糖尿病(真性)的主要病因是A、胰岛素分泌量相对或绝对不足B、甲状腺功能亢进C、肾上腺皮质功能亢进D、肾小管对糖的再吸收能力减低E、腺垂体功能亢进正确答案:A 解析:原发性糖尿病包括1型糖尿病和2型糖啄病,其主要原因是胰岛素兮泌的绝对不足或相对不足。

467、下列试验中血标本受溶血影响最小的是A、钾离子B、葡萄糖C、LDHD、ASTE、ACP正确答案:B 解析:待测物在红细胞内的浓度高于血浆时,溶血可使测定结果偏高,有些物质如LDH、ACP、AST、K+等在红细胞内的浓度比血浆高22~160倍,轻微溶血都对结果影响很大;血红蛋白可干扰胆固醇的酶法测定,抑制胆红素的重氯反应等;溶血也干扰某些光谱分析。因此应尽量避免溶血。

468、临床中ABO血型鉴定最常采用的方法为A、正向间接凝集反应B、反向间接凝集反应C、玻片凝集法D、试管凝集法E、间接凝集抑制反应正确答案:C 解析:ABO血型鉴定最常采用的方法是玻片凝集法。

469、下列疾病均有血清尿素氮增商而血清肌晰正常,但不包括A、慢性肾衰竭尿毒症B、急性传染病C、上消化道出血D、大面积烧伤E、甲状腺功能亢进正确答案:A 解析:甲状腺功能亢进、大面积烧伤及急性传染病时蛋白质分解代谢增强,啄素氯合成增加;上消化道出血时由于大量血液蛋白质的消化产物在肠道被吸收,血中尿素氮浓度可暂时性升高,以上几种情况均只有血清尿素氮浓度增高而肌酐不增高。慢性肾衰竭时血清尿素氮和肌酐均升高。

470、防止微生物进入机体或其他物品的操作技术被称为A、消毒B、防腐C、灭菌D、无菌E、无菌操作正确答案:E 解析:防止微生物进入机体或其他物品的操作技术即为无菌操作。471、急性心肌梗死(AMI)的早期标志物是A、ASTB、LDC、肌红蛋hD、CK-MBE、肌钙蛋白正确答案:C 解析:肌红蛋白在AMI发作后0.5~2小时内即升高,被确定为AMI的早期标志物。472、鉴别良忡肿瘤和恶性肿瘤最重要的是A、生长速度快慢B、局部有无疼痛C、X线片上肿块边缘是否清楚D、病理组织捡查E、局部皮肤有无发热正确答案:D 473、有关恶性肿瘤的临床表现,下列哪项是错误的A、疼痛为初发症状B、常因坏死、血形成溃疡C、局部不一定扪及肿块D、可出现淋巴道和血道转移E、消瘦、乏力、发热常为晚期表现正确答案:A 474、下列哪一项不是MRI检查的禁忌证A、置有心脏起搏器B、有碘过敏史C、枪炮伤后弹片存留D、体内有胰岛素泵E、颅脑手术后动脉夹存留正确答案:B 475、X线平片不能显示小儿长骨的A、骨干B、骨骺骨化中心C、骺板和骺软骨D、干骺端E、骨髓腔正确答案:C 476、患者男性,58岁,单位体检X线胸片示右上肺第2前肋间见直径2cm结节病灶,边界清,密度均匀,病灶周围有少许条索影;CT显示右上肺结节病灶无强化,肺窗结节病灶周围见片状、条索影。右肺余肺野、左侧肺野清晰,两侧肺门未见异常。最可能的诊断是A、右上肺肺炎B、右上肺肺癌C、右上肺结核D、右上肺错构瘤E、右上肺支气管扩张正确答案:C 477、左室增大需考虑的疾病是A、主动脉关闭不全B、二尖瓣关闭不全C、扩张型心肌病D、心衰E、以上均是正确答案:E 478、年轻女性双侧乳房经前胀痛,经后可缓解,超声检查显示乳腺结构紊乱,同声分布不均,呈粗大光点、光斑。最可能是A、乳腺增生B、乳腺炎C、乳腺纤维瘤D、乳腺囊肿E、乳腺脓肿正确答案:A 479、老年女性患者,原有糖尿病,右上腹隐痛数日,突发腹痛弥漫加剧。超声表现:胆囊偏小,壁厚毛糙,胆汁少,透声差,囊内见多枚强回声团后伴声影,胆囊周围见液暗区,可诊断为A、胆结石伴急性胆囊炎B、胆囊结石伴胆囊癌C、坏疽性胆囊炎D、胆囊结石胆囊炎伴胆囊穿孔E、胆囊结石伴慢性胆囊炎正确答案:D 480、关于心电图波形的含义,下列哪项是错误的A、P波代表心房除极波B、QRS波代表心室除极波C、T波代表心房复极波D、T波代表心室复极波E、P-R间期代表心房除极终点至心室除极开始的间正确答案:E 解析:P-R(Q)间期是从P波起点至心室除极开始这段时间,不论心室开始是否有Q波还是R波,统称P-R间期。它代表心房肌开始除极至心室肌开始除极的这一段时间。

481、关于正常窦性P波方向的描述是A、Ⅰ、Ⅱ、aVL、V1导联倒置,aVR正负双向B、Ⅰ、Ⅲ、avIL、V5到V6导联倒置,aVR导联直立C、Ⅱ、Ⅲ、aVF、V1负正双向,aVR导联直立D、Ⅰ、Ⅱ、aVF、V3到V6导联直立,aVR导联倒置E、Ⅰ、Ⅱ、Ⅲ、V1到V6导联倒置,aVR导联倒置正确答案:D 解析:正常窦性P波的方向应该是Ⅰ、Ⅱ、aVF、V3到V6导联直立,aVR导联倒置。Ⅱ导联P波直立,aVR导联倒置,即为寞性心律。

482、关于ST段位移幅度描述错误的是A、ST段下移任何导联≯0.05mVB、ST段上升V1、V2≯0.3mVC、ST段上升V3≯ 0.5mVD、ST段上升V4~V6及肢导联均≯0.1mVE、以上均不是正确答案:E 解析:ST段下移在任何导联≯0.05mV,ST段上升在V1、V2≯0.3mV,V3≯0.5mV,V4~V6及肢导联均≯0.1mV。ST段异常偏移,可以是心肌缺血、损伤的心电图表现。483、描述常规心电图导联应包括A、肢体导联和单极加压肢体导联B、胸前导联V1~V3C、肢体导联和单极加压肢体导联+V3R、V4R、V5RD、Ⅰ、Ⅱ、Ⅲ、aVR、aVL、aVF、V1~V6E、V1~V9正确答案:D 解析:常规心电图为标准导联应包括Ⅰ、Ⅱ、Ⅲ、单极加压肢体导联aVR、aVL、avF、胸前导联V1~V6。484、24小时动态心电图有效记录时间应该是A、≥20小时B、≤20小时C、≥22小时D、≥24小时E、<24小时正确答案:C 解析:中华医学会心电生理和起搏分会电生理学组制定的动态心电图工作的建议指出:动态心电图24小时连续监测,剔除伪差和干扰,有效记录时间不应少于22小时。

485、风心病主动脉瓣狭窄患者伴随的胸痛与下述哪一项有关A、炎症刺激B、缺血、缺氧C、肌张力改变D、肿瘤浸润E、物理、化学因子正确答案:B 解析:风心病主动脉瓣狭窄患者伴随的胸痛其产生的机制可能与心肌缺氧、耗氧量增大、左室收缩期室壁张力过高有关。

486、大叶性肺炎典型患者咳何种痰A、黄痰B、白色泡沫痰C、铁锈色痰D、胶冻状痰E、黯红色黏稠血痰正确答案:C 解析:大叶性肺炎典型患者咳铁锈色痰。487、引起长期慢性咳嗽的疾病有A、百日咳B、气管肿瘤C、支气管异物D、急性咽喉炎E、慢性支气管炎正确答案:E 解析:常见引起慢性咳嗽的疾病有:慢性咽喉疾病、慢性支气管疾病、慢性肺部疾病等。慢性支气管炎咳嗽的特点是:咳嗽伴大量的痰液咳出,以早晨为主,伴有急性感染时痰量增多且呈脓性,颜色变黄。慢性支气管炎常有两年以上的病史,每年咳嗽持续3个月以上。其治疗首先要戒烟,加强锻炼,增强体质,减少呼吸道感染的发作次数。

488、引起金属音调咳嗽的疾病有下列哪种A、喉肿瘤B、喉炎C、声带炎D、喉结核E、纵隔肿瘤正确答案:E 解析:金属音调咳嗽,见于纵隔肿瘤、原发性支气管肺癌、主动脉瘤等压迫气管时。

489、在我国,咯血的主要原因首推A、肺梗死B、肺结核C、肺炎D、矽肺E、恶性肿瘤转移正确答案:B 解析:在我国肺结核是最常见的咯血原因之一。

490、下列疾病中,哪一种咳黏稠黯红色血痰A、心绞痛B、心包炎C、肺梗死D、心肌梗死E、心包积液正确答案:C 解析:肺栓塞是指嵌塞物质进入肺动脉及其分支,阻断组织血液供应所引起的病理和临床状态。慢性肺梗死可有咯血,咳黏稠黯红色血痰等症状。

491、下列引起心悸的病因中,哪一项是由心脏搏出量增加所致A、饮用浓茶后B、口服阿托品后C、低血糖时D、心房颤动发作时E、剧烈运动时正确答案:C 解析:在引起心悸的病因中,由心排血量增加的病变可见低血糖发作。低血糖的表现多种多样,主要有交感神经兴奋症状和精冲症状。交惑神经兴奋症状表现为:饥饿感、心悸、出汗、头痛、视物不清等。精神症状表现为发呆、答非所问、精神不安、意识不清、昏迷等。492、下列哪种疾病可出现心悸伴晕厥或抽搐A、风湿热B、心肌炎C、心包炎D、感染性心内膜炎E、病态窦房结综合征正确答案:E 解析:病态窦旁结综合征临床表现轻重不一,部分患者合并短阵室上性快速心律失常发作,又称慢快综合征。快速心律失常发作时,心率可突然加速达100次/分以上,持续时间长短不一,心动过速突然中止后可有心脏暂停伴或不伴晕厥发作。严重心动过缓或心动过速引起的心悸伴晕厥或抽搐。

493、上消化道出血的患者,其出血部位一般不低于A、贲门B、幽门C、十二指肠球部D、屈氏韧带水平E、空肠下端正确答案:D 解析:呕血是上消化道疾病(指屈氏韧带以上的消化器官,包括食管、胃、十二指肠、肝、胆、胰疾病)或全身性疾病所致的急性上消化道出血,血液经口腔呕出。消化道出血患者有呕血者,其出血部位一般不低干屈氏韧带水平。

494、中老年人呕血伴反复发作的、无明显规律的上腹痛并伴有厌食及消瘦者,应考虑A、胃癌B、慢性胃炎C、十二指肠炎D、消化性溃疡E、急性胃黏膜病变正确答案:A 解析:胃癌早期症状常不明显,随着病情的进展,出现无明显规律的上腹部疼痛、食欲不振、消瘦、体重减轻和贫血等。肿瘤表面溃疡时出现呕血、黑便等症状,后期常有癌细胞转移,出现腹部肿块、左锁骨上淋巴结肿大、黑便、腹水及严重营养不良等。

495、上消化道出血除呕血外常出现黑便,其机制是A、血红蛋白与肠内硫化物结合形成硫化铁B、血红蛋白与肠内硫化物结合形成硫化亚铁C、血红蛋白与肠内细菌分泌的氨基酸氧化酶结合D、血红蛋白与肠内的肠激酶结合E、血红蛋白与肠内的黏液结合正确答案:B 解析:消化道出血,若血液在肠内停留时间较长,红细胞破坏后,血红蛋白在肠道内与硫化物结合形成硫化亚铁,使粪便呈黑色,由于附有黏液而发亮,类似柏油样,称为柏油样便。496、常引起便血的结肠疾病是A、肠伤寒B、Crohn病C、肠结核病D、急性细菌性痢疾E、急性出血性坏死性肠炎正确答案:D 解析:急性细菌性痢疾起病急,表现为畏寒、发热、腹痛、腹泻,排黏液脓血便伴里急后重。左下腹压痛明显。

497、中度昏迷与深昏迷最有价值的鉴别是A、不能唤醒B、大小便失禁C、无自主运动D、深浅反射均消失E、各种刺激无反应正确答案:D 解析:中度昏迷对痛刺激的反应消失,生理反应存在,生命体征正常。而深昏迷除生命体征存在外,其他均消失。

498、患者需要强烈刺激才能唤醒,醒时答非所问、反应迟钝,这种意识状态属于A、嗜睡B、昏睡C、浅昏迷D、意识模糊E、深度昏迷正确答案:B 解析:昏睡患者处于熟睡状态,不易被唤醒。须在强烈刺激下(如压迫眶上神经、摇动患者等)可被唤醒,很快又再入睡。醒时答话含糊或答非所问。499、结合胆红素进入胆道后还原为尿胆原,在肠内被吸收入肝的比例是A、1%~5%B、5%~10%C、10%~20%D、20%~25%E、25%~30%正确答案:C 解析:结合胆红素进入肠腔后,由肠道细菌脱氢的作用还原为尿胆原,大部分随粪便排出,称为粪胆原;小部分(10%~20%)经回肠下段或结肠重吸收,通过门静脉血回到肝,转变为胆红素;或未经转变再随胆汁排入肠内,这一过程称为胆红素的“肠肝循环”。从肠道重吸收的尿胆原,有很少部分(每日不超过6.8/μmol/L)进入体循环,经肾排出。

500、血清总胆红素(TB)的正常参考值为A、0.7~1.7μmol/LB、1.5~15.1μmlol/LC、1.7~l7.1μmol/LD、17~34μmol/LE、34~170μmol/L正确答案:C 解析:血清总胆红素(TB)的正常参考值1.7~17.1μmol/L(0.1~1.0mg/dl),结合胆红素(CB)0~3.42μmol/L(0~0.2mg/dl),非结合胆红素(UCB)1.7~13.68μmol/L(0.1~0.8mg/dl)。

下载全科副主任医师考试试题要点word格式文档
下载全科副主任医师考试试题要点.doc
将本文档下载到自己电脑,方便修改和收藏,请勿使用迅雷等下载。
点此处下载文档

文档为doc格式


声明:本文内容由互联网用户自发贡献自行上传,本网站不拥有所有权,未作人工编辑处理,也不承担相关法律责任。如果您发现有涉嫌版权的内容,欢迎发送邮件至:645879355@qq.com 进行举报,并提供相关证据,工作人员会在5个工作日内联系你,一经查实,本站将立刻删除涉嫌侵权内容。

相关范文推荐

    全科医学试题

    第一部分 全科医学基础: 一、单选题1.全科医学学科是 A 自20世纪60年代起源的新型二级临床专业学科 D 强调使用相对简便而有效的手段解决社区居民大部分健康问题 E 强调在......

    乡村全科执业助理医师考试试题

    乡村全科执业助理医师考试试题 A1A2型题 1.关于脊柱外伤与脊髓损伤的关系的叙述,下列哪项是错误的() A.脊髓损伤节段与椎骨受伤平面不一致 B.胸椎较固定,所以胸椎骨的脱位多无......

    西藏2016年下半年全科主治医生考试试题

    西藏2016年下半年全科主治医生考试试题 一、单项选择题(共 25题,每题2分,每题的备选项中,只有1个事最符合题意) 1、关于急性上消化道出血,下述哪项说法是不正确的 A:出血量超过50~1......

    当阳市全科公共卫生医师团队考试试题

    当阳市全科公共卫生医师团队考试试题 1. 基本公共卫生服务项目包括:建立居民健康档案;健康教育;预防接种;传染病防治;高血压、糖尿病等慢性病管理、儿童保健、孕产发妇保健、老年......

    广东省全科医学试题.

    1. 单项选择题2.(1分)1. 牙髓治疗的方法如下,除外 A. 盖髓术 B. 龈下刮治术 C. 根管治疗术 D. 牙髓塑化法 E. 干髓术 正确答案:B 3. 4.(1分)2. 免疫功能低下病人的医院感......

    社区全科考试-药学

    单选题 11、11.实践循证医学实践步骤有 A、一步 1、1.社区药学岗位人员培训的基本技能不包括 B、两步 A、审方C、三步 B、调配D、四步 C、发药E、五步 D、药物咨询 E、药品......

    口袋妖怪全科混合考试[精选]

    口袋妖怪全科混合考试 考试注意事项: ⒈请大家用2【B】铅笔填涂“达”题卡,之后用高级黑色中性笔将答案腾抄到试卷上。 ⒉请勿在试卷和“达”题卡上透露自己的真实信息,以免被......

    西藏2015年下半年初级主治医师(全科)考试试题

    西藏2015年下半年初级主治医师(全科)考试试题 一、单项选择题(共 25题,每题2分,每题的备选项中,只有1个事最符合题意) 1、全科医生应诊的四大任务不包括 A:确认并处理现存问题 B:......